Hall and Chantigan Chapter 6 – General Anesthesia – Flashcards

question
418. A 78-year-old patient with a history of hypertension and adult-onset diabetes for which she takes chlorpropamide (Diabinese) is admitted for elective cholecystectomy. On the day of admission, blood glucose is noted to be 270 mg/dL, and the patient is treated with 15 units of regular insulin subcutaneously (SQ) in addition to her regular dose of chlorpropamide. Twenty-four hours later after overnight fasting, the patient is brought to the operating room (OR) without her daily dose of chlorpropamide and is anesthetized. A serum glucose is measured and found to be 35 mg/dL. The most likely explanation for this is A. Insulin B. Chlorpropamide C. Hypovolemia D. Effect of general anesthesia E. It is a normal finding in fasting patients
answer
418. (B) Patients with insulin-dependent diabetes and noninsulin-dependent diabetes require special consideration when presenting for surgery. Geriatric age patients come to the OR in the fasting state and without having taken their morning dose of their oral diabetic agent. Chlorpropamide is the longest-acting sulfonylurea and has a duration of action up to 72 hours. Accordingly, it is prudent to measure serum glucose before inducing anesthesia and periodically during the course of the anesthetic and surgery. Regular insulin has a peak effect 2 to 3 hours after SQ administration and a duration of action approximately 6 to 8 hours and would therefore not cause a serum glucose of 35 mg/dL 24 hours after it was administered (Stoelting: Pharmacology and Physiology in Anesthetic Practice, ed 4, pp 479, 483-484).
question
419. Select the true statement. A. Dibucaine is an ester-type local anesthetic B. A dibucaine number of 20 is normal C. The dibucaine number represents the quantity of normal pseudocholinesterase D. Neuromuscular blockade with succinylcholine would last several hours in a patient with a dibucaine number of 80 E. None of the above
answer
419. (E) Dibucaine is an amide-type local anesthetic that inhibits normal pseudocholinesterase by approximately 80%. In patients who are heterozygous for atypical pseudocholinesterase, enzyme activity is inhibited by 40% to 60%. In patients who are homozygous for atypical pseudocholinesterase, enzyme activity is inhibited by only 20%. The dibucaine number is a qualitative assessment of pseudocholinesterase. Quantitative as well as qualitative determination of enzyme activity should be carried out in any patient who is suspected of having a pseudocholinesterase abnormality (Stoelting: Basics of Anesthesia, ed 5, p 140).
question
420. A 56-year-old patient with a history of liver disease and osteomyelitis is anesthetized for tibial débridement. After induction and intubation, the wound is inspected and débrided with a total blood loss of 300 mL. The patient is transported intubated to the recovery room, at which time the systolic blood pressure falls to 50 mm Hg. Heart rate is 120 beats/min, arterial blood gases (ABGs) are PaO2 103, PaCO2 45, pH 7.3, with 97% O2 saturation with 100% FIO2. Mixed venous blood gases are PvO2 60, PaCO2 50, pH 7.25, with 90% O2 saturation. Which of the following diagnoses is most consistent with this clinical picture? A. Myocardial infarction B. Congestive heart failure C. Cardiac tamponade D. Sepsis with acute respiratory distress syndrome E. Hypovolemia
answer
420. (D) All hypotension can be broadly broken down into two main categories: decreased cardiac output and decreased systemic vascular resistance. Flow or cardiac output can be further subdivided into problems related to decreased heart rate (i.e., bradycardia versus problems related to decreases in stroke volume). Normal PO2 in mixed venous blood is 40 mm Hg. Increased mixed venous arterial oxygen levels can be due to many conditions including high cardiac output, sepsis, left-to-right cardiac shunts, impaired peripheral uptake (e.g., cyanide), decreased oxygen consumption (e.g., hypothermia) as well as sampling error. The other choices in this question all represent conditions whereby cardiac output is diminished and consequently would not be consistent with the data given in the question (Morgan: Clinical Anesthesiology, ed 4, pp 560- 561).
question
421. Normal tracheal capillary pressure is A. 5 to 15 mm Hg B. 15 to 25 mm Hg C. 25 to 35 mm Hg D. 35 to 45 mm Hg E. 45 to 55 mm Hg
answer
421. (C) Tracheal capillary arteriolar pressure (25-35 mm Hg) is important to keep in mind in patients who are intubated with cuffed endotracheal tubes. If the endotracheal tube cuff exerts a pressure greater than capillary arteriolar pressure, tissue ischemia may result. Persistent ischemia may lead to destruction of tracheal rings and tracheomalacia. Endotracheal tubes with low-pressure cuffs are recommended in patients who are to be intubated for periods longer than 48 hours because this will minimize the chances for development of tissue ischemia (Miller: Anesthesia, ed 6, p 1630).
question
422. How many hours should elapse before performing a single-shot spinal anesthetic in a patient who is receiving 1 mg/kg enoxaparin twice a day for the treatment of a deep vein thrombosis? A. 2 hours B. 6 hours C. 12 hours D. 18 hours E. 24 hours
answer
422. (E) Enoxaparin, dalteparin, and ardeparin are low-molecular-weight heparins (LMWH). Because of the possibility of spinal and epidural hematoma in the anticoagulated patient with neuraxial blockade, caution is advised. The plasma half-life of LMWH is two to four times longer than standard heparin. These drugs are commonly used for prophylaxis for deep vein thrombosis. These drugs are also used at high doses for treatment of deep vein thrombosis and (off label) as "bridge therapy" for patients chronically anticoagulated with Coumadin. In these patients who are being prepared for surgery, Coumadin is discontinued and LMWH started. With high-dose enoxaparin administration (1 mg/kg twice daily) it is recommended to wait at least 24 hours before administration of a single-shot spinal anesthetic (Barash: Clinical Anesthesia, ed 5, p 713; Miller: Anesthesia, ed 6, pp 1677, 2742-2743; Second Consensus Conference on Neuraxial Anesthesia and Anticoagulation, April 25-28, 2002 www.asra.com/consensus-statements/2.html ).
question
423. Which of the following peripheral nerves is most likely to become injured in patients who are under general anesthesia? A. Ulnar nerve B. Median nerve C. Radial nerve D. Common peroneal nerve E. Sciatic and peroneal nerve
answer
423. (A) The principal mechanism of peripheral nerve injury is ischemia caused by stretching or compression of the nerves. Anesthetized patients are at increased risk for peripheral nerve injuries because they are unconscious and unable to complain about uncomfortable positions that an awake patient would not tolerate and because of reduced muscle tone that facilitates placement of patients into awkward positions. The ulnar nerve in particular is vulnerable because it passes around the posterior aspect of the medial epicondyle of the humerus. The ulnar nerve may become compressed between the medial epicondyle and the sharp edge of the operating table, leading to ischemia and possible nerve injury which may be transient or permanent (Stoelting: Basics of Anesthesia, ed 5, pp 299-300).
question
424. Renal failure associated with fluoride toxicity anesthesia most closely resembles A. Papillary necrosis B. Acute tubule necrosis C. Hepatorenal syndrome D. Central diabetes insipidus E. Nephrogenic diabetes insipidus
answer
424. (E) Methoxyflurane is extensively metabolized resulting in the liberation of the fluoride anion. When the inorganic fluoride levels are less than 50 µmol/L no evidence of renal injury is seen. With levels of 50 to 80 µmol/L (which develops with 2.5-3 MAC hours of methoxyflurane use) moderate injury occurs and with levels of 80 to 120 µmol/L severe injury develops. Several patients have died when levels were above 120 µmol/L. Fluoride in appropriate concentrations is capable of making the kidney unresponsive to ADH. This condition is known as nephrogenic diabetes insipidus and is the reason why methoxyflurane was withdrawn from clinical practice. Enflurane (now obsolete in the United States) also is capable of yielding free fluoride ions, but at levels much less than with methoxyflurane. Sevoflurane is defluorinated through oxidative metabolism and can produce serum fluoride levels that can peak above 50 µmol/L, although typically levels are commonly around 30 µmol/L. Renal failure has not been clinically seen with sevoflurane use, possibly because the drug is rapidly excreted via the lungs (methoxyflurane is very slowly excreted via the lungs). Very small amounts of free fluoride ions are produced through the metabolism of isoflurane and halothane. Desflurane is very resistant to defluorination and is not associated with nephrotoxicity (Miller: Anesthesia, ed 6, pp 248-251).
question
425. A 45-year-old obese male is in the intensive care unit (ICU) after an elective open lung biopsy. Which of the following would provide the best prophylaxis against deep vein thrombosis in this patient? A. Pneumatic compression boots B. Heparin 5000 units SQ every 8 hours C. Early ambulation D. Dextran 10 mL/kg IV during surgery E. Incentive spirometry
answer
425. (C) The incidence of deep vein thrombosis can be reduced from 30% to less than 10% in patients undergoing thoracoabdominal surgery if low-dose heparin (5000 units) is administered 2 hours before surgery and every 8 to 12 hours thereafter (until the patient is able to walk). Although the reduction in deep vein thrombosis in these patients is clear, it is not certain if this therapy prevents pulmonary embolism or reduces mortality. Aspirin, Coumadin, dextran, and compression boots may be of benefit in specific clinical situations. Early ambulation, however, is the best prophylaxis against deep vein thrombosis. Coughing does nothing to prevent deep vein thromboses (Hines: Stoelting's Anesthesia and Co-Existing Disease, ed 5, pp 155-157)
question
426. A patient with which of the following eye diseases would be at greatest risk for retinal damage from hypotension during surgery? A. Strabismus B. Cataract C. Glaucoma D. Severe myopia E. Open eye injury
answer
426. (C) Blood flow to the retina can be decreased by either a decrease in mean arterial pressure or an increase in intraocular pressure. Decreased blood flow and stasis are more likely in patients with glaucoma because of their elevated intraocular pressure. During periods of prolonged hypotension, the incidence of retinal artery thrombosis increases in these patients (Hines: Stoelting's Anesthesia and Co-Existing Disease, ed 5, p 235; Stoelting: Basics of Anesthesia, ed 5, pp 463-469).
question
427. Naltrexone is A. A narcotic with local anesthetic properties B. An opioid agonist-antagonist similar to nalbuphine C. A pure opioid antagonist with a shorter duration of action than naloxone D. An opioid antagonist used for treatment of previously detoxified heroin addicts E. A synthetic opioid derived from oxymorphone
answer
427. (D) Naloxone (Narcan) is a competitive inhibitor at all opioid receptors but has the greatest affinity for µ-receptors. Its duration of action is relatively short (elimination half-life of about 1 hour). For this reason, one must be vigilant for the possibility of renarcotization when reversing long acting narcotics. Naltrexone (ReVia) is the Ncyclopropylmethyl derivative of oxymorphone with a long elimination half-life of 8 to 12 hours. It is currently only available as an oral preparation and is used to block the euphoric effects of injected heroin in addicts who have been previously detoxified. Nalmefene (Revex) is another opioid antagonist that can be administered orally or parenterally and has an extremely long duration of action (elimination terminal half life of 8.5 hours) (Longnecker: Principles and Practice of Anesthe
question
428. Which of the following mechanisms is most frequently responsible for hypoxia in the recovery room? A. Ventilation/perfusion mismatch B. Hypoventilation C. Hypoxic gas mixture D. Intracardiac shunt E. Abnormal gas diffusion
answer
(A) In the recovery room, the most common cause of postoperative hypoxemia is a uneven ventilation/perfusion distribution caused by loss of lung volume resulting from small airway collapse and atelectasis. Risk factors for ventilation/perfusion mismatch in the postoperative period include old age, obstructive lung disease, obesity, increased intraabdominal pressure, and immobility. Supplemental oxygen should be administered to keep the PaO2 in the 80 to 100 mm Hg range, which is associated with a 95% saturation of hemoglobin. Other measures can be taken to restore lung volume, which include recovering obese patients in the sitting position, coughing, and deep breathing (Barash: Clinical Anesthesia, ed 5, pp 1391-1395).
question
429. Hypoparathyroidism secondary to the inadvertent surgical resection of the parathyroid glands during total thyroidectomy typically results in symptoms of hypocalcemia how many hours postoperatively? A. 1 to 2 hours B. 3 to 12 hours C. 12 to 24 hours D. 24 to 72 hours E. Greater than 72 hours
answer
429. (D) Airway obstruction after total thyroidectomy may be caused by a postoperative hematoma, compression of the trachea, tracheomalacia, bilateral recurrent laryngeal nerve damage, or hypocalcemia resulting from inadvertent removal of the parathyroid glands. Although the airway symptoms of hypocalcemia can develop as early as 1 to 3 hours after surgery, they typically do not develop until 24 to 72 hours postoperatively. Because the laryngeal muscles are particularly sensitive to hypocalcemia, early symptoms may include inspiratory stridor, labored breathing, and eventual laryngospasm. Therapy consists of IV administration of calcium gluconate or calcium chloride (Stoelting: Basics of Anesthesia, ed 5, pp 443-444).
question
430. Damage to which nerve may lead to wrist drop? A. Radial B. Axillary C. Median D. Musculocutaneous E. Ulnar
answer
430. (A) Damage to the radial nerve is manifested by weakness in abduction of the thumb, inability to extend the metacarpophalangeal joints, wrist drop, and numbness in the webbed space between the thumb and index fingers. The radial nerve passes around the humerus between the middle and lower portions in the spiral groove posteriorly. As it wraps around the bone, the radial nerve can become compressed between it and the OR table, resulting in nerve injury (Barash: Clinical Anesthesia, ed 5, p 650).
question
431. The most common cause of bronchiectasis is A. Cigarette smoking B. Air pollution C. ?1-Antitrypsin deficiency D. Recurrent bronchial infections E. Squamous cell carcinoma
answer
431. (D) Bronchiectasis is one of several obstructive lung diseases characterized by a diminished FEV1 when pulmonary function is evaluated. It is characterized by permanently dilated bronchi that frequently contain purulent secretions. The affected bronchi are often highly vascularized, giving rise to the possibility of hemoptysis. Collateral circulation through the intercostal and bronchial arteries is also possible in these patients. If these vessels connect with the pulmonary circulation, pulmonary hypertension and eventual cor pulmonale are possible sequelae. Any patient with chronic bronchial infections may develop bronchiectasis (Hines: Stoelting's Anesthesia and Co-Existing Disease, ed 5, p 175).
question
432. A 6-year-old child is transported to the recovery room after a tonsillectomy. The patient was anesthetized with isoflurane, fentanyl, and N2O. Twenty minutes before emergence and tracheal extubation, droperidol was administered. The anesthesiologist is called to the recovery room because the patient is "making strange eye movements." The patient's eyes are rolled back into his head, and his neck is twisted and rigid. The most appropriate drug for treatment of these symptoms is A. Dantrolene B. Thiopental C. Glycopyrrolate D. Chlorpromazine E. Diphenhydramine
answer
432. (E) Drugs that block dopamine receptors may cause acute dystonic reactions in some patients. The incidence with droperidol is about 1%. Treatment is the administration of a drug that crosses the blood brain-barrier with anticholinergic properties such as diphenhydramine or benztropine. Although glycopyrrolate is an anticholinergic drug, it would not be useful in this setting because it does not cross the blood-brain barrier (Hines: Stoelting's Anesthesia and Co-Existing Disease, ed 5, p 644; Stoelting: Pharmacology and Physiology in Anesthetic Practice, ed 4, p 414).
question
433. A 32-year-old military officer is unable to oppose the left thumb and left little finger after an 8-hour exploratory laparotomy under general anesthesia. He had an IV induction through a peripheral IV and had a second IV placed in the antecubital fossa after he was asleep. Damage to which of the following nerves would most likely account for this deficit? A. Radial B. Ulnar C. Median D. Musculocutaneous E. Median antebrachial cutaneous nerve
answer
433. (C) The median nerve is most frequently injured at the antecubital fossa by extravasation of IV drugs (e.g., thiopental) that are toxic to neural tissue, or by direct injury caused by the needle during attempts to cannulate the medial cubital or basilic veins. The median nerve provides sensory innervation to the palmar surface of the lateral three and one-half fingers and adjacent palm, and motor function to the abductor pollicis brevis, flexor pollicis brevis, and opponens pollicis muscles (Stoelting: Basics of Anesthesia, ed 5, p 301).
question
434. Pheochromocytoma would be most likely to coexist with which of the following? A. Insulinoma B. Pituitary adenoma C. Primary hyperaldosteronism (Conn's syndrome) D. Medullary carcinoma of the thyroid E. Carcinoid tumor
answer
434. (D) Pheochromocytoma is an endocrine tumor (with release of catecholamines) in which 90% of patients are hypertensive, 90% of the tumors originate in one adrenal medulla, and 90% of all pheochromocytomas are benign. This disease is rare (<0.1% of hypertension in adults), but when it occurs, it is often seen with a triad of diaphoresis, tachycardia, and headache in patients with hypertension. Other symptoms include palpitations, tremulousness, weight loss, hyperglycemia, hypovolemia, and in some cases dilated cardiomyopathy and congestive heart failure. Death as a result of pheochromocytoma is due to cardiac conditions (e.g., myocardial infarction, congestive heart failure) or an intracranial bleed. In about 5% of cases, pheochromocytomas show an autosomal dominant pattern and may coexist with other endocrine diseases such as medullary carcinoma of the thyroid and hyperparathyroidism. This combination is called multiple endocrine neoplasia or MEN type II or IIA (Sipple's syndrome). MEN type IIB consists of pheochromocytoma, medullary carcinoma of the thyroid, and neuromas of the oral mucosa. The von Hippel-Lindau disease consists of hemangiomas of the nervous system (i.e., retina or cerebellum) and 10% to 25% of these patients also have a pheochromocytoma. The average sized pheochromocytoma contains 100 to 800 mg of norepinephrine (Barash: Clinical Anesthesia, ed 5, p 1142; Hines: Stoelting's Anesthesia and Co-Existing Disease, ed 5, pp 388-393).
question
435. The plasma concentration of which of the following liver enzymes is increased in patients with biliary obstruction? A. Serum glutamic-oxaloacetic transaminase B. Serum glutamic-pyruvic transaminase C. Lactate dehydrogenase D. Alkaline phosphatase E. Alcoholic dehydrogenase
answer
435. (D) Serum glutamic-oxaloacetic transaminase (aspartate aminotransferase), serum glutamic-pyruvic transaminase (alanine aminotransferase), and lactate dehydrogenase all are elevated in patients with liver disease. The serum level of alkaline phosphatase may be a specific indicator of biliary obstruction. Because this enzyme is also produced in the intestines, bone, and placenta, other serum tests must be ordered to differentiate among these potential sources. Concurrent measurement of the serum ?-glutaryl transferase (GGT), leucine aminopeptidase, or 5?-nucleotidase levels can be measured simultaneously with alkaline phosphatase to determine the origin of the latter (Miller: Anesthesia, ed 6, pp 754-755).
question
436. The onset of delirium tremens after abstinence from alcohol usually occurs in A. 8 to 24 hours B. 24 to 48 hours C. 2 to 4 days D. 4 to 7 days E. Greater than 7 days
answer
436. (C) Although early mild symptoms of alcohol withdrawal can be seen within 6 to 8 hours after a substantial drop in the serum alcohol levels, delirium tremens (DTs) which is seen in about 5% of patients, is a life-threatening medical emergency which develops 2 to 4 days after the cessation of alcohol in alcoholics. Symptoms of DT's include hallucinations, combativeness, hyperthermia, tachycardia, hypertension or hypotension, and grand mal seizures. Treatment of severe alcohol withdrawal consists of fluid replacement, electrolyte replacement, and IV vitamin administration with particular attention paid to thiamine. Aggressive administration of benzodiazepines is indicated to prevent seizures (5 to 10 mg of diazepam every 5 minutes until the patient becomes sedated but not unconscious). ?-Blockers are used to suppress overactivity of the sympathetic nervous system, and lidocaine may be effective in the treatment of cardiac dysrhythmias (Hines: Stoelting's Anesthesia and CoExisting Disease, ed 5, p 543).
question
437. A 78-year-old retired coal miner with an intraluminal tracheal tumor is scheduled for tracheal resection. Which of the following is a relative contraindication for tracheal resection? A. Need for postoperative mechanical ventilation for underlying lung disease B. Tumor located at the carina C. Documented liver metastases D. Ischemic heart disease with a history of congestive heart failure E. Tracheal diameter of 0.5 cm at the level of the tumor
answer
437. (A) Operations on the trachea may be indicated in patients who have tracheal tumors or patients who had a previous trauma to the trachea resulting in tracheal stenosis or tracheomalacia. Eighty percent of the operations on the trachea involve segmental resection with primary anastomosis, 10% involve resection with prosthetic reconstruction, and another 10% involve insertion of a T-tube stent. These operations frequently are very complicated and require constant communication between the surgeon and the anesthesiologist. Preoperative pulmonary function tests are indicated in all patients who are to undergo elective tracheal resection. Severe lung disease necessitating postoperative mechanical ventilation is a relative contraindication for tracheal resection because positive airway pressure may cause wound dehiscence (Miller: Anesthesia, ed 6, pp 1912-1913).
question
438. A 78-year-old patient with multiple myeloma is admitted to the ICU for treatment of hypercalcemia. The primary risk associated with anesthetizing patients with hypercalcemia (levels of 14 to 16 mg/dL) is A. Coagulopathy B. Cardiac dysrhythmias C. Hypotension D. Laryngospasm E. Fluid imbalance
answer
438. (B) Hypercalcemia is associated with a number of signs and symptoms, including hypertension, dysrhythmias, shortening of QT interval, kidney stones, seizure, nausea and vomiting, weakness, depression, personality changes, psychosis, and even coma. Generally patients with total serum calcium levels of 12 mg/dL or less do not require any intervention, with the possible exception of rehydration with saline. Higher calcium levels may be associated with clinical symptoms and should be treated before anesthetizing the patient. Caution should be taken with digitalis administration to any patient who is hypercalcemic because some patients may exhibit extreme digitalis sensitivity (Fleisher: Anesthesia and Uncommon Diseases, ed 5, pp 414-415; Longnecker: Principles and Practice of Anesthesiology, ed 2, pp 309-310). NORMAL CALCIUM LEVELS Serum Calcium Serum Ionized Calcium Conventional units (mEq/L) 4.5-5.5 mEq/L 2.1-2.6 mEq/L (mEq/L) Conventional units mg/dL 9.0-11.0 mg/dL 4.25-5.25 mg/dL (mg/dL) SI units (mmol/L) 2.25-2.75 mmol/L 1.05-1.30 mmol/L
question
439. Just before induction of general anesthesia for an 85-year-old demented male with an ischemic bowel, he mentions to you that he forgot to take his green capped eye drops. He states that not taking it daily will result in blindness. The green capped eye drops are A. NaCl drops used to prevent his eye from drying out B. Antibiotic drops C. Steroids D. Used to produce miosis E. Any of a number of eye medication and surgery should be delayed until the eye drops are identified
answer
439. (D) Red-top eye drops cause mydriasis and should be used with caution in patients with closed angle glaucoma. Green top eye drops cause miosis and the pupillary constriction helps keep the drainage route open in patients with glaucoma and helps prevent an acute attack of glaucoma. Clear or white top eye drops do not change pupillary size
question
440. A normal healthy 3-year-old child was involved in a motor vehicle accident. He is coming emergently to the OR. Drug doses need to be calculated, but his weight is not known. What value should be used to estimate the 3-year-old child's weight? A. 8 kg B. 10 kg C. 12 kg D. 14 kg E. 16 kg
answer
440. (D) When reviewing growth curves, the normal 40-week term newborn weighs about 3.5 kg. Children then double their birthweight by 5 months and triple their weight by 1 year. Therefore, the average 1-year-old weighs 10 kg (22 pounds). From the age of 1 to 6 years, children gain about 2 kg per year. Thus, an average 2-year-old weighs 12 kg, 3-year-old weighs 14 kg, 4-year-old weighs 16 kg, 5-year-old weighs 18 kg, and 6-year-old weighs 20 kg. From age 6 to 10 years, children gain about 3 kg per year (Motoyama: Smith's Anesthesia for Infants and Children, ed 7, pp 1203-1205).
question
441. A 62-year-old male undergoes an emergency craniotomy for subdural hematoma. Two years earlier, a VVIpacemaker was placed for third-degree heart block. The patient received vancomycin 1 g IV, before arriving in the OR. General anesthesia is induced with thiopental 300 mg IV and the lungs are hyperventilated to a PaCO2 of 25 mm Hg by mask. Just before tracheal intubation, the patient's heart rate decreases from 70 to 40 beats/min and the pacemaker spikes that were previously present in lead II of the electrocardiogram disappear. The most likely cause of bradycardia in this patient is A. Hypocarbia B. Vancomycin allergy C. Acute increase in intracranial pressure D. A side effect of thiopental E. Pacemaker battery failure
answer
441. (A) Causes for acute pacemaker malfunction in the OR are numerous and include threshold changes, inhibition, generator failure, and lead or electrode dislodgement or breakage. A VVI pacemaker may be inhibited by myopotentials. In this regard, administration of succinylcholine could actually inhibit a VVI pacemaker. Similarly, electrocautery can inhibit a VVI pacemaker through electromagnetic interference. Should this occur, a magnet should be placed over the pacemaker to convert it into a VOO pacemaker, eliminating the possibility of further inhibition. Pacemakers should be evaluated preoperatively to eliminate the possibility of generator failure. Lead breakage or dislodgement is an unlikely cause of pacemaker failure unless the surgeon is working in the vicinity of the electrodes. Acute threshold changes are almost always associated with changes in the serum potassium concentration. In this particular patient, hyperventilation causes a respiratory alkalosis that results in the intracellular shifting of serum potassium. The net result is that the electrical threshold for the pacemaker is raised, preventing ventricular capture (Miller: Anesthesia, ed 6, pp 1426-1427; Thomas: Manual of Cardiac Anesthesia, ed 2, pp 382-383).
question
442. A 28-year-old obese patient has diminished breath sounds bilaterally at the lung bases 18 hours after an emergency appendectomy under general anesthesia. Which of the following maneuvers would be LEAST effective in preventing postoperative pulmonary complications in this patient? A. Coughing B. Voluntary deep breathing C. Performing a forced vital capacity D. Use of incentive spirometry E. Sitting up in bed
answer
442. (C) Therapies aimed at increasing functional residual capacity (FRC) of the lungs are useful in reducing the incidence of post-operative pulmonary complications. Forced expiratory maneuvers may lead to airway closure, which would be of no benefit for this patient (Miller: Anesthesia, ed 6, pp 2713, 2818-2819).
question
443. Below what value of cerebral blood flow (CBF) will signs of cerebral ischemia first begin to appear on the electroencephalogram (EEG)? A. 6 mL/100 g/min B. 15 mL/100 g/min C. 22 mL/100 g/min D. 31 mL/100 g/min E. 40 mL/100 g/min
answer
443. (C) The human brain is able to maintain neuronal function in the face of decreasing CBF below the normal level of 50 mL/100 g/min. Because O2 delivery is directly related to CBF, EEG evidence of cerebral ischemia will appear if CBF is diminished sufficiently. The CBF reserve, however, is substantial and the first signs of cerebral ischemia do not appear on EEG until CBF has fallen to approximately 22 mL/100 g/min. When CBF has fallen to 15/100 g/min, the EEG becomes isoelectric. Irreversible membrane damage and cellular death do not occur, however, until CBF falls to 6 mL/100 g/min. Areas of the brain in which CBF falls in the 6 to 15 mL/100 g/min range are referred to as zones of ischemic penumbra. Several hours may elapse in these areas of the brain before irreversible membrane damage occurs (Miller: Anesthesia, ed 6, pp 833-834).
question
444. A 67-year-old patient is mechanically ventilated in the ICU 2 days after repair of a ruptured abdominal aortic aneurysm. To maintain PaO2 in the 60 to 65 range, 10 cm H2O positive end-expiratory pressure (PEEP) is added to the ventilator cycle. The patient's blood pressure has averaged 110/65 before addition of PEEP. After addition of PEEP, the blood pressure is noted to slowly fall to an average of approximately 95/50. The best explanation for this decrease in blood pressure is A. Tension pneumothorax B. Decreased venous return to the heart C. Increased afterload on the right side of the heart D. Increased afterload on the left side of the heart E. Decreased cardiac output from global myocardial ischemia
answer
444. (B) Positive end-expiratory pressure (PEEP) is the maintenance of positive airway pressure during the entire ventilator cycle. The addition of PEEP to the ventilator cycle is often recommended when PaO2 is not maintained above 60 mm Hg, when breathing an FIO2 of 0.50 or greater. Although not completely understood, PEEP is thought to increase arterial oxygenation, pulmonary compliance, and FRC by expanding previously collapsed but perfused alveoli, thereby decreasing shunt and improving ventilation/perfusion matching. An important adverse effect of PEEP is a decrease in arterial blood pressure caused by a decrease in venous return, left ventricular filling and stroke volume, and cardiac output. These effects are exaggerated in patients with decreased intravascular fluid volume. Other potential adverse effects of PEEP include pneumothorax, pneumomediastinum, and subcutaneous emphysema (Miller Anesthesia, ed 6, pp 2820-2821; Stoelting: Basics of Anesthesia, ed 5, p 596).
question
445. A 64-year-old male undergoes an elective cholecystectomy. Other than essential hypertension, for which he takes propranolol, he is in good health. The patient is anesthetized with isoflurane, N2O, and fentanyl, and paralyzed with d-tubocurarine. At the end of the operation, neuromuscular blockade is antagonized with pyridostigmine and atropine, the trachea is suctioned, and the patient is extubated and taken to the recovery room. Oxymorphone is administered IV for analgesia. One hour after arrival in the recovery room, the patient's heart rate decreases from 70 to 40 beats/min. Which of the following would most likely account for bradycardia in this patient? A. Recurarization B. Oxymorphone C. Pyridostigmine D. Propranolol E. Paradoxical effect of atropine
answer
445. (C) Reversal of neuromuscular blockade with an anticholinesterase drug requires coadministration of an anticholinergic drug to prevent the muscarinic side effects (e.g., bradycardia and salivation) from the neuromuscular reversal agent. The onset of neuromuscular reversal activity is most rapid with edrophonium, followed by neostigmine and pyridostigmine. The durations of action of edrophonium and neostigmine are similar, but pyridostigmine has a longer duration of action. Of the anticholinergic drugs, glycopyrrolate has a longer duration of action than atropine and for this reason should be coadministered with pyridostigmine. In this question, the patient received long-acting pyridostigmine in combination with short-acting atropine. After the effects of atropine wore off, the antimuscarinic effects of pyridostigmine became evident, resulting in bradycardia (Stoelting: Pharmacology and Physiology in Anesthetic Practice, ed 4, pp 258-259).
question
446. Which of the following is most closely associated with minimum alveolar concentration (MAC)? A. Blood/gas partition coefficient B. Oil/gas partition coefficient C. Vapor pressure D. Brain/blood partition coefficient E. Molecular weight
answer
446. (B) As a rough approximation, if one divides 150 by the MAC for any given volatile anesthetic, the quotient will be approximately equal to the oil/gas partition coefficient. For example, if one were to divide the MAC of halothane (0.75) into 150, the quotient would be 200, which is very close to the actual oil/gas partition coefficient for halothane (224). Similarly, if one were to divide the MAC of enflurane (1.68) into 150, the quotient would be 89, which is very similar to the oil/gas partition coefficient for enflurane (98). The fact that anesthetics with a high oil/gas partition coefficient (i.e., lipid-soluble agents) have lower MACs supports the Meyer-Overton theory (critical volume hypothesis) (Stoelting: Pharmacology and Physiology in Anesthetic Practice, ed 4, p 29).
question
447. A 15-year-old, 65-kg patient with Cushing's disease is to undergo a transsphenoidal hypophysectomy to remove a pituitary adenoma. General anesthesia is induced with thiopental IV and tracheal intubation is facilitated with vecuronium 0.25 mg/kg IV. Anesthesia is maintained with isoflurane, N2O and O2. Mannitol 1 g/kg is administered IV to reduce intracranial pressure. At the end of the operation, the patient is extubated and taken to the ICU. Over the next 6 hours the patient has a total urine output of 8.3 L. Serum sodium concentration is 154 mEq/L, serum potassium concentration is 4.8 mEq/L, and serum glucose concentration is 160 mg/dL. Urine specific gravity is 1.002 and urine osmolality is 125 mOsm/L. The most likely cause of the large urine output is A. Osmotic diuresis from mannitol B. Excess mineralocorticoid activity C. Hyperglycemia D. Nephrogenic diabetes insipidus E. Central diabetes insipidus
answer
447. (E) Diabetes insipidus is characterized by hypernatremia, serum hyperosmolality, polyuria, and urine hypoosmolality. Diabetes insipidus may occur after any intracranial procedure, but it is particularly common in surgery involving the pituitary gland. It may develop intraoperatively, but it commonly develops 4 to 12 hours postoperatively. Intravenous half normal saline and dextrose 5% in water are started as replacement fluids. The pharmacologic treatment for diabetes insipidus is synthetic ADH, 1-(3-mercaptoproprionic acid)-D-arginine vasopressin (DDAVP) commonly started when the urine output is greater than 350-400 mL/hr. In a conscious patient it is not essential to administer DDAVP because the patient may increase his oral intake to compensate for polyuria. In the unconscious patient, however, administration of DDAVP is necessary. Vasopressin (DDAVP) may be administered SQ, IV, or intranasally. Fortunately, diabetes insipidus related to surgery and head trauma usually is transient (Hines: Stoelting's Anesthesia and Co-Existing Disease, ed 5, pp 403-404; Miller: Anesthesia, ed 6, p 2159)
question
448. Scopolamine should not be given as a premedication in patients with which of the following neurologic diseases? A. Parkinson's disease B. Alzheimer's disease C. Multiple sclerosis D. Narcolepsy E. Amyotrophic lateral sclerosis
answer
448. (B) The principal feature of Alzheimer's disease is progressive dementia. The onset typically occurs after age 60 years and may affect as many as 20% of patients more than age 80 years. In addition to age, other risk factors include history of serious head trauma (e.g., boxing), Down syndrome, and presence of the disease in a parent or sibling. One biochemical feature of this disease is a decrease in the enzyme choline acetyltransferase in the brain. There is a strong correlation between reduced enzyme activity and decreased cognitive function. Interestingly, administration of the anticholinergic drugs scopolamine or atropine (but not glycopyrrolate which does not cross the blood-brain barrier) causes confusion similar to that seen in the early stages of Alzheimer's disease. Conversely, administration of anticholinesterase drugs capable of penetrating the blood-brain barrier, such as donepezil (Aricept), galantamine, rivastigmine (Exelon), and tacrine are used to treat patients with Alzheimer's disease. Physostigmine may have beneficial effects in some patients as well. Scopolamine is therefore a poor choice for premedication in patients with Alzheimer's disease (Hines: Stoelting's Anesthesia and Co-Existing Disease, ed 5, p 227; Morgan: Clinical Anesthesiology, ed 4, p 652).
question
449. A 63-year-old male patient is scheduled to undergo a right hemicolectomy under general anesthesia. Anesthesia is induced with thiopental 4 mg/kg IV and fentanyl 100 µg IV. Succinylcholine 1.5 mg/kg IV is administered to facilitate tracheal intubation. Anesthesia is maintained with isoflurane and N2O. After all four twitches of the train-of-four stimulus return to baseline values, pancuronium 5 mg IV is administered. Gentamicin 80 mg and cefazolin 1 g are administered IV as a prophylactic treatment. At the end of surgery, two of four thumb twitches can be elicited to train-of-four stimulation of the ulnar nerve and neuromuscular blockade is antagonized with neostigmine 0.05 mg/kg IV and atropine 0.015 mg/kg IV. The patient, however, begins to move before the incision is completely closed, and succinylcholine 40 mg IV is given. Fifteen minutes later, all anesthetics are discontinued and the patient is ventilated with 100% O2, but the patient remains apneic. The most likely cause of apnea is A. Fentanyl B. Recurarization C. Succinylcholine D. Thiopental E. Gentamicin
answer
449. (C) At the end of any general anesthetic, spontaneous ventilation must be restored before the patient can be extubated. The differential diagnosis for persistent apnea includes muscle relaxants (inadequate reversal or pseudocholinesterase deficiency), volatile anesthetics, narcotics, hypocarbia, damage to the phrenic nerves bilaterally, and the possibility of a central nervous system (CNS) event. Succinylcholine is hydrolyzed by pseudocholinesterase to succinylmonocholine and choline. This is further hydrolyzed by plasma cholinesterase to succinic acid and choline. All of the anticholinesterase agents used to reverse nondepolarizing neuromuscular blockade also inhibit pseudocholinesterase. Administration of succinylcholine to any patient who has already received an anticholinesterase will result in a prolonged block from the succinylcholine because it can no longer be easily hydrolyzed. In this patient, therefore, succinylcholine would be by far the most likely cause of apnea at the end of the operation (Stoelting: Pharmacology and Physiology in Anesthetic Practice, ed 4, p 218).
question
450. A 53-year-old female with endometrial cancer is undergoing an abdominal hysterectomy under general anesthesia with enflurane. During the first hour of anesthesia, urine output is 100 mL. Blood loss is minimal. When the patient is placed in the Trendelenburg position, the urine output declines to virtually zero. The most likely explanation for this sudden decrease in urine output in this patient is A. Pooling of urine in the dome of the bladder B. Kinking of the urinary catheter C. Fluoride toxicity from enflurane D. Increased antidiuretic hormone (ADH) production from surgical stimulation E. Hypovolemia
answer
450. (A) Pooling of urine in the dome of the bladder should be considered as a possible cause of oliguria in a patient in the Trendelenburg position. Acute hypovolemia is an unlikely cause of oliguria in this patient in the absence of bleeding. Fluoride toxicity from the metabolism of enflurane is extremely rare and is associated with nonoliguric renal failure. See also the answer to question 424 (Morgan: Clinical Anesthesiology, ed 4, pp 739- 741).
question
451. Which of the following diseases is not associated with a decrease in DLCO? A. Emphysema B. Lung resection C. Pulmonary emboli D. Anemia E. Obesity
answer
451. (E) DL is defined as the diffusing capacity of the lung. When a nontoxic low concentration of carbon monoxide is used for the measurement it is called DLCO. The normal value of DLCO is 20 to 30 mL/min/mm Hg and is influenced by the volume of blood (hemoglobin) within the pulmonary circulation. Thus, diseases associated with a decrease in pulmonary blood volume (i.e., anemia, emphysema, hypovolemia, pulmonary hypertension) will be reflected by a decrease in the DLCO. DLCO is also decreased with oxygen toxicity as well as pulmonary edema. Conditions associated with an increased DLCO include the supine position, exercise, obesity and leftto-right cardiac shunts (Barash: Clinical Anesthesia, ed 5, pp 806-807; Miller: Anesthesia, ed 6, p 1011).
question
452. Each of the following postoperative complications of thyroid surgery can result in upper airway obstruction EXCEPT A. Tracheomalacia B. Tetany C. Cervical hematoma D. Bilateral recurrent laryngeal nerve injury E. Bilateral superior laryngeal nerve injury
answer
452. (E) Patients undergoing thyroid surgery are at risk for airway obstruction from a number of causes. Postoperative hemorrhage sufficient to cause a large hematoma could compress the trachea and cause airway obstruction because of the close proximity of the thyroid gland to the trachea. Permanent hypoparathyroidism is a rare complication that may cause hypocalcemia leading to progressive stridor followed by laryngospasm. The most common nerve injury after thyroid surgery is damage to the abductor fibers of the recurrent laryngeal nerve. Unilaterally, this is manifested as hoarseness. Bilateral recurrent laryngeal nerve damage, however, may lead to airway obstruction during inspiration. Selective injury of the adductor fibers of the recurrent laryngeal nerve is a possible complication of thyroid surgery. This injury would leave the vocal cords open because the abductor fibers would be unopposed, placing the patient at great risk for aspiration. The superior laryngeal nerve has an extrinsic branch that innervates the cricothyroid muscle (which tenses the vocal cords), and an internal branch that provides sensory innervation to the pharynx above the vocal cords. Bilateral damage to this nerve would result in hoarseness and would predispose the patient to aspiration but would not lead to airway obstruction per se (Stoelting: Basics of Anesthesia, ed 5, pp 443-444).
question
453. The most sensitive early sign of MH during general anesthesia is A. Tachycardia B. Hypertension C. Fever D. Hypoxia E. Increased end-expiratory CO2 tension (PECO2)
answer
453. (E) Malignant hyperthermia is a clinical syndrome that may develop rapidly or take hours to manifest, sometimes not occurring until the patient is in the recovery room. Clinical signs include hypertension, tachycardia, respiratory acidosis, metabolic acidosis, muscle rigidity, myoglobinuria, and fever. The diagnosis of MH is unlikely, however, if only one of these signs is manifested. Because MH is a metabolic disorder, one of the first sensitive signs is an increase in the production of CO2 and concomitant respiratory acidosis. This is the most reliable early sign of the syndrome (Miller: Anesthesia, ed 6, p 1178).
question
454. A 78-year-old female is anesthetized for a right hemicolectomy for 3 hours. At the end of the operation the patient's blood pressure is 130/85 mm Hg, heart rate is 84 beats/min, core body temperature is 35.4° C, and PECO2 on mass spectrometer is 38 mm Hg. Which of the following would be the LEAST plausible reason for prolonged apnea in this patient? A. Residual neuromuscular blockade B. Narcotic overdose C. Cerebral hemorrhage D. Unrecognized obstructive pulmonary disease and high baseline PaCO2 E. Persistent intraoperative hyperventilation
answer
454. (E) Hyperventilation to PaCO2 of 20 mm Hg or higher for more than 2 hours will result in active transport of HCO3 out of the CNS. This results in spontaneous breathing at a lower (not higher) PaCO2. The other choices should be included in the differential diagnosis of apnea (Hines: Stoelting's Anesthesia and Co-Existing Disease, ed 5, pp 359-361; Miller: Anesthesia, ed 6, p 718).
question
455. A 68-year-old woman with severe rheumatoid arthritis undergoes pulmonary function evaluation before an elective abdominal surgery. Forced expiratory volume in one second (FEV1) and forced vital capacity (FVC) are within normal limits; however, the maximum voluntary ventilation (MVV) is only 40% of predicted. The next step in the pulmonary function evaluation of this patient should be to A. Obtain ABGs on room air B. Obtain a flow-volume loop C. Obtain a measurement of peak flow D. Obtain a ventilation/perfusion scan E. Assume, in the face of normal FEV1, poor effort on the part of the patient and proceed
answer
455. (B) Maximum voluntary ventilation (MVV) is a nonspecific pulmonary function test that measures the endurance of the ventilatory muscles and indirectly reflects the compliance of the lung and thorax as well as airway resistance. A decreased MVV may be caused by impairment to inspiration or expiration. In this patient, FEV1 is normal, which strongly suggests that the ventilatory impairment is during inspiration. A flow-volume loop would be a very useful confirmatory test (Barash: Clinical Anesthesia, ed 5, pp 805-808, 815-817).
question
456. Which of the following is NOT a component of the post-anesthetic discharge scoring system (PADSS) used to evaluate the suitability of a patient to be discharged from an ambulatory surgical facility? A. Drinking B. Ambulation C. Nausea and vomiting D. Pain E. Surgical bleeding
answer
456. (A) Guidelines for safe discharge of patients from ambulatory surgical centers include stable vital signs, ability to walk without dizziness, controlled pain, absence of nausea and vomiting, and minimal surgical bleeding. The PADSS is a tool for objectively assessing a patient's readiness for discharge from the surgical center and includes these five criteria. Requirements to drink fluids and to void before home discharge are controversial and are not parameters included in the PADSS (Barash: Clinical Anesthesia, ed 5, pp 1242-1243; Miller: Anesthesia, ed 6, pp 2708-2709).
question
457. During emergency repair of a mandibular jaw fracture in an otherwise healthy 19-year-old male, the patient's temperature is noted to rise from 37° C on induction to 38° C after 2 hours of surgery. Which of the following informational items would be LEAST useful in ruling out MH in this patient? A. Normal heart rate and blood pressure B. History of negative caffeine-halothane contracture test carried out 6 months earlier C. History of a uncomplicated general anesthetic at age 16 years with halothane and succinylcholine D. Normal ABGs drawn when the patient's temperature reached 38° C E. No increase in respiration rate with spontaneous breathing
answer
457. (C) Malignant hyperthermia is a difficult diagnosis to make on clinical grounds alone. Signs of MH may be fulminant or very subtle. They may occur immediately after induction or may not be manifested until the patient has reached the recovery room or even later. Malignant hyperthermia is a disorder of metabolism and is associated with hypertension, tachycardia, dysrhythmias, respiratory acidosis, metabolic acidosis, muscular rigidity, rhabdomyolysis, and fever. Contrary to what one might believe based on the name of this disease, fever is typically a late finding. Other diseases that may mimic MH include alcohol withdrawal, acute cocaine toxicity, bacteremia, pheochromocytoma, hyperthyroidism, and neuroleptic malignant syndrome. An elevation in temperature alone with normal blood gases, heart rate, and blood pressure, and no evidence of muscle breakdown would very likely not be due to MH. If a patient had been previously subjected to muscle biopsy and caffeine-halothane contracture testing with negative results, MH would be exceedingly rare, although a false-negative result is possible. A history of a previous anesthetic without MH triggering would be of little reassurance in a patient in whom a MH episode is suspected. It is not uncommon for MH-susceptible individuals to not trigger when a triggering anesthetic is administered initially but develop fulminant MH with a subsequent anesthetic (Miller: Anesthesia, ed 6, pp 1180-1186).
question
458. Which of the following drugs is useful in the treatment of asthma by specifically interfering with the leukotriene pathway? A. Fluticasone (Flovent) B. Ipratropium bromide (Atrovent) C. Triamcinolone (Azmacort) D. Montelukast (Singulair) E. Salmeterol (Serevent)
answer
458. (D) Asthma is an inflammatory illness that has bronchial hyperreactivity and bronchospasm as a result. Treatment is first directed at the inflammatory component as the underlying problem, reserving bronchodilators for symptomatic use. Because leukotrienes may function as inflammatory mediators, the leukotriene pathway inhibitors such as zileuton and the leukotriene receptor antagonist montelukast (Singulair) are being used for treatment of asthma. Zileuton and montelukast are only available as oral preparations, whereas the other drugs listed are given by inhalation. Fluticasone and triamcinolone are anti-inflammatory corticosteroids. Ipratropium is a quaternary ammonium compound formed by the introduction of an isopropyl group to the N atom of atropine and produces effects similar to those of atropine. One unexpected finding is a relative lack of effect on mucociliary clearance, which makes it useful in patients with airway disease, especially if parasympathetic tone of the airways is increased. Salmeterol is a ?2-selective adrenergic drug (Hardman: Goodman ; Gilman's The Pharmacological Basis of Therapeutics, ed 11, pp 721-725, 730-731).
question
459. A 68-year-old, 100-kg patient is undergoing a transurethral resection of the prostate gland under general anesthesia. Upon arrival in the recovery room, the patient appears restless and confused. Serum sodium is checked and found to be 110 mEq/L. How many mEq of sodium are needed to raise the serum [Na+] to 120 mEq/L? A. 300 mEq B. 400 mEq C. 500 mEq D. 600 mEq E. 700 mEq
answer
459. (D) Acute decreases in serum sodium, due to absorption of bladder irrigating fluids, rarely cause symptoms unless the sodium level drops below 120 mEq/L. At this level, tissue edema may develop and clinical neurologic signs (e.g., restlessness, nausea, confusion, seizures, coma) or ECG changes (e.g., widening of the QRS complex, elevation of the ST segment, ventricular tachycardia, or ventricular fibrillation) may be manifested. Treatment of mild decreases in serum sodium (i.e., 120 to 135 mEq/L with no neurologic or ECG changes) is by fluid restriction and/or administration of a diuretic such as furosemide. When the sodium levels drops below 120 mEq/L and neurologic symptoms or changes in the ECG develop, sodium chloride administration is needed. To calculate the amount needed, one multiplies the patient's total body water (i.e., 0.6 × body weight = TBW) by the change in sodium desired. In this case, the TBW is 60 L (0.6 × 100 kg) and the change of sodium is 10 mEq (120 mEq/L ? 110 mEq/L), thus 60 L × 10 mEq/L = 600 mEq. Caution is advised in administering sodium because too rapid administration may lead to demyelinating CNS lesions. The recommended rate of 3% sodium chloride (513 mEq/L) is 1 to 2 mL/kg/hour. Serum sodium levels should be checked at least every hour until the sodium level increases above 120 mEq/L (Barash: Clinical Anesthesia, ed 5, pp 188-192, 1026-1028).
question
460. Trismus after administration of succinylcholine IV signals the onset of MH in what percentage of patients? A. Less than 50% B. 50% C. 75% D. 80% E. Greater than 80%
answer
460. (A) Trismus (masseter spasm) is characterized by rigidity of the jaw muscles while the limb muscles remain flaccid after administration of succinylcholine. Trismus may herald the onset of MH in some patients, but may be due to a number of other causes and may occur in normal patients. It previously had been believed that 50% of patients who experience trismus after administration of succinylcholine would go on to develop MH. Recent evidence suggests, however, that the incidence is less. If masseter spasm occurs in a patient after administration of succinylcholine, the most conservative course would be to cancel the operation. If cancellation of the operation is not feasible, then a nontriggering anesthetic should be used and the anesthesiologist should pay close attention for any signs of MH (Miller: Anesthesia, ed 6, pp 1180-1181).
question
461. A 45-year-old male is brought to the OR emergently for repair of a ruptured abdominal aortic aneurysm. The patient is pretreated with d-tubocurarine 3 mg, anesthesia is induced with ketamine 2 mg/kg IV, and tracheal intubation is facilitated with succinylcholine 1.5 mg/kg IV. Immediately after tracheal intubation, the patient's blood pressure falls from 110/80 to 50/20 mm Hg. What is the most likely cause of the sudden severe hypotension in this patient? A. Hypovolemia B. Direct myocardial depression from ketamine C. Vasovagal response to direct laryngoscopy D. Arteriolar vasodilation from succinylcholine-mediated histamine release E. Ganglionic blockade from d-tubocurarine
answer
461. (B) Ketamine is unique among the IV induction agents in that it usually produces cardiac stimulation manifested by increased heart rate, mean arterial pressure, and cardiac output. Ketamine is believed to have a centrally mediated sympathetic nervous system-stimulating effect. This effect is, however, not related to dose. In isolated rabbit and canine hearts and in intact dogs, ketamine has been demonstrated to produce myocardial depression. Clinically, however, the myocardial depressant properties of ketamine are overridden by its sympathetic nervous system stimulating properties. When systemic catecholamines have been depleted or when the patient is under deep anesthesia, the myocardial depressant properties of ketamine may predominate (Stoelting: Pharmacology and Physiology in Anesthetic Practice, ed 4, p 172).
question
462. Malignant hyperthermia is believed to involve a generalized disorder of membrane permeability to A. Sodium B. Potassium C. Calcium D. Magnesium E. Phosphate
answer
462. (C) In the normal muscle cell, depolarization results in release of calcium from the sarcoplasmic reticulum. The increased intracellular calcium concentration results in muscle contraction. The calcium then is rapidly taken up via calcium pumps back into the sarcoplasmic reticulum, resulting in relaxation. Both the release and reuptake of calcium are energy-requiring processes, i.e., result in the hydrolysis of adenosine triphosphate (ATP). Dantrolene, the pharmacologic treatment for MH, blocks release of calcium from the sarcoplasmic reticulum without affecting the reuptake process. The defect in MH is thought to be decreased control of intracellular calcium stores preventing muscle relaxation (Miller: Anesthesia, ed 6, pp 1170-1179).
question
463. A 25-year-old male with a history of testicular cancer is scheduled to undergo an exploratory laparotomy under general anesthesia. He has received bleomycin for metastatic disease. Which of the following is an important consideration concerning the pulmonary toxicity of bleomycin? A. N2O should not be used B. Preoperative pulmonary function tests should be obtained C. The patient should be ventilated at a slow rate and inspiratory-to-expiratory (I:E) ratio of 1:3 D. Aminophylline should be started preoperatively E. FIO2 should be less than 0.3
answer
463. (E) Approximately 4% of patients treated with bleomycin develop pulmonary toxicity, which manifests as severe pulmonary fibrosis and hypoxemia. Death from severe pulmonary toxicity occurs in approximately 1% to 2% of patients treated with bleomycin. Patients who are at greater risk for bleomycin-induced pulmonary toxicity include elderly patients, those receiving more than 200 to 400 mg, those with coexisting lung disease, and those recently exposed to bleomycin. In addition, there is evidence that prior radiotherapy and possibly receipt of enriched concentrations of O2 (i.e., inspired oxygen > 30%) during surgery increase risk of pulmonary toxicity. Clinically, patients gradually develop dyspnea, a nonproductive cough, and hypoxemia, and pulmonary function tests typically demonstrate changes in gas flow and lung volumes consistent with restrictive pulmonary disease. If radiographic evidence such as bilateral diffuse interstitial infiltrates appears, pulmonary fibrosis usually is irreversible (Stoelting: Pharmacology and Physiology in Anesthetic Practice, ed 4, pp 564- 565).
question
464. A 39-year-old obese female undergoes an abdominal hysterectomy under general anesthesia. Induction of anesthesia is uneventful. SaO2 is 98% during the first 15 minutes of the operation. However, when her head is flexed and she is placed in the Trendelenburg position to improve surgical exposure, SaO2 falls to 90%. The most likely explanation for this desaturation is A. Diffusion hypoxia B. Decreased functional residual capacity (FRC) C. Mainstem intubation D. Decreased cardiac output E. Venous air embolism
answer
464. (C) Head flexion can advance the tube up to 1.9 cm toward the carina and in some cases convert an endotracheal intubation into an endobronchial intubation. Extension of the head has the opposite effect and can withdraw the tube up to 1.9 cm, resulting in extubation of some patients. Turning the head laterally can move the distal tip of the endotracheal tube about 0.7 cm away from the carina. The Trendelenburg position causes a cephalad shift of the mediastinum and can cause the endotracheal tube to migrate distally as well (Lobato: Complications in Anesthesiology, p 834; Stoelting: Basics of Anesthesia, ed 5, p 232).
question
465. How long after intravitreal injection of sulfur hexafluoride and air can N2O be used without risk of increasing intraocular pressure? A. 1 hour B. 24 hours C. 10 days D. 1 month E. Never
answer
465. (C) Sulfur hexafluoride is sometimes injected in the vitreous in patients with a detached retina to mechanically facilitate reattachment. To prevent changes in the size of the gas bubble, the patients should be given 100% O2 15 minutes before injection of sulfur hexafluoride. If these patients are anesthetized with general anesthesia within 10 days, N2O should not be given because N2O can diffuse into the gas bubble, increasing intraocular pressure, and may result in blindness (Barash: Clinical Anesthesia, ed 5, p 982).
question
466. A 54-year-old female is undergoing a total thyroidectomy under general anesthesia. The patient is awakened in the OR, the mouth and pharynx are suctioned, and after intact laryngeal reflexes are demonstrated, the endotracheal tube is removed. Two days later, the anesthesiologist is consulted because the patient has severe stridor and upper airway obstruction. The most likely cause of airway obstruction in this patient is A. Damage to the recurrent laryngeal nerve B. Damage to the superior laryngeal nerve C. Tracheomalacia D. Hypocalcemia E. Hematoma
answer
466. (D) The symptoms of hypocalcemia, which may be manifested as laryngospasm or laryngeal stridor, usually develop within the first 24 to 96 hours after total thyroidectomy. After the airway is established and secured, the patient should be treated with IV calcium in the form of either calcium gluconate or calcium chloride (Barash: Clinical Anesthesia, ed 5, pp 199, 1133).
question
467. A 27-year-old obese woman is scheduled to undergo foot surgery under general anesthesia. She underwent a subtotal thyroidectomy 3 years ago and takes levothyroxine (Synthroid). Which of the following laboratory tests would be the most useful in evaluating whether this patient is euthyroid? A. Total plasma thyroxine (T4) B. Total plasma triiodothyronine (T3) C. Thyroid-stimulating hormone (TSH) D. Resin triiodothyronine uptake E. Radioactive iodine uptake
answer
467. (C) Because the circulating levels of T3 and T4 regulate TSH release from the anterior pituitary gland by a negative feedback mechanism, a normal plasma concentration of TSH confirms an euthyroid state. The pharmacologic treatment of choice for patients with hypothyroidism is sodium levothyroxine (T4). Sodium levothyronine (triiodothyronine, T3) and desiccated thyroid are alternate therapeutic agents (Barash: Clinical Anesthesia, ed 5, pp 1130-1134; Hines: Stoelting's Anesthesia and Co-Existing Disease, ed 5, pp 378-381).
question
468. An 85-year-old male with no previous medical history except for cataracts is undergoing a transurethral resection of the prostate gland under spinal anesthesia. Twenty minutes into the procedure the patient becomes restless. Over the next 20 minutes his blood pressure increases from 110/70 to 140/90 mm Hg and his heart rate slows from 90 to 50 beats/min. The patient is noted to have some difficulty breathing. The most likely cause of these symptoms in this patient is A. Volume overload B. Hyponatremia C. High spinal D. Bladder perforation E. Autonomic hyperreflexia
answer
468. (A) Large quantities of irrigating fluid can be absorbed during transurethral resection of the prostate gland because the open venous sinuses in the prostate allow the irrigation fluid to be absorbed. From 10 to 30 mL of fluid per minute are absorbed on the average. During long cases, this can amount to several liters, causing hypertension, reflex bradycardia, and pulmonary congestion. Treatment consists of fluid restriction and a loop diuretic (e.g., furosemide) when the [Na+] level is greater than 120 mEq/L. Rarely does the amount of fluid absorbed cause significant hyponatremia (i.e., ;120 mEq/L). In these cases of significant hyponatremia, 3% sodium chloride may be infused slowly intravenously (in addition to the loop diuretic and fluid restriction) until the sodium level reaches 120 mEq/L (Barash: Clinical Anesthesia, ed 5, pp 1027-1028).
question
469. A 17-year-old patient with third-degree burns over 30% of his body is scheduled for débridement and skin grafting 12 days after sustaining a thermal injury. Select the true statement regarding the use of depolarizing and nondepolarizing muscle relaxants in this patient, compared with normal patients. A. Sensitivity to both depolarizing and nondepolarizing muscle relaxants is increased B. Sensitivity to both depolarizing and nondepolarizing muscle relaxants is decreased C. Sensitivity to depolarizing muscle relaxants is increased while sensitivity to nondepolarizing muscle relaxants is decreased D. Sensitivity to depolarizing muscle relaxants is decreased while sensitivity to nondepolarizing muscle relaxants is increased E. Sensitivity to nondepolarizing is unchanged while sensitivity to depolarizing muscle relaxants is increased
answer
469. (C) Patients who have sustained thermal injuries are at risk for massive potassium release and potential cardiac arrest if succinylcholine is administered 24 hours or more after they sustain the burn, and they remain at risk until the burn has healed. This increased sensitivity to succinylcholine is thought to be related to proliferation of extrajunctional receptors. These same receptors are thought to be related to the increased requirement for nondepolarizing neuromuscular blocking agents in these patients (Barash: Clinical Anesthesia, ed 5, p 1288).
question
470. A nervous 57-year-old woman with morbid fear of intramuscular injections is scheduled for breast biopsy and is given a premedication in the waiting area before surgery. The anesthesiologist is summoned 45 minutes later because the patient is complaining of dry mouth and has a heart rate of 45 beats/min. Which of the following premedications is most likely responsible for these side effects? A. Scopolamine B. Meperidine C. Midazolam D. Clonidine E. Droperidol
answer
470. (D) Scopolamine, an anticholinergic, has stronger antisialagogue effects than glycopyrrolate or atropine. Scopolamine has better sedative and amnesic effects than atropine. Glycopyrrolate, which does not cross the blood-brain barrier, has no CNS or bradycardiac effects. Although heart rate may decrease with atropine and scopolamine, these effects are minimal (i.e., about 4 to 8 beats/min). These anticholinergics usually are administered parenterally; however, a scopolamine patch could be applied topically. Meperidine may produce a dry mouth in some patients, but it is much more likely to increase the heart rate rather than lower it because of its modest atropine-like effects. Meperidine is well absorbed from the gastrointestinal tract but is usually administered parenterally. Midazolam, which is usually administered parenterally, may produce bradycardia or tachycardia in selected patients but does not cause a dry mouth and may produce excessive salivation. Droperidol can produce tachycardia is some patients and is usually given parenterally. Clonidine is the most likely agent to cause bradycardia, hypotension and dry mouth. The oral dose is 5 ?g/kg (Stoelting: Basics of Anesthesia, ed 5, pp 170-172; Stoelting: Pharmacology and Physiology in Anesthetic Practice, ed 4, pp 340-344).
question
471. A 65-year-old patient with a history of chronic obstructive pulmonary disease and coronary artery disease (CAD) undergoes an appendectomy uneventfully under general anesthesia. In the recovery room, ABGs are as follows: PaO2 60 mm Hg, PaCO2 50 mm Hg, pH 7.35, and hemoglobin 8.1 g/dL. Which of the following steps would produce the greatest increase in O2 delivery to the myocardium? A. Administration of 100% O2 with a close-fitting mask B. Administration of 35% O2 with a Venturi mask C. Withhold narcotics D. Transfuse with 2 units of packed red blood cells (RBCs) E. Administer 1 ampule of HCO3
answer
471. (D) One gram of hemoglobin can combine with 1.34 mL of O2. None of the other choices in this question will do as much to increase the O2-carrying capacity of this patient's blood as a transfusion (Stoelting: Pharmacology and Physiology in Anesthetic Practice, ed 4, p 849).
question
472. Allergic reactions occurring during the immediate perioperative period are most commonly attributable to administration of A. Muscle relaxants B. Local anesthetics C. Antibiotics D. Opioids E. ?-Blockers
answer
472. (A) Many of the drugs commonly administered during surgery and anesthesia have the potential to evoke allergic reactions (e.g., pentothal, propofol, local anesthetics, antibiotics, and protamine, as well as other materials used during surgery, such as vascular graft material, chymopapain, and latex). Virtually all drugs administered IV have been reported to cause allergic reactions. Possible exceptions include benzodiazepines and ketamine. An allergic reaction should be considered when there is an abrupt fall in blood pressure accompanied by increases in heart rate that exceed 30% of the control values. Greater than 60% of all drug-induced allergic reactions observed during the perioperative period are attributable to muscle relaxants. Latex allergy is thought to be responsible for 15% of allergic reactions under anesthesia, sometimes including reactions originally attributed to other substances. Patients at risk for latex allergy include health care workers and patients with spina bifida. Although most drug-induced allergic reactions develop within 5 to 10 minutes of exposure, latex signs typically take more than 30 minutes to develop (Hines: Stoelting's Anesthesia and Co- Existing Disease, ed 5, pp 527-530; Miller: Anesthesia, ed 6, p 1092).
question
473. Caution is advised when using succinylcholine in patients with Huntington's chorea because A. They are at increased risk for MH B. Potassium release may be excessive C. They may have a decreased concentration of pseudocholinesterase D. There may be adverse interactions between succinylcholine and phenothiazine E. Succinylcholine increases intracranial pressure
answer
473. (C) Decreased levels of pseudocholinesterase have been reported in patients with Huntington's chorea. For this reason, the effects of succinylcholine may be prolonged in some of these patients. It has been suggested that the sensitivity to nondepolarizing muscle relaxants is also increased (Hines: Stoelting's Anesthesia and Co- Existing Disease, ed 5, p 229).
question
474. Which of the following would not result in an increase in intraocular pressure? A. Increase in PaCO2 from 35 to 40 mm Hg B. Arterial hypoxemia C. 100 mg IM succinylcholine D. Acute rise in venous pressure from coughing E. 100 mg IV succinylcholine in patient in whom eye muscles have been detached from the globe
answer
474. (A) Normal intraocular pressure is 10 to 22 mm Hg. In general, IV anesthetics, with the possible exception of ketamine, decrease intraocular pressure. In addition, nondepolarizing neuromuscular blockers, inhaled anesthetics, narcotics, carbonic anhydrase inhibitors, osmotic diuretics, and hypothermia decrease intraocular pressure. However, elevation of PaCO2 out of the physiologic range, as seen with hypoventilation as well as arterial hypoxemia, will increase intraocular pressure. Depolarizing neuromuscular blockers, such as succinylcholine, also increase intraocular pressure. This increase in intraocular pressure occurs when succinylcholine is administered IM or IV. Pretreatment with a nondepolarizing muscle relaxant before administering succinylcholine may attenuate the rise in intraocular pressure. The mechanism for the increase in intraocular pressure after succinylcholine use is related to drug-induced cycloplegia rather than contraction of extraocular muscles, as this increase in intraocular pressure will occur even if the intraocular muscles are cut. The greatest increase in intraocular pressure occurs with coughing or vomiting, where the intraocular pressure may increase as much as 35 to 50 mm Hg. The proposed mechanism for the acute increase in intraocular pressure is an increase in venous pressure. There does not appear to be a change in intraocular pressure with changes within normal physiologic ranges in arterial blood pressure or PaCO2 (Barash: Clinical Anesthesia, ed 5, pp 978-980; Stoelting: Basics of Anesthesia, ed 5, pp 464-465).
question
475. An apnea hypopnea index of 30 means? A. Episodes of hypopnea are 30 times more common than apnea B. Episodes of apnea are 30 times more common than hypopnea C. Episodes of apnea and hypopnea occur at a rate of 30 per hour D. Apnea/hypopnea episodes last 30 seconds E. Apnea/hypopnea episodes occur at rate of 30 per sleep cycle
answer
475. (C) The apnea-hypopnea index (AHI) is used to quantify the number of apnea or hypopnea episodes that occur per hour. Apnea is defined as no ventilation for periods of 10 seconds or more. Hypopnea is defined as a 50% decrease in airflow or a decrease sufficient to cause a decrease in oxygen saturation of 4%. An apnea- hypopnea index of greater than 30 signifies severe OSA (Barash: Clinical Anesthesia, ed 5, p 1042; Lobato: Complications in Anesthesiology, p 625).
question
476. Which of the following preoperative pulmonary function tests is NOT associated with an increased operative risk for pneumonectomy? A. FEV1 less than 50% of the FVC B. FEV1 less than 2 L C. Maximum breathing capacity less than 50% of predicted D. Residual volume/total lung capacity less than 50% E. Hypercarbia on room air ABGs
answer
476. (D) Any patient who is scheduled for a pneumonectomy should undergo a series of preoperative pulmonary function tests. These tests are generally conducted in three phases. The tests listed in this question pertain to the first battery of pulmonary function tests, which are whole-lung tests. Residual volume to total lung capacity greater than 50% (not <50%) is associated with an increased operative risk. If the results of any of the initial whole-lung tests are below the acceptable limits, a second phase of testing should be carried out in which the function of each lung is evaluated separately. The predicted postoperative FEV1 after the second phase of pulmonary function testing is carried out should be greater than 0.85 L. If the criteria for the second level of pulmonary function testing cannot be met and pneumonectomy is still desired, then a third level of testing should be carried out. During the third phase of testing, postoperative conditions mimicking pneumonectomy are produced by occluding the pulmonary artery with a balloon on the side that is to be resected. Results of this test that are consistent with poor outcome after pneumonectomy include mean pulmonary artery pressure greater than 40 mm Hg, PaCO2 greater than 60 mm Hg, or PaO2 less than 45 mm Hg (Miller: Anesthesia, ed 6, pp 1852-1854).
question
477. A 26-year-old male patient is undergoing an emergency exploratory laparotomy under general anesthesia with isoflurane. SaO2 is 89% on the pulse oximeter. PaO2 on ABGs is 77 mm Hg. The patient's core body temperature is 35° C. What is the corrected PaO2? A. 68 mm Hg B. 72 mm Hg C. 77 mm Hg D. 86 mm Hg E. 92 mm Hg
answer
477. (A) Measured PaO2 should be decreased about 6% for each degree Celsius cooler the patient's temperature is than the electrode (37° C). Because the patient is 2° C cooler than the electrode, a 12% decrease (9 mm Hg) would be expected in this patient (77 mm Hg ? 9 mm Hg = 68 mm Hg) (Stoelting: Basics of Anesthesia, ed 5, p 325).
question
478. A 27-year-old patient with a 10-year history of Crohn's disease is scheduled to undergo drainage of a rectal abscess under general anesthesia. His preoperative medications include prednisone, sulfasalazine, and cyanocobalamin. He has no known allergies and is otherwise healthy. Before induction of anesthesia the patient is noted to have central cyanosis and the pulse oximeter shows a SaO2 of 89%, which does not increase after the administration of 100% O2 for 2 minutes. ABGs are as follows: PaO2 490 mm Hg, PaCO2 32 Hg, pH 7.43, SaO2 89%. The most likely cause of these findings is A. Presence of sulfhemoglobin B. Presence of methemoglobin C. Presence of cyanhemoglobin D. Presence of carboxyhemoglobin E. Blood gas error
answer
478. (A) The two main causes of central cyanosis are decreased arterial oxygen saturation and hemoglobin abnormalities (e.g., methemoglobinemia and sulfhemoglobinemia). Sulfasalazine (Azulfidine) can cause the formation of sulfhemoglobin. Sulfhemoglobin, like methemoglobin, may cause low O2 saturation in the face of high PaO2. There is no treatment for sulfhemoglobinemia except to wait for the destruction of the erythrocytes (Hines: Stoelting's Anesthesia and Co-Existing Disease, ed 5, pp 287-288; Kasper: Harrison's Principles of Internal Medicine, ed 16, pp 210-211).
question
479. The muscle relaxant of choice (i.e., minimal cardiovascular changes) during resection of a pheochromocytoma is A. Mivacurium B. Pancuronium C. Curare D. Atracurium E. Vecuronium
answer
479. (E) Muscle relaxants that stimulate histamine release or cause increased sympathetic outflow should not be given to patients with pheochromocytoma. Vecuronium has no histamine-releasing properties and does not stimulate the sympathetic nervous system. Mivacurium, curare, and metocurine and atracurium all release histamine to some degree. Pancuronium is associated with tachycardia and should be avoided in patients with pheochromocytoma. Of the listed relaxants in this question, vecuronium is the best choice; however, cisatracurium and rocuronium also have minimal effects on the cardiovascular system and would be good choices in patients with pheochromocytomas as well (Hines: Stoelting's Anesthesia and Co-Existing Disease, ed 5, p 392; Stoelting: Pharmacology and Physiology in Anesthetic Practice, ed 4, pp 223-241).
question
480. In a given patient, if a creatinine of 1.0 corresponds to a glomerular filtration rate (GFR) of 120 mL/min, a creatinine of 4.0 would correspond to A. 20 mL/min B. 30 mL/min C. 40 mL/min D. 50 mL/min E. 60 mL/min
answer
480. (B) Serum creatinine is inversely proportional to the GFR. With the increase in creatinine by a factor of 4, the GFR is divided by 4; i.e., 120/4 = 30 mL/min. (Lobato: Complications in Anesthesiology, p 433; Miller: Anesthesia, ed 6, pp 786-788).
question
481. The incidence of each of the following is increased in patients with Down syndrome (trisomy 21) EXCEPT A. Malignant hyperthermia B. Hypothyroidism C. Smaller trachea D. Occipito-atlantoaxial instability E. Congenital heart disease
answer
481. (A) Trisomy 21 or Down syndrome is the most common human chromosomal syndrome seen. An increase incidence of congenital hypothyroidism occurs. About one fourth of children with Down syndrome and many adults have smaller tracheas than predicted and require an endotracheal tube that is one or two sizes smaller. One should avoid unnecessary flexion or extension of the neck during intubation because occipito-atlantoaxial instability occurs in about 15% to 20% of patients. Because subluxation is relatively uncommon, routine neck radiographs for all Down patients are excessive. More than 40% of Down syndrome children have congenital heart disease, (e.g., endocardial cushion defects, ventricular septal defects, tetralogy of Fallot, patent ductus arteriosus). Although some children have hypotonia, an increased incidence of MH has not been reported in these patients (Baum: Anesthesia for Genetic, Metabolic, and Dysmorphic Syndromes of Childhood, ed 2, pp 105-107; Hines: Stoelting's Anesthesia and Co-Existing Disease, ed 5, pp 611-612; Miller: Anesthesia, ed 6, p 1099).
question
482. A 55-year-old male is to undergo a transurethral resection of the prostate gland under general anesthesia. The patient has a 40-pack-per-year smoking history and a history of congestive heart failure. The patient receives metoclopramide and scopolamine preoperatively. General anesthesia is induced with ketamine and the patient undergoes the procedure uneventfully. However, in the recovery room the patient complains of not being able to see objects "up close." Which of the following would be the most likely cause of this complaint? A. Emergence delirium from ketamine anesthesia B. Effect of scopolamine C. Effect of glycine in the irrigating solution D. Corneal abrasion E. Hyponatremia
answer
482. (B) Scopolamine is an anticholinergic that may produce mydriasis and cycloplegia. This can result in the inability of patient's eyes to accommodate (Stoelting: Basics of Anesthesia, ed 5, p 172).
question
483. Malignant hyperthermia and neuroleptic malignant syndrome share each of the following characteristics EXCEPT A. Generalized muscular rigidity B. Hyperthermia C. Effectively treated with dantrolene D. Tachycardia E. Flaccid paralysis after administration of vecuronium
answer
483. (E) Neuroleptic malignant syndrome is a potentially fatal disease that affects 0.5% to 1% of all patients being treated with neuroleptic (antipsychotic) drugs. The syndrome develops gradually over 1 to 3 days in young males and is characterized by the following: (1) hyperthermia, (2) skeletal muscle rigidity, (3) autonomic instability manifested by changes in blood pressure and heart rate, and (4) fluctuating levels of consciousness. The mortality from neuroleptic malignant syndrome is 20% to 30%. Liver transaminases and creatine phosphokinase levels are often elevated in these patients. Treatment includes supportive care and administration of dantrolene. This disease may mimic MH because of its many similarities. One difference between neuroleptic malignant syndrome and MH is the fact that nondepolarizing muscle relaxants such as vecuronium or cisatracurium will cause flaccid paralysis in patients with neuroleptic malignant syndrome but not in patients with MH (Stoelting: Pharmacology and Physiology in Anesthetic Practice, ed 4, p 412).
question
484. A 23-year-old male involved in a motor vehicle accident is brought to the OR for open reduction and internal fixation of bilateral leg fractures under general anesthesia. During the surgery the patient is transfused with 7 units of type AB, Rh-negative packed RBCs and 3 units of platelets. At the end of the procedure the endotracheal tube is removed and the patient is taken to the ICU. Postoperatively the patient complains of shortness of breath and arterial hypoxemia is noted. His temperature is 38° C, heart rate is 146 beats/min, blood pressure is 105/69 mm Hg, and respiratory rate is 36 breaths/min. In addition, the patient is noted to have a fine petechial rash on his neck, chest and shoulders. Which of the following is the most likely cause of these signs and symptoms? A. Pulmonary embolism B. Transfusion reaction from packed RBCs C. Transfusion reaction from platelets D. Fat embolism E. Sepsis
answer
484. (D) The classic signs of fat embolism include tachycardia, dyspnea, mental confusion, and fever, and frequently there may be a petechial rash on the upper part of the body. Fat embolism is more common after long bone fractures (e.g., femur and tibia) and usually occurs between 12 and 72 hours after long bone fractures (Hines: Stoelting's Anesthesia and Co-Existing Disease, ed 5, p 193).
question
485. Remifentanil is metabolized primarily by A. Kidneys B. Liver C. Hoffman elimination D. Pseudocholinesterase E. Nonspecific esterases
answer
485. (E) Remifentanil is an ultrashort-acting narcotic. Chemically it is a derivative of piperidine (like fentanyl), but remifentanil has an ester linkage and is rapidly broken down by nonspecific plasma as well as tissue esterases. The elimination half-life is less than 20 minutes and is best administered by a continuous infusion. Pseudocholinesterase deficiency or renal or hepatic failure does not affect remifentanil's rapid metabolism (Barash: Clinical Anesthesia, ed 5, pp 371, 374; Stoelting: Pharmacology and Physiology in Anesthetic Practice, ed 4, p 114).
question
486. A 3-year-old child is brought to the OR after aspiration of a peanut. After an inhalation induction the trachea is intubated. The peanut is extracted through a rigid bronchoscope but then is lost in the upper airway. The anesthesiologist notes that he can no longer ventilate the patient's lungs. What should be the next step in the management of this problem? A. Needle cricothyroidotomy B. Emergency tracheotomy C. Placement of a chest tube D. Push the peanut more distally E. Attempt jet ventilation
answer
486. (D) If a peanut or other foreign body becomes lost in the upper airway such that ventilation of the patient is impossible and retrieval is not feasible, the person performing the bronchoscopy should push the foreign body distally past the carina so that gas exchange can take place. Once the patient is stabilized, another attempt to retrieve the foreign body can be made (Motoyama: Smith's Anesthesia for Infants and Children, ed 7, pp 815- 818).
question
487. Patients who undergo extracorporeal shock-wave lithotripsy are at increased risk for A. Venous air embolism B. Pneumothorax C. Peripheral neuropathies D. Postdural puncture headache with spinal anesthesia E. Hypotension with regional anesthesia at the end of the procedure
answer
487. (E) Anesthesia for extracorporeal shock wave lithotripsy may be accomplished with either general anesthesia or epidural anesthesia. When a patient is submerged in the stainless steel tub, the peripheral vasculature becomes compressed by the hydrostatic pressure, resulting in an increase in preload. Removing the patient from the tank has the opposite effect. In patients who have received epidural anesthesia, there is an increased incidence of hypotension caused by epidural-induced sympathectomy after they emerge from the bath (Barash: Clinical Anesthesia, ed 5, pp 1030-1032).
question
488. The most common reason for admitting outpatients to the hospital following general anesthesia is A. Angina B. Inability to void C. Inability to ambulate D. Surgical pain E. Nausea and vomiting
answer
488. (E) The most common reason for unexpected hospital admission after outpatient general anesthesia, as well as a prolonged recovery-room stay (for both adults and children), is nausea and vomiting. Two other reasons for a prolonged recovery-room stay are pain and drowsiness (Barash: Clinical Anesthesia, ed 5, p 1242).
question
489. A 37-year-old male with myasthenia gravis arrives in the emergency room confused and agitated after a 2- day history of weakness and increased difficulty breathing. ABGs on room air are PaO2 60 mm Hg, PaCO2 51mm Hg, HCO3? 25 mEq/L, pH 7.3, SaO2 of 90%. His respiratory rate is 30 breaths/min and tidal volume (VT) is 4 mL/kg. After administration of edrophonium 2 mg IV, his VT declines to 2 mL/kg. What should be the most appropriate step in the management of this patient at this time? A. Tracheal intubation and mechanical ventilation B. Repeat the test dose of edrophonium C. Administer neostigmine 1 mg IV D. Administer atropine 0.4 mg IV E. Emergency tracheostomy and mechanical ventilation
answer
489. (A) Cholinergic crisis can be differentiated from myasthenic crisis by administering small IV doses of anticholinesterases. With a cholinergic crisis, there are significant muscarinic effects (e.g., salivation, bradycardia, miosis) and an accentuated muscle weakness. Because this patient's VT decreased with the administration of edrophonium, the diagnosis of cholinergic crisis is made. Although atropine may be needed to treat the cholinergic symptoms, muscle weakness will be worse and these patients need to be intubated until the muscle strength returns (Hines: Stoelting's Anesthesia and Co-Existing Disease, ed 5, p 452).
question
490. Select the FALSE statement regarding tramadol (Ultram). A. Ondansetron may interfere with part of tramadol's analgesia B. Tramadol is associated with seizures in patients taking selective serotonin reuptake inhibitors (SSRIs) C. It exhibits monoamine oxidase (MAO) inhibiting effects D. Its analgesic effects are partially antagonized by naloxone E. It is relatively safe in patients whose pain makes them suicidal
answer
490. (E) Tramadol, a synthetic codeine analogue, is a centrally acting analgesic. It can be used for mild to moderate pain but is not as effective as morphine or meperidine for severe or chronic pain. One drawback for tramadol's perioperative use is its high incidence of nausea and vomiting. Its mechanism of action for analgesia is complex. It is a weak ?-receptor agonist, it inhibits serotonin and norepinephrine reuptake, and it enhances serotonin release. Tramadol-induced analgesia is not entirely reversed with naloxone, however, the respiratory depression and sedation can be reversed. Ondansetron, a serotonin antagonist, may interfere with part of tramadol's analgesic action. Because of its low ?-receptor agonist activity, it may be less likely to produce physical dependence than other stronger narcotics. Seizures have been reported in patients receiving tramadol alone. The drug should be used with caution in patients taking drugs that lower the seizure threshold, such as tricyclic antidepressants and SSRIs. It has some MAO inhibiting activity and should not be used in patients taking MAO inhibitors. Another warning is its use in patients who are depressed or suicidal. Tramadol is not recommended in depressed or suicidal patients since excessive doses, either alone or with other CNS depressants including alcohol, are a major cause of drug-related deaths with fatalities reported within the first hour of overdosage. Patients who are depressed or suicidal are better managed with non- narcotic analgesics (Hardman: Goodman and Gilman's The Pharmacological Basis of Therapeutics ed 10, p 590; Physicians Desk Reference-2009, ed 63, pp 2428-2431; Stoelting: Pharmacology and Physiology in Anesthetic Practice, ed 4, p 117).
question
491. Which of the following patients would not be a good candidate for outpatient inguinal hernia repair under general anesthesia? A. A 62-year-old pharmacist who lives 10 miles away B. A 20-year-old healthy college student who had a renal transplant 3 years earlier C. A 38-year-old housewife with a hiatal hernia D. A premature infant who is 43 weeks postconceptual age E. A 29-year-old diabetic who is well controlled on insulin
answer
491. (D) Premature infants are at increased risk for development of apnea until they have reached 60 weeks postconceptual age (PCA), which is defined as gestational age at birth (GA) plus chronologic age. Apnea in these patients is central apnea, that is, apnea associated with the absence of respiratory effort related to immaturity of the CNS. Infants with a history of apnea as well as anemia (Hct <30) are especially at risk. Because postoperative apnea is highest in the first 4 to 6 hours and may present up to 12 hours after surgery, monitoring of all infants (38 weeks GA) that have not yet reached 44 weeks PCA and healthy preterm infants (<38 weeks) that have not reached 50 weeks PCA are admitted for overnight monitoring. See also question 631 (Motoyama: Smith's Anesthesia for Infants and Children, ed 7, pp 24-25, 562-563, 875- 876, 1163-1164).
question
492. A 72-year-old male undergoes emergency repair of an abdominal aortic aneurysm. In the first hour after release of the suprarenal cross-clamp, urine output is only 10 mL. After administration of furosemide 20 mg IV, urine output increases to 100 mL/hr. Urine [Na+] is 43 mEq/L and urine osmolality is 210 mOsm/L. The most likely cause of the initial oliguria is A. Fluoride toxicity B. Renal hypoperfusion C. Acute tubular necrosis D. Increased ADH E. Impossible to differentiate
answer
492. (E) In the absence of diuretics, oliguria associated with urine sodium concentration greater than 40 mEq/L and urine osmolality less than 400 mOsm/L is strongly suggestive of intrinsic renal disease (e.g., acute tubule necrosis) whereas prerenal causes have urine sodium concentration less than 20 mEq/L and urine osmolality greater than 400 mOsm/L. Furosemide, mannitol and dopamine, however, obscures the accurate diagnosis (Hines: Stoelting's Anesthesia and Co-Existing Disease, ed 5, pp 325-327; Stoelting: Basics of Anesthesia, ed 5, pp 430-431).
question
493. A healthy 25-year-old man is anesthetized for a sagittal split osteotomy. Anesthesia is induced with propofol, morphine, and vecuronium and maintained with 1.5% isoflurane and 50% N2O. After induction, the nose is prepped with 4% lidocaine and 1% phenylephrine, and the patient is intubated through the right naris. Before emergence, the surgeon performs a bilateral inferior alveolar nerve block. The patient is reversed with neostigmine and glycopyrrolate. When the patient awakens he is noted to have an 8-mm pupil on the right and a 3-mm pupil on the left. Results of physical examination are otherwise unremarkable. The most likely explanation for the dilated pupil is A. Right stellate ganglion block B. Accidental introduction of lidocaine into right eye C. Accidental introduction of phenylephrine into right eye D. Right ciliary ganglion block E. Glycopyrrolate
answer
493. (C) In an unconscious patient, a unilateral dilated pupil would be a matter of grave concern. In an awake patient with a normal neurologic examination, however, it is less worrisome. An inferior alveolar nerve block involves injection of about 2 mL of 2% lidocaine around the inferior alveolar nerve just behind the molars in the lower jaw. Even a grossly misdirected needle probably could not reach the stellate ganglion, but were it possible, the result would be a Horner's syndrome (miosis, not mydriasis, ptosis, anhidrosis, and vasodilation over the face). Blockade of the ciliary ganglion could cause mydriasis on the ipsilateral side, but reaching the ciliary ganglion, located between the optic nerve and lateral rectus muscle about 1 cm from the posterior limit of the orbit, would be almost impossible with a needle directed toward the mandible. Glycopyrrolate administered systemically does not cause mydriasis, as it is not capable of crossing the blood-brain barrier. Lidocaine instilled directly into the eye does not produce mydriasis, but phenylephrine does. Care must be taken not to spray local anesthetic (with or without vasoconstrictor) into the eyes while applying topical anesthesia to the nares (Stoelting: Pharmacology and Physiology in Anesthetic Practice, ed 4, p 304).
question
494. A 40-year-old male is undergoing a left inguinal hernia repair under general anesthesia in San Diego, Calif. N2O is administered at 3 L/min, O2 at 1 L/min, and isoflurane at 0.85%. What minimum alveolar concentration (MAC) is this patient receiving? A. 0.8 B. 1.25 C. 1.50 D. 1.75 E. 2.0
answer
494. (C) MAC is the minimum alveolar concentration of anesthetic that will prevent movement of 50% of patients when a skin incision is made at sea level (e.g., San Diego). MAC × 1.3 will prevent movement in 95% of patients. In this question, total gas flow is 4 L/min (1 L/min + 3 L/min). Roughly 75% of the total gas is N2O. The MAC of N2O is 104%. The patient is receiving about 0.75 MAC N2O. The MAC for isoflurane is 1.15. A concentration of 0.85% would represent 0.75 MAC. Because MACs are additive, the total MAC would be 1.5 (Barash: Clinical Anesthesia, ed 5, pp 397-398; Stoelting: Basics of Anesthesia, ed 5, p 82).
question
495. An otherwise healthy 140-kg, 24-year-old male is scheduled for thyroid surgery under general anesthesia. Which of the following statements concerning his cardiac output at 140 kg compared with his cardiac output at his ideal body weight (70 kg) is correct? A. Cardiac output is diminished by factor of 2 B. Cardiac output is diminished by 10% C. Cardiac output is the same D. Cardiac output is increased by 10% E. Cardiac output is doubled
answer
495. (E) Cardiac output increases by about 100 mL/min for each kilogram of weight gained. It is estimated that every (E) kilogram of adipose tissue contains nearly 3000 m of additional blood vessels. The additional cardiac output is due to ventricular dilation and increased stroke volume, as resting heart rates are not increased in obese patients (Hines: Stoelting's Anesthesia and Co-Existing Disease, ed 5, p 302; Stoelting: Basics of Anesthesia, ed 5, p 448).
question
496. Fenoldopam may be used as an alternative to which of the following? A. Epinephrine B. Phenylephrine C. Dopexamine D. Dopamine E. Sodium nitroprusside
answer
496. (E) Fenoldopam (Corlopam) is a selective dopamine-1 receptor agonist with significant vasodilating properties. It has moderate affinity for ?2-receptors but has no affinity for dopamine-2, ?1, ?, 5-HT1, or 5-HT2 receptors. It is used for treatment of patients with severe hypertension (especially with reduced renal function) and is administered as an IV infusion. It can be used as an alternative to sodium nitroprusside and has the advantage of no thiocyanate toxicity, rebound effect, or "coronary steal" effect, but it does contain sodium bisulfite and is contraindicated in patients with a known sulfite sensitivity. Dopexamine (Dopacard) is a synthetic analogue related to dopamine with intrinsic activity at dopamine as well as ?2-receptors and is used as an inotropic agent (Hardman: Goodman ; Gilman's The Pharmacologic Basis of Therapeutics, ed 10, p 227; Miller: Anesthesia, ed 6, pp 649-650; Stoelting: Pharmacology and Physiology in Anesthetic Practice, ed
question
497. A 58-year-old hemophiliac is scheduled for total knee arthroplasty. His factor VIII levels are 35% of normal. Which of the following would be the most appropriate therapy before surgery? A. Administer sufficient cryoprecipitate to raise factor VIII levels to 50% normal B. Administer sufficient factor VIII concentrate and platelets to raise levels to 50% normal C. Transfuse fresh frozen plasma until factor VIII levels are 100% normal D. Administer factor VIII concentrates until levels are 100% normal E. None of the above
answer
497. (D) Ideally, factor VIII levels should be raised to 100% predicted before elective surgery to ensure that the levels will not fall below 30% intraoperatively. Thirty percent of the normal factor VIII concentration or greater is thought to be necessary for a patient who is to undergo major surgery. Elimination half-time of factor VIII is 12 hours. This may be accomplished with factor VIII concentrate or cryoprecipitate. Fresh frozen plasma is no longer considered therapy for hemophilia (Hines: Stoelting's Anesthesia and Co-Existing Disease, ed 5, pp 419-420).
question
498. A 16-year-old boy whose maternal uncle has hemophilia A is scheduled for wisdom tooth extraction. Which test below would be the best screening test for hemophilia A? A. Partial thromboplastin time (PTT) B. Prothrombin time (PT) C. Thrombin time D. Platelet count E. Bleeding time
answer
498. (A) Hemophilia A is associated with decreased levels of factor VIII. PTT tests the intrinsic coagulation cascade (A) and would be abnormally elevated in all but the most mild disease. A normal PTT is 25 to 35 seconds. Platelet count PT and bleeding times are normal (Hines: Stoelting's Anesthesia and Co-Existing Disease, ed 5, p 420; Kasper: Harrison's Principles of Internal Medicine, ed 16, pp 680-681).
question
499. The reason four twitches are used in the train-of-four to determine degree of neuromuscular blockade versus five (or more) is A. Comparison of greater than four twitches is too difficult B. Four twitches informs the user of the degree of blockade in the useful clinical range (i.e., 75% to 100% blockade) C. Post-tetanic facilitation will begin to appear after four twitches D. Additional twitches may damage the nerve by overstimulation E. There would be no additional decrement in twitch height after four twitches
answer
499. (E) Conventional peripheral nerve stimulators deliver four twitches at 2 Hz spaced 0.5 second apart. These devices were designed with the knowledge that successive twitches deplete acetylcholine stores. After the fourth twitch, there is no additional decrement in twitch height (Stoelting: Basics of Anesthesia, ed 5, p 150).
question
500. A 57-year-old male is undergoing a right hemicolectomy under general anesthesia. The patient has no history of cardiac disease. During the operation 5-mm ST-segment elevation is noted on lead II and the patient develops complete heart block. The coronary artery most likely affected is A. Circumflex coronary artery B. Right coronary artery C. Left main coronary artery D. Left anterior descending coronary artery E. Branch to obtuse margin
answer
500. Inferior ischemia is associated with blockage or spasm of the right coronary artery. The right coronary artery (B) supplies blood to the atrioventricular node in 90% of patients. Complete heart block therefore is not unexpected in patients with severe CAD involving the right coronary artery (Hines: Stoelting's Anesthesia and Co-Existing Disease, ed 5, p 19).
question
501. Each of the following may increase MAC for volatile anesthetics EXCEPT A. Cocaine B. Hyperthyroidism C. Monoamine oxidase (MAO) inhibitor therapy D. Tricyclic antidepressants E. Hypernatremia
answer
501. (B) Minimum alveolar concentration (MAC) is influenced by a variety of disease states, conditions, drugs, and (B) other factors. Drugs that increase CNS catecholamines, such as MAO inhibitors, tricyclic antidepressants, acute amphetamine ingestion, and cocaine, increase MAC. Other factors that increase MAC include hyperthermia, hypernatremia, patients with natural red hair, and infancy. It is interesting that MAC values are higher for infants than for neonates or older children and adults. Thyroid gland dysfunction including hyperthyroidism does not affect the MAC. Factors that lower MAC include narcotics, IV anesthetics, local anesthetics (except cocaine) and other sedatives, age (6% per decade), hypothermia, hypoxia, severe anemia (e.g., Hgb < 5). The following table modified from the references in this question summarizes the impact of various factors on MAC (Barash: Clinical Anesthesia, ed, 5 pp 397-398; Morgan: Clinical Anesthesiology, ed 4, p 165; Stoelting: Basics of Anesthesia, ed 5, p 83
question
502. A 37-year-old patient with history of manic-depressive illness is scheduled to undergo surgery for removal of an intramedullary rod in the left tibia. Which of the following statements regarding potential untoward effects of lithium therapy is NOT true? A. Long-term administration may be associated with nephrogenic diabetes insipidus B. Administration of succinylcholine to patients treated with lithium may result in hyperkalemia C. Long-term therapy may be associated with hypothyroidism D. Duration of action of pancuronium may be prolonged E. Administration of thiazide diuretics may increase plasma lithium concentrations
answer
502. (B) Long-term lithium therapy in patients with manic-depressive illness may be associated with nephrogenic (B) diabetes insipidus. Hypothyroidism may develop in about 5% of patients because lithium can inhibit the release of thyroid hormones. Lithium is almost 100% renally excreted. Reabsorption occurs at the proximal convoluted tubule and is inversely related to the concentration of sodium in the glomerular filtrate. Consequently, administration of diuretics (mainly thiazide, but to a lesser extend loop diuretics) may lead to the development of toxic lithium levels. Lithium has sedative properties and may reduce the need for IV and inhalational anesthetic agents. It may prolong the duration of action of both pancuronium and succinylcholine, but it is not associated with an exaggerated release of potassium when succinylcholine is administered (Hardman: Goodman & Gilman's The Pharmacological Basis of Therapeutics, ed 10, pp 508-509; Hines: Stoelting's Anesthesia and Co-Existing Disease, ed 5, p 539).
question
503. Treatment of hypotension in a patient anesthetized for resection of metastatic carcinoid would be best accomplished with A. Epinephrine B. Ephedrine C. Vasopressin (DDAVP) D. Angiotensin E. Octreotide
answer
503. (E) Carcinoid tumors can arise wherever enterochromaffin cells are present. Most (>70%) originate in the intestine (E) and about 20% originate in the lung. Of those that originate in the gastrointestinal tract, 50% occur in the appendix, 25% in the ileum, and 20% in the rectum. These interesting tumors were called carcinoid because they were originally believed not to metastasize. We now know this is not true. The hormones released by the nonmetastatic tumors reach the liver by the portal vein and are rapidly inactivated. However, once metastases reach the liver, the released hormones reach the systemic circulation and produce signs and symptoms of the "carcinoid syndrome." Symptoms include cutaneous flushing, abdominal pain, vomiting, diarrhea, hypotension or hypertension, bronchospasm, and hyperglycemia. The natural hormone somatostatin suppresses the release of serotonin and other vasoactive substances from the tumor. Because the half-life is about 3 minutes, somatostatin is given by infusion. Octreotide is a synthetic somatostatin analogue with a half-life of 2.5 hours and is given SQ or IV for the prevention and treatment of carcinoid symptoms (e.g., hypotension, hypertension, bronchospasm). However, the treatment of hypotension in patients with carcinoid disease is different because ephedrine, epinephrine, and norepinephrine can release vasoactive hormones from the tumor and make the hypotension worse. Hypotension is best treated with fluids and IV octreotide or somatostatin. Hypertension is treated with deepening the anesthetic and administering octreotide, somatostatin or labetalol. Bronchospasm is treated with IV octreotide, somatostatin or nebulized ipratropium. When giving anesthesia to these patients it is probably wise to avoid drugs that release histamine and other vasoactive hormones that may precipitate symptoms. Propofol or etomidate are good induction agents, followed by maintenance anesthesia with a volatile anesthetic (e.g., isoflurane, sevoflurane or desflurane) and/or nitrous oxide with oxygen. Vecuronium, cisatracurium and rocuronium appear to be safe muscle relaxants. Fentanyl, sufentanil, alfentanil and remifentanil and benzodiazepines are also safe to use. The serotonin antagonist ondansetron is a useful antiemetic (Barash: Clinical Anesthesia, ed 5, pp 1058-1059; Hines: Stoelting's Anesthesia and Co-Existing Disease, ed 5, pp 289-291; Physicians Desk Reference-2009, ed 63, pp 2300- 2306).
question
504. A 75-year-old male patient is scheduled to undergo elective orchiectomy for prostate cancer. The patient has selected spinal anesthesia. What is the minimum dermatomal level that must be achieved to carry out this operation? A. T1 B. T4 C. T10 D. L3 E. S1
answer
504. (C) Testicular innervation can be traced up to the T10 dermatomal level. For this reason, any operation that (C) involves manipulation or traction on the testicles must have adequate anesthesia to prevent pain. This can be achieved with spinal or epidural anesthesia, which is associated with a T10 level of blockade (Barash: Clinical Anesthesia, ed 5, p 701).
question
505. A 31-year-old patient has been in the ICU on a ventilator for 24 hours after a motor vehicle accident. The patient does not open his eyes to any stimulus and has no verbal or motor response. The Glasgow Coma Scale corresponding to this patient would be A. 0 B. 1 C. 2 D. 3 E. 4
answer
505. (D) The Glasgow Coma Scale has three categories: eye opening for which a maximum of 4 points can be (D) received; best verbal response for a maximum of 5 points; and best motor response for a maximum of 6 points. The higher the score the better the response, the minimal score for each category is 1. Mild head injury scores are 13 to 15, moderate are 9 to 12 and severe are 3 to 8. This severe head-injured patient is totally unresponsive and would receive a score of 3 (Barash: Clinical Anesthesia, ed 5, p 782).
question
506. Hypoglycemia is more likely to occur in the diabetic surgical patient with which of the following diseases? A. Renal disease B. Rheumatoid arthritis requiring high-dosage prednisone C. Chronic obstructive lung disease treated with a terbutaline inhaler and aminophylline D. Manic-depressive disorder treated with lithium E. Congestive heart failure
answer
506. (A) Insulin metabolism involves both the liver and kidneys. Renal dysfunction, however, has a greater impact on (A) insulin metabolism than does hepatic dysfunction. In fact, unexpected prolonged effects of insulin sometimes are seen in patients with renal disease (Stoelting: Pharmacology and Physiology in Anesthetic Practice, ed 4, p 478).
question
507. Which of the following is most likely to be associated with a falsely elevated SaO2 as measured by pulse oximetry? A. Hemoglobin F B. Carboxyhemoglobin C. Bilirubin D. Fluorescein dye E. Methylene blue dye
answer
507.(B) Most pulse oximeters illuminate tissue with two wavelengths of light; 660-nm red light and 940-nm infrared (B) light. Since carboxyhemoglobin has an absorbance at 660 nm, very similar to O2 hemoglobin, it produces a falsely elevated SaO2 when present in the blood. Hemoglobin F, bilirubin, and fluorescein dye have no effect on pulse oximetry. Methylene blue, as well as indigo carmine and indocyanine green, lowers the SaO2 as measured by pulse oximetry. Methemoglobin absorbs red and infrared light equally well and gives saturation readings of 85% (Barash: Clinical Anesthesia, ed 5, pp 672, 1280; Stoelting: Basics of Anesthesia, ed 5, pp 312-313).
question
508. The most sensitive test for detecting primary hypothyroidism in the preoperative evaluation of a patient in whom hypothyroidism is suspected is A. Thyroid-stimulating hormone (TSH) level B. Total plasma T3 level C. Total plasma T4 level D. Resin T3 uptake E. Antithyroid antibodies
answer
508.(A) Hypothyroidism occurs when there is a decreased amount of the circulating thyroid hormones (T4, T3, or (A) both). It is estimated to occur in about 0.5% to 0.8% of the adult population. There are many causes of hypothyroidism, and they are classified into two main groups. The most common group (95% of cases) is primary hypothyroidism, where the thyroid gland is not capable of making enough hormones despite an adequate amount of TSH (thyrotropin). Secondary hypothyroidism (5% of cases) occurs when there is CNS dysfunction, i.e., hypothalamic or pituitary disease and low levels of TSH. It is the free and not total T3 or T4 level that parallels thyroid status. The T3 or T4 resin uptake test is used to measure free amounts of T3 or T4, but it is the TSH level that is used to differentiate primary from secondary hypothyroidism. Although antithyroid antibodies such as thyroid antimicrosomal antibody are seen in 95% of patients with multinodular diffuse goiter of Graves' disease, the most common type of hyperthyroidism, it also is seen in about 10% of adults with no thyroid disease (Barash: Clinical Anesthesia, ed 5, pp 1130-1131; Hines: Stoelting's Anesthesia and Co-Existing Disease ed 5, pp 384-386; Miller: Anesthesia, ed 6, pp 1047-1048).
question
509. Gabapentin (Neurontin) as used in the treatment of chronic pain belongs to the same broad class of drugs as A. Carbamazepine B. Imipramine C. Clonidine D. Aspirin E. Fluoxetine (Prozac)
answer
509. (A) Gabapentin, an anticonvulsant, was developed to be a centrally active GABA agonist but does not appear to (A) interact with GABA receptors. Its mechanism for producing analgesia is unclear, but it may involve inhibition of voltage-activated calcium channels as well as potentiating GABA release. Carbamazepine slows the recovery rate of voltage-gated sodium channels, but it also is an anticonvulsant. Carbamazepine is indicated in the treatment of trigeminal neuralgia (Stoelting: Pharmacology and Physiology in Anesthetic Practice, ed 4, pp 571-575; Loeser: Bonica's Management of Pain, ed 3, pp 106, 1728-1730).
question
510. Which of the following medical conditions would not prompt the administration of prophylactic antibiotics for this 67-year-old patient scheduled to undergo a right hemicolectomy? A. A prosthetic cardiac valve B. Mitral valve prolapse C. Cardiac transplant recipient with cardiac valvulopathy D. Previous history of treated infective endocarditis E. All need prophylactic antibiotics
answer
510. (B) In 2007, the American Heart Association revised the guidelines for "Prevention of Infective Endocarditis." (B) Patients with mitral valve prolapse (MVP) do not need prophylaxis. Only patients in the highest risk groups, such as prosthetic cardiac valves, previous infective endocarditis, several types of congenital heart disease and cardiac transplantation recipients who develop cardiac valvulopathy need prophylactic antibiotics for elective GI or GU procedures (see also question 915) (Wilson W, Taubert KA, Gewitz, et al.; Prevention of infective endocarditis - Guidelines from the American Heart Association. Circulation, 2007; 115:1736-1754.
question
511. A 47-year-old morbidly obese patient develops bilateral blindness (only able to perceive light) after a 6 hour, 3 segment laminectomy and fusion. The patient received 6 units of blood and 5 L of Lactated Ringer's solution. A mean arterial blood pressure was maintained at 50 to 60 mm Hg. The most likely structure involved in this visual loss is A. Central retinal artery B. Optic nerve C. Retina D. Cerebral cortex E. Central retinal vein
answer
511. (B) Perioperative visual loss associated with non-ocular surgery is rare and may result from corneal trauma, retinal artery occlusion, retinal vein occlusion, optic nerve ischemia or cortical disease. Although overall a rare problem, it may develop in up to 1% of prone spinal surgical cases and is most commonly due to ischemic optic neuropathy. The cause is unknown and multifactorial. Associated factors include prolong intraoperative hypotension, anemia (Hgb ; 8), large intraoperative blood loss, prolonged surgery and facial edema. It is more common in males and in patients with peripheral vascular disease, diabetes mellitus, and tobacco users (Lobato: Complications in Anesthesiology pp 418-422; Miller: Anesthesia, ed 6, pp 3005-3011).
question
512. All of the following concerning postoperative shivering are true EXCEPT A. May increase metabolism and oxygen consumption significantly B. May be treated with meperidine C. May be treated with droperidol D. May be treated with clonidine E. Does not occur in the absence of hypothermia
answer
512. (E) Postoperative shivering or postanesthetic tremor can occur during recovery from all types of general anesthesia. If profound, shivering can increase metabolic rate and O2 consumption (100%-200%) with an associated increase in cardiac output and minute ventilation. Although shivering usually occurs in patients with decreased body temperature, it also may occur in patients with normal body temperature after anesthesia. Postanesthesia shivering is best treated by a combination of supplemental oxygen, rewarming the patient and/or administering IV meperidine. Other less frequently used pharmacologic treatments include clonidine, magnesium sulfate, calcium chloride, chlorpromazine, droperidol, and other opioids (e.g., butorphanol). Application of radiant heat to the face, head, neck, chest, and abdomen has been shown to eliminate shivering within minutes in postoperative patients, despite low core body temperatures (Longnecker: Principles and Practice of Anesthesiology, ed 2, pp 2318-2319; Morgan Clinical Anesthesiology, ed 4, pp 771- 772; Stoelting: Basics of Anesthesia, ed 5, pp 574-575).
question
513. Electrocardiographic (ECG) changes associated with hyperkalemia include A. Increased P wave amplitude B. Shortened PR interval C. Narrowed QRS complex D. Narrowed and peaked T waves E. Increase in U-wave amplitude
answer
513. (D) The ECG signs of hyperkalemia include narrowed and peaked T waves (earliest manifestation of hyperkalemia), decrease in P wave amplitude, prolonged PR interval, and a widened QRS interval. In extreme cases, the ECG can appear as a sine wave as well as cardiac arrhythmias (e.g., sinus arrest, supraventricular tachycardia, atrial fibrillation, PVCs, ventricular tachycardia and ventricular fibrillation). These changes are potentiated by hypocalcemia and intravenous calcium can rapidly correct some of these ECG changes. An increase in U-wave amplitude suggests hypokalemia, not hyperkalemia (Miller: Anesthesia, ed 6, p 1106).
question
514. A 24 year old is undergoing open reduction of an ankle fracture under general anesthesia with sevoflurane, N2O and O2 through an laryngeal mask airway (LMA). Just after the vaporizer dial is turned up to 2% the patient begins spontaneously breathing, but the inspiratory valve is not fully closing. The likely result of this (malfunctioning valve) is an increased in the inspired concentration of A. N2O B. CO2 C. O2 D. Sevoflurane E. All of the above
answer
514. (B) If the inspiratory valve becomes stuck in the open position, it will "malfunction" only during exhalation since, during inhalation, it is supposed to be open. During the exhalation phase of breathing, exhaled gases will exit through the expiratory valve into the expiratory limb of the circuit and beyond (proper path) as well as through the inspiratory valve into the inspiratory limb of the circuit (errant path). Gases traveling into the inspiratory limb (old gas) will be returned to the patient with next breath. The volume of recently exhaled gas is now drawn back into the patient's lungs along with the "new" gas that would be inspired in a fully functional breathing circuit. The net effect is that oxygen, sevoflurane, and N2O will all be diluted, but the patient rebreathes CO2, thus it will be the only gas with an increased inspired concentration (normal inspired CO2 is zero) as a result of the stuck inspiratory valve (Barash: Clinical Anesthesia, ed 5, p 580).
question
515. All of the following are associated with acromegalic patients undergoing transsphenoidal hypophysectomy EXCEPT A. Enlargement of the tongue and epiglottis B. Narrowing of the glottic opening C. Nasal turbinate enlargement D. 20% to 30% incidence of difficult intubation E. Continuous positive airway pressure (CPAP) should be used postoperatively since obstructive sleep apnea (OSA) is very common
answer
515. (E) Enlargement of the tongue and epiglottis predisposes the patient to upper airway obstruction and makes visualization of the vocal cords more difficult. The vocal cords are enlarged, making the glottic opening narrower. In addition, subglottic narrowing may be present as well as tracheal compression from an enlarged thyroid (seen in about 25% of acromegalic patients). This often necessitates the use of a narrower endotracheal tube than one might choose based on the facial enlargement. The placement of nasal airways may be more difficult due to the enlarged nasal turbinates. The use of CPAP is contraindicated after transsphenoidal hypophysectomy (Barash: Clinical Anesthesia, ed 5, pp 776, 1149; Fleisher: Anesthesia and Uncommon Diseases, ed 5, pp 21-22; Hines: Stoelting's Anesthesia and Co-Existing Disease, ed 5, pp 402- 403).
question
516. Evidence of an anaphylactic reaction to atracurium 1 to 2 hours after the episode could be best established by measuring blood levels of A. Tryptase B. Laudanosine C. Histamine D. Bradykinin E. Cortisol
answer
516. (A) There are four types of immune mediated allergic reactions. Anaphylaxis is a Type I IgE mediated reaction (A) that involves mast cells and basophils. Anaphylactoid reactions appear like anaphylaxis but are not immune mediated. Tryptase is a neutral protease normally stored in mast cells but is released into systemic circulation during anaphylactic but not anaphylactoid reactions. Tryptase levels would need to be measured within 1 to 2 hours of the suspected allergic reaction. Plasma histamine levels return to baseline within 30 to 60 minutes of an anaphylactic reaction. Laudanosine is a normal metabolic product of atracurium metabolism (Hines: Stoelting's Anesthesia and Co-Existing Disease, ed 5, p 526; Morgan: Clinical Anesthesiology, ed 4, p 972).
question
517. Which of the following findings is NOT consistent with a diagnosis of malignant hyperthermia? A. PaCO2 150 mm Hg B. MVO2 50 mm Hg C. pH 6.9 D. Arterial oxygen saturation 85% on 100% FIO2 E. Onset of symptoms an hour after end of operation
answer
517. (B) Signs of MH reflect the hypermetabolic state (up to 10 times normal) that develops. Clinical signs include (B) tachycardia, tachypnea, arterial hypoxemia, hypercarbia (e.g., PaCO2 100-200 mm Hg), metabolic and respiratory acidosis (e.g., pH 6.80-7.15), hyperkalemia, hypotension, muscle rigidity, trismus after succinylcholine administration, and increased body temperature. Mixed venous oxygen tension would be very low. The clinical presentations are quite variable and some reactions may not develop until the postoperative period (Hines: Stoelting's Anesthesia and Co-Existing Disease, ed 5, pp 620-622).
question
518. A 52-year-old business executive undergoes a radical retropubic prostatectomy uneventfully under general isoflurane anesthesia. He takes fluoxetine (Prozac) for depression. Upon discharge, which of the following analgesics would be the best choice for post-operative pain management in this patient? A. Oxycodone plus aspirin (Percodan) B. Hydrocodone with acetaminophen (Vicodin) C. Codeine with acetaminophen (Tylenol #3) D. Hydromorphone (Dilaudid) E. All would be equally effective
answer
518. (D) The selective serotonin reuptake inhibitor (SSRI) fluoxetine, is one of the most potent inhibitors of the (D) cytochrome P-450 enzymes CYP3A4 and CYP2D6. CYP2D6 facilitates the conversion of codeine to morphine, meaning the response from a "normal" dose would be less than expected because of decreased conversion. Oxycodone and hydrocodone are metabolized by CYP2D6 to their active form as well and a "normal" dose of these would give less response than expected. Thus, codeine, oxycodone and hydrocodone would be poor analgesic choices for patients taking SSRIs. CYP3A4 is responsible for the metabolism of fentanyl, sufentanil and alfentanil. Remifentanil is metabolized by non-specific plasma esterases (Miller: Anesthesia, ed 6, p 101).
question
519. Anesthesia is induced in a 50-year-old, 125-kg man for anterior cervical fusion. The patient is placed on a ventilator. Peak airway pressure is noted to be 20 cm H2O with O2 saturation 99% on pulse oximeter. An hour later, the peak airway pressure rises to 40 cm H2O, PaCO2 is 38 mm Hg on mass spectrometer, on O2 saturation falls to 88%. Blood pressure and heart rate are unchanged. The most likely cause of these findings is? A. Mainstem intubation B. Thrombotic pulmonary embolism C. Tension pneumothorax D. Venous air embolism E. Laboratory error
answer
519. (A) Symptoms of a mainstem or bronchial intubation include asymmetric chest expansion, unilateral breath (A) sounds, elevation of peak airway pressures, and arterial blood gas abnormalities (e.g., hypoxemia). Frequently, bronchial intubation is intentional (e.g., thoracic surgery with double lumen endotracheal tubes) but if undetected with a single lumen tube, atelectasis, hypoxia and pulmonary edema may result in time. Peak airway pressures can also increase with many conditions such as airway obstruction (e.g., kinked endotracheal tube, secretions, overinflated cuffs), bronchospasm, increasing VT, increase in chest wall muscle tone (rigid chest with narcotics, coughing), and tension pneumothorax. If a tension pneumothorax develops, associated hypotension usually is present. Pulmonary embolism would not cause the peak airway pressure to rise as in this case (Lobato: Complications in Anesthesiology, pp 101-102).
question
520. The phase of liver transplantation where the greatest degree of hemodynamic instability is expected is A. Induction B. Dissection phase C. Anhepatic phase D. Reperfusion phase E. Emergence
answer
520. (D) Although hemodynamic instability can occur at any time during liver transplantation, it is during the initial part (D) of the reperfusion phase, when the vascular clamps are removed from the liver graft, when cardiovascular instability is most marked. At this time there can be profound hypotension, reduced cardiac contractility, cardiac arrhythmias as well as hyperkalemic cardiac arrest. Epinephrine, atropine, calcium, and sodium bicarbonate should be available, as well as blood products, during this critical part of the surgery (Miller: Anesthesia, ed 6, pp 2249-2252; Stoelting: Basics of Anesthesia, ed 5, p 534).
question
521. Which of the following drugs is (are) likely to prolong nondepolarizing neuromuscular blockade? A. Prednisone B. Diltiazem C. Clindamycin D. Dantrolene E. All of the above
answer
521. (E) Metabolic and physiologic conditions as well as certain medications can contribute to a prolonged duration of (E) action of nondepolarizing neuromuscular blockade. Metabolic and physiologic conditions include respiratory acidosis, myasthenia syndromes, hepatic/renal failure, hypocalcemia, hypothermia, and hypermagnesemia. Both inhaled and local anesthetics as well as corticosteroids, many antibiotics (e.g., polymyxins, aminoglycosides, lincosamines [e.g., clindamycin], metronidazole [Flagyl]), calcium channel blockers, dantrolene and furosemide can prolong nondepolarizing neuromuscular blockade (Stoelting: Basics of Anesthesia, ed 5, p 566).
question
522. Drugs considered suitable for patients who are susceptible to malignant hyperthermia MH include all EXCEPT A. Etomidate B. N2O C. Calcium chloride D. Ketamine E. All of the above are safe
answer
522. (E) Drugs considered unsafe for patients susceptible to MH include all of the volatile anesthetics and the (E) depolarizing muscle relaxant succinylcholine. All local anesthetics (both amide and ester), N2O, opiates, barbiturates, ketamine, propofol, vasoactive drugs, calcium salts, antibiotics, antihistamines, nondepolarizing muscle relaxants, as well as drugs used to reverse nondepolarizing muscle relaxants, are safe (i.e., nontriggering) in patients susceptible to MH (Barash: Clinical Anesthesia, ed 5, pp 533-534; Miller: Anesthesia, ed 6, p 1182).
question
523. Near the end of a three-hour colectomy the surgeon complains that the patient is not relaxed. Two twitch monitor placed at different locations show only one twitch of a train of four. Blood gases are reported to be pH 6.9, CO2 82, K 4.6 and acetate 4.6. Most appropriate action would be A. Administer more vecuronium B. Administer bicarbonate C. Administer succinylcholine D. Increase minute ventilation E. Administer dantrolene
answer
523. (E) Rare muscle diseases can have dramatic anesthetic implications. Malignant hyperthermia (MH) is among the (E) most important manifestations of a muscular disorder. Malignant hyperthermia is thought to be caused by alterations in calcium control in muscle sarcoplasmic reticulum in response to succinylcholine or potent volatile anesthetics (most likely mediated by mutations of the ryanodine receptor). Because MH is a disorder in muscle metabolism, rigidity during administration of a volatile anesthetic or after succinylcholine use may be the presenting sign. Additionally, administration of any muscle relaxant would not provide muscle relaxation and succinylcholine would be contraindicated. The patient does have a respiratory and metabolic acidosis and significantly increasing minute ventilation with 100% oxygen and the use of sodium bicarbonate would be needed; however, stopping the triggering agent and administration of dantrolene is most important (Hines:
question
524. A 22-year-old parturient is anesthetized for an emergency laparoscopic cholecystectomy. She is in the 24th week of gestation and receives general sevoflurane anesthesia and has received rocuronium for muscle relaxation. Just prior to emergence, muscle relaxation is reversed with glycopyrrolate and neostigmine. Three minutes later, the fetal heart rate falls to 88 beats per minute. The most likely cause of this is A. Fetal head compression B. Maternal hypoxia C. Fetal hypoxia D. Uteroplacental insufficiency E. Reversal agents
answer
524. (E) Atropine and scopolamine cross the placenta easily, whereas glycopyrrolate is poorly transferred across the (E) placenta. Although neostigmine crosses the placenta poorly, enough does cross the placenta and can cause fetal bradycardia in utero. That is why it is better to reverse muscle relaxants in pregnant patients for non-delivery surgery with neostigmine and atropine (Chestnut: Obstetric Anesthesia, ed 3, p 60).
question
525. Each of the following is associated with an increased risk of nausea and vomiting with anesthesia EXCEPT A. Smoking B. Female gender C. History of motion sickness D. History of postoperative nausea and vomiting E. Postoperative codeine
answer
525. (A) The overall estimated incidence of postoperative nausea and vomiting (PONV) for all surgical procedures is (A) 25% to 30%. Several factors are associated with an increased risk of PONV including patient related factors (e.g., a previous history of PONV, history of motion sickness, nonsmoking, female gender, pregnancy, obesity), certain surgical procedures (e.g., eye muscle surgery, ear, nose, and throat (ENT), dental), as well as postoperative pain and narcotic use (Lobato: Complications in Anesthesiology pp 571-573; Stoelting: Basics of Anesthesia, ed 5 p 575).
question
526. Ketorolac is contraindicated in patients undergoing scoliosis surgery because of A. Renal effects B. Risk of postoperative hemorrhage C. Effects on bone healing D. Effects on pulmonary function E. Paradoxical effect with bone pain
answer
526. (C) Ketorolac is one of the few nonsteroidal anti-inflammatory drugs (NSAIDs) approved for parenteral use. (C) Although NSAIDs have analgesic and anti-inflammatory effects without ventilatory depression; they also inhibit platelet aggregation, can produce gastric ulceration, are associated with renal dysfunction and may impair bone healing. Nonsteroidal anti-inflammatory drugs are contraindicated in patients undergoing spinal fusion, where bone healing is essential to a successful surgical procedure (Stoelting: Pharmacology and Physiology in Anesthesia Practice, ed 4, pp 276-282, 2872-2888).
question
527. Causes of sickling in patients with sickle-cell anemia include all of the following EXCEPT A. Inhaled nitric oxide B. Dehydration C. Metabolic acidosis D. Hypothermia E. Hypoxemia
answer
527.(A) Sickle-cell anemia is an inherited disease that affects approximately 0.3% to 1% of the black population in the (A) United States. Affected patients are homozygous for hemoglobin S such that 70% to 98% of the hemoglobin found in their RBCs is of the unstable S type, resulting in severe hemolytic anemia. Factors that favor the formation of sickle cells include arterial hypoxemia, acidosis, dehydration, and reductions in body temperature. Inhaled nitric oxide and other new investigational drugs may help reduce the sickling process and may even unsickle cells (Fleisher: Anesthesia and Uncommon Diseases, ed 5, pp 362-363; Hines: Stoeling's Anesthesia and Co-Existing Disease, ed 5, pp 411-412).
question
528. Which of the following factors is the greatest predictor of sleep apneas in an adult? A. Neck circumference B. Edentulousness C. Weight D. Body mass index E. Micrognathia
answer
528. (C)?? Although many books suggest that obesity is the most common cause of OSA, more recent data suggests (C) that a large neck circumference (>44 cm) reflects pharyngeal fat deposition and is more strongly correlated with OSA than obesity (BMI >30). Other risk factors include male gender, middle age, evening alcohol consumption or sleep-inducing medications (Hines: Stoelting's Anesthesia and Co-Existing Disease, ed 5, pp 299-301; Miller: Anesthesia, ed 6, p 1030; Stoelting: Basics of Anesthesia, ed 5, pp 411-412).
question
529. Greatest number of malpractice claims made against anesthesiologists (according to the ASA closed claims task force) is associated with which adverse outcome? A. Airway trauma B. Brain damage C. Nerve damage D. Death E. Eye injury
answer
529. (D) The ASA closed claims task force lists the leading causes of malpractice claims against anesthesiologists in (D) the 1990s to be death (22%), followed by nerve damage (21%), and brain damage (10%) (Barash: Clinical Anesthesia, ed 5, p 101).
question
530. Resynchronization therapy A. Is indicated for short QRS complexes B. Synchronizes QRS with breathing C. Requires pacemaker implantation D. Is usually accomplished with biphasic defibrillator E. Is contraindicated in patients with coronary artery disease
answer
530. (C) Cardiac resynchronization therapy (CRT) is used in patients with heart failure (EF ; 35%) and ventricular (C) conductive delay (prolonged QRS complex usually is 120-150 msec). The conduction delay creates a mechanical dyssynchrony and worsens the heart failure. Cardiac resynchronization therapy requires biventricular pacing with one lead in the coronary sinus to activate the LV. Cardiac resynchronization therapy has nothing to do with breathing. Although CRT has nothing to do with an ICD, many patients may require both as the typical scenario is a patient with poor LV function is also at risk for sudden death. Most of these patients also have underlying CAD (Hines: Stoelting's Anesthesia and Co-Existing Disease, ed 5, p 112; Miller: Anesthesia, ed 6, pp 1418-1419, 1427-1428).
question
531. The underlying feature in patients with syndrome X is A. Hypertension B. Coronary artery disease C. Hypoglycemia D. Insulin resistance E. Morbid obesity
answer
531. (D) Patients with syndrome X (also called metabolic syndrome X) have insulin resistance that leads to elevated (D) levels of insulin and the metabolic changes that occur with elevated insulin levels, except that hypoglycemia does not develop. Associated with it are low levels of high-density lipoproteins, hypertension, and increased plasminogen activator inhibitor-1 levels, which are associated with coronary artery disease. Many of these patients are obese (Miller: Anesthesia, ed 6, p 1777).
question
532. A 65-year-old hospitalized patient is being treated for pain from pancreatic cancer and is well-controlled on 30 mg IV morphine per day. What is the equivalent total oral daily dosage of morphine in this patient for discharge planning? A. 10 mg B. 30 mg C. 90 mg D. 120 mg E. 150 mg
answer
532. (C) The parenteral to oral conversion for morphine sulfate is 1:3, thus 30 mg morphine parenterally would be (C) similar to 30 mg × 3 = 90 mg of morphine orally. The parenteral to oral conversion for methadone is 1:2 (Hardman: Goodman ; Gilman's The Pharmacological Basis of Therapeutics, ed 11, p 606).
question
533. The effect of inhaled anesthetics is determined by A. The partial pressure of the anesthetic B. The percent of the anesthetic in the inspired gas C. Partial pressure for N2O and percent of inspired gas for volatiles D. Partial pressure for N2O and desflurane and percent inspired gas for all others E. Partial pressure only for agents with vapor pressure less than or equal to 250
answer
533. (A) It is the partial pressure of the inhalation anesthetic in the brain (Pbr) that determines activity (Stoelting: (A) Basics of Anesthesia, ed 5, pp 82-83).
question
534. Hazards of O2 administration include A. Retinopathy of prematurity B. Retention of CO2 C. Adsorption atelectasis D. Bronchopulmonary dysplasia E. All of the above
answer
534. Retinopathy of prematurity (retrolental fibroplasia) is a hazard associated with O2 administration to neonates (E) up to 44 weeks (gestational age + life age). It is especially a hazard in the extremely premature (birth weight less than 1000 g and gestational age less than 28 weeks). Bronchopulmonary dysplasia is a chronic lung disorder that afflicts infants who required mechanical ventilation at birth to treat respiratory distress syndrome. CO2 retention is a hazard in patients with chronic obstructive lung disease. Adsorption atelectasis is a potential hazard of oxygen administration in any patient receiving oxygen concentrations greater than 50%. It results from rapid uptake of oxygen into the circulation greater than the delivery of oxygen by ventilation. Normally, the presence of nitrogen serves as an internal splint, protecting the alveoli from collapse. Prolonged high concentration of oxygen can damage "normal lungs" if given for prolonged periods of time and may lead from mild irritation to tracheobronchitis to pulmonary interstitial edema to pulmonary fibrosis (Miller: Anesthesia, ed 6, pp 711, 716, 717, 2853; Morgan: Clinical Anesthesiology, ed 4, pp 1028-1029).
question
535. Which of the following nerves is NOT derived from a cranial nerve? A. Great auricular B. Infraorbital C. Supratrochlear D. Supraorbital E. Mental
answer
535. (A) All of the nerves listed in this question are derived from the fifth cranial nerve (trigeminal nerve) except the (A) great auricular nerve. The ophthalmic nerve (V1 branch of trigeminal nerve) gives rise to the supratrochlear, infratrochlear, and supraorbital nerves. The infraorbital nerve is a branch of V2 (maxillary branch of the trigeminal nerve). The mental nerve is a branch of V3 (mandibular nerve). The great auricular nerve arises from branches of C2 and C3 spinal nerves and innervates the skin of the outer ear, the mastoid process and the parotid gland (Miller: Anesthesia, ed 6, pp 1706-1707, 2548).
question
536. A 45-year-old female is experiencing progressive mental deterioration over a 6 hour period, 5 days after emergency evacuation of a large subarachnoid hemorrhage and clipping of a middle cerebral artery aneurysm. The most likely cause for deterioration is: A. Cerebral edema B. Hyponatremia C. Recurrent cerebral hemorrhage D. Vasospasm E. Improper placement of the aneurysm clip
answer
536. (D) Cerebral vasospasm is often associated in patients who have suffered a subarachnoid bleed. Angiographic (D) evidence of vasospasm can be noted in up to 70% of patients, however, clinical vasospasm with detectable ischemia (e.g., mental confusion, lethargy, focal motor and speech impairments) is detected in about 30% of patients. When clinical vasospasm develops, it usually occurs between 4 and 12 days after the bleed. Although it may resolve spontaneously, it may also progress to coma and death within a few hours or days. Rebleeding tends to occur earlier, i.e., within 24 hours (Barash: Clinical Anesthesia, ed 5, p 778).
question
537. Which of the following treatments should not be used in the management of thyrotoxicosis? A. Aspirin B. Cold crystalloid C. Cholestyramine D. Dexamethasone E. Esmolol
answer
537. (A) Thyroid storm (thyrotoxicosis) is a medical emergency with a 10% to 75% mortality rate. It usually occurs in (A) poorly controlled or undiagnosed Graves' disease patients. It may develop intraoperatively but more likely develops 6 to 18 hours after surgery. Symptoms include mental status changes, tachycardia, fever, cardiac arrhythmias. Treatment includes large doses of dexamethasone, which inhibits the synthesis, release and conversion of T4 to T3. Propylthiouracil inhibits thyroid hormone production. Sodium iodine is used to inhibit release of thyroid hormone from the thyroid gland. Propranolol helps to control the tachycardiac response and may inhibit conversion of T4 in the periphery. Acetaminophen is used to help control the temperature. Aspirin is contraindicated since it displaces thyroid hormones from thyroglobin and could aggravate the disease (Morgan: Clinical Anesthesiology, ed 4, pp 1016, 1017; Stoelting: Basics of Anesthesia, ed 5, p 443).
question
538. The most common adverse cardiac event in the pediatric population is A. Hypotension B. Bradycardia C. Premature ventricular contraction (PVC) salvos D. Bigeminy E. Tachycardia
answer
538. (B) The most common adverse cardiac event in pediatric population is bradycardia. An outcome study from the (B.) Medical College of Virginia examined the incidence of bradycardia in nearly 8000 children younger than 4 years old. The most common causes of bradycardia are cardiac disease or surgery and inhalation anesthesia, followed by hypoxemia. Of those children who had bradycardia, hypotension occurred in 30%, asystole or ventricular fibrillation in 10%, and death in 8%. Tachycardia, which is common, is not an adverse event (Motoyama: Smith's Anesthesia for Infants and Children, ed 7, p 1161).
question
539. Each of the following is a predictor of difficulty with mask ventilation EXCEPT A. Presence of beard B. BMI greater than 26 C. Presence of teeth D. Age greater than 55 E. History of snoring
answer
539. (C) Mask ventilation, one of the most basic anesthesia techniques, can be challenging in some patients. Patients (C) who are prone to airway obstruction can be more difficult due to extra airway tissue (i.e. obese patients with a BMI > 26), patients without teeth (i.e., tongue is closer to the roof of the mouth and face conformity may not fit the mask well), patients who snore (i.e., already have reason for airway obstruction). Mask ventilation can also be more difficult in patients who have a beard (i.e., harder to get a good mask seal), patients whose age is greater than 55, patients with facial tumors and patients with facial trauma. Use of an oral airway may be needed in many of these patients (Stoelting: Basics of Anesthesia, ed 5, p 215; Miller: Anesthesia, ed 6, pp 1623-1625).
question
540. In a patient with compartment syndrome, which of the following signs would be the last to appear? A. Pulselessness B. Pain C. Swelling D. Paralysis E. Paresthesia
answer
540. (A) Whenever perfusion to an extremity is inadequate (e.g., trauma or poor perfusion), hypoxic edema develops, (A) producing swelling. When this occurs in a compartment, tissue pressures rise, decreasing capillary perfusion. Symptoms of compartment syndrome include extreme pain unrelieved by analgesics, paresthesias, paralysis and pallor. Extensive rhabdomyolysis may develop as well as permanent nerve and muscle injury in the compartment. Because the problem is at the tissue level, pulses and capillary refill may still be present. Treatment includes fasciotomy to relieve the elevated pressure (Barash: Clinical Anesthesia, ed 5, pp 652- 653, 1279; Miller: Anesthesia, ed 6, pp 2478-2479).
question
541. Select the TRUE statement regarding the dose and duration, respectively, of local anesthetics for spinals in infants compared with adults A. Greater dose and longer duration B. Greater dose and shorter duration C. Greater dose and duration is the same D. Smaller dose and longer duration E. Smaller dose and shorter duration
answer
541. (B) The amount and distribution of CSF is different in neonates compared with adults. The neonate has about 4 mL/kg of CSF compared to the adult's 2 mL/kg. In addition, almost half of the neonate's CSF is in the spinal subarachnoid space, compared with about a quarter of the adult's CSF in the spinal subarachnoid space. These factors help explain why the dose is greater in neonates and infants and of shorter duration compared to adults (Motoyama: Smith's Anesthesia for Infants and Children, ed 7, p 467).
question
542. A No. 6 endotracheal tube indicates which size? A. 6 mm internal diameter B. 6 mm external diameter C. 6 mm external circumference D. 6 mm internal circumference E. 6 cm external radius
answer
542. (A) Endotracheal tube sizes are measured according to the internal diameter (ID). They are available in 0.5 mm ID increments (Stoelting: Basics of Anesthesia, ed 5, p 218).
question
543. If a patient were to become trapped in the MRI scanner by a metal object and the engineers decided to quench the magnet, the greatest hazard to the patient would be A. Heat B. Cold C. Fire D. Explosion E. Noise
answer
543. (B) Magnetic resonance imaging (MRI) scanners have superconducting electrical currents that produce large magnetic fields (up to 6 m) and are always "on". The presence of any ferromagnetic objects in the room may cause a missile-type injury when the objects are strongly attracted to the scanner. If a patient is pinned into the scanner by a magnetic object that flew into the scanner, the MRI technicians may have to turn off the superconducting magnet. During magnetic shutdown (quench) the scanner will become extremely cold (Stoelting: Basics of Anesthesia, ed 5, pp 553-554).
question
544. A 25-year-old black male is brought to the emergency room unconscious. Supplemental oxygen is administered and a pulse oximeter is placed on his finger and a reading of 98% is recorded. Arterial gas sampling at the same time shows PaO2 of 190 mm Hg, pH 7.2 and O2 saturation of 90%. Presence of which of the following could explain the discrepancies between these two readings? A. Methemoglobin (Hb Met) B. Sickle cell hemoglobin C. Carboxyhemoglobin (HbCO) D. Hemoglobin shifted to right E. Pulse oximeter error
answer
544. (C) Carbon monoxide is a colorless, odorless gas which binds to hemoglobin with an affinity over 200 times stronger than oxygen. Inhalation of CO is a major cause of morbidity and mortality in the United States. A dual wave (660 nm and 940 nm) pulse oximeter is incapable of distinguishing CO hemoglobin from oxyhemoglobin, but the distinction is easily made in the clinical laboratory with a co-oximeter. Significant quantities of methemoglobin would result in a saturation of 85% of the pulse oximeter. The slight right shift from a mild acidemia would be insufficient to account for 90% saturation in the face of a PaO2 of 190. Furthermore, the pulse oximeter reading would be nearly the same as the co-oximeter value (Miller: Anesthesia, ed 6, pp 1450- 1451; Hines: Stoelting's Anesthesia and Co-Existing Disease, ed 5, p 552).
question
545. During surgery for correction of scoliosis, somatosensory evoked potential (SSEP) monitoring is employed. An increase in SSEP latency and decrease in amplitude could be explained by each of the following EXCEPT A. Anterior spinal artery syndrome B. Ischemia of posterior tibial nerve C. Hypotension D. 2 MAC isoflurane anesthesia E. Propofol infusion (200 µg/kg/min)
answer
545. (A) The pathway for SSEP monitoring of the lower extremity starts with a stimulus of the posterior tibial nerve, (A) which generates an impulse that passes through the dorsal root ganglion into the dorsal (posterior) columns and then to the dorsal column nuclei. Second order nerves carry the impulse across the midline to the thalamus and the impulse travels over third order nerves to the sensory cortex of the brain. Electrodes in the scalp record the electrical activity in the brain. Severe hypotension or ischemia in any portion of the pathway along which the induced signal is conducted can result in a reduced evoked potential amplitude or increased latency. Volatile anesthetic administration in MAC values greater than 0.5 to 0.75 can produce a similar effect. Barbiturates, benzodiazepines, propofol and other sedative drugs can likewise interfere with SSEP monitoring. Anterior spinal artery syndrome affects the anterior (motor) portion of the spinal cord and does not interfere with SSEP monitoring (Stoelting: Basics of Anesthesia, ed 5, pp 313-314).
question
546. In which of the following conditions would the response to atropine be most pronounced? A. Diabetic autonomic neuropathy B. Brain death C. Status post heart transplant D. High (C8) spinal anesthesia E. A patient with chronic atrial fibrillation, complete heart block with VVI pacemaker
answer
546. (D) Diabetic autonomic neuropathy can affect the autonomic nervous system to such an extent that atropine and propranolol would have little effect (because there would be nothing to block). After heart transplantation, the new heart (donor heart) is denervated and will not respond to autonomic nervous system blocking drugs. Brain death by definition is associated with absence of autonomic function. A high spinal would be associated with total sympathectomy and propranolol would have no effect on heart rate, but the vagus nerve would be unaffected. Atropine would have no effect on a patient with atrial fibrillation and complete heart block (Hines: Stoelting's Anesthesia and Co-Existing Disease, ed 5, pp 22, 213, 375; Stoelting: Basics of Anesthesia, ed 5, pp 260, 438-439).
question
555. Decreased FEV1/FVC ratio 556. Decreased total pulmonary compliance 557. Increased total lung capacity 558. Decreased FRC 559. Decreased FEV1, normal FEV1/FVC ratio 560. Increased lung compliance due to loss of elastic recoil of the lung A. Pulmonary emphysema B. Chronic bronchitis C. Restrictive pulmonary disease D. Pulmonary emphysema and chronic bronchitis E. Pulmonary emphysema and restrictive pulmonary disease
answer
555. 556. (C) 557. (D) 558. (C) 559. (C) 560. (A) (D) Pulmonary function tests can be used to classify patients with chronic pulmonary disease into those with obstructive airway diseases (e.g., asthma, pulmonary emphysema, and chronic bronchitis) and those with restrictive pulmonary diseases (e.g., pulmonary fibrosis, scoliosis). The forced expiratory volume in 1 second or FEV1 is the amount of air expired in 1 second and commonly is expressed as a percentage of the forced vital capacity, or FEV1/FVC. The normal FEV1/FVC is 75% to 80%. In the presence of obstructive airway disease, FEV1 of less than 70% has mild obstruction, less than 60% has moderate obstruction and less than 50% has severe obstruction. Patients with obstructive lung disease also have a normal (asthma) or increase in (bronchitis, emphysema) total lung capacity and functional residual capacity or FRC. In the presence of restrictive pulmonary disease, FEV1 is reduced but because FVC is also reduced, the FEV1/FVC is normal. Patients with restrictive disease have a total lung capacity (TLC), FRC, and total pulmonary compliance that are reduced. In patients with pulmonary emphysema, lung compliance is increased because the elastic recoil of the lungs is decreased (Miller: Anesthesia, ed 6, pp 999-1010; Stoelting: Basics of Anesthesia, ed 5, pp 406-411).
question
561. Weakness of all muscles below the knee 562. Foot drop; loss of dorsal extension of the toes 563. Weakness of the muscles that extend the knee 564. Inability to adduct the leg; diminished sensation over the medial side of the thigh 565. Most commonly caused by placement of patient into the lithotomy position 566. Numbness over the lateral aspect of the thigh A. Sciatic nerve injury B. Common peroneal nerve injury C. Femoral nerve injury D. Obturator nerve injury E. Lateral femoral cutaneous nerve injury
answer
561. 562. (B) 563. (C) 564. (D) 565. (B) 566. (E) (A) In many cases of peripheral nerve injuries the mechanism of injury is largely unknown; however, stretching or compression of the nerves can lead to nerve ischemia and damage. In the lithotomy position, hyperflexion of the hips and/or extension of the knees can aggravate stretch of the sciatic nerve. Also in the lithotomy position, compression of the common peroneal nerve between the head of the fibula and the metal supporting frame can occur. The common peroneal nerve is the most common nerve injured in the lithotomy position. Proper padding between the metal leg braces and positioning of the legs will limit the occurrence of these injuries. The sciatic nerve provides motor function for all the skeletal muscles below the knees and sensory innervation for the lateral half of the leg and most of the foot. Injury to the common peroneal nerve, a branch of the sciatic nerve, causes a foot drop from the impaired ankle dorsiflexion and the loss of foot eversion and toe extension. Injury to the femoral or obturator nerves can occur with excessive retraction during lower abdominal surgery. The obturator nerve can also be injured during a difficult forceps vaginal delivery or by excessive flexion of the thigh to the groin. Injury to the femoral nerve will manifest as decreased extension of the knee (paresis of the quadriceps femoris muscle) and numbness over the anterior aspect of the thigh and medial/anteromedial side of the leg. The inability to adduct the leg and thigh as well as numbness over the medial side of the thigh are clinical manifestations consistent with damage to the obturator nerve. Excessive flexion of the hip on the abdomen can cause a neuropathy of the lateral femoral cutaneous nerve (sensory only) resulting in numbness of the lateral aspect of the thigh (Miller: Anesthesia, ed 6, pp 1155-1159; Stoelting: Basics of Anesthesia, ed 5, pp 292, 294, 301).
question
547. Skin lesions all appear at the same stage and at the same time 548. Ciprofloxacin for 60 days is prophylaxis for exposed patients 549. Not contagious 550. Treatment may include streptomycin, gentamicin or tetracycline 551. Treatment includes trivalent equine antitoxin 552. Three primary types: cutaneous, gastrointestinal and inhalation 553. Vaccine may prevent or greatly attenuate symptoms if given within 4 days of exposure 554. Hemorrhagic fever A. Smallpox B. Anthrax C. Plague D. Botulism E. Ebola virus
answer
547. 548. (B) 549. (D) 550. (C) 551. (D) 552. (B) (A) 553. 554. (E) (A) There are three categories of biological weapons (A,B,C). All of the diseases in this question are in the highly contagious Category A agents. Smallpox is caused by a virus (Variola major) and in 1980 was declared extinct by the World Health Organization. The incubation period was 7 to 14 days and patients with the disease presented with malaise, headache, fever. Two to 4 days later a characteristic rash develops where all lesions are at the same stage (papules, vesicles, pustules and scabs). Exposed patients and health care workers who received a vaccination within 4 days of exposure had greatly attenuated symptoms. Unvaccinated patients who were untreated had a mortality rate of greater than 30%. Patients who previously had been vaccinated had a lower mortality rate. Treatment includes the drug cidofovir. Anthrax is caused by an aerobic Gram-positive spore forming bacillus (Bacillus anthracis) and has three primary forms (cutaneous, gastrointestinal, inhalational). Weaponized anthrax is mainly an inhalational disease. Inhalational anthrax symptoms occur within 1 to 7 days of exposure and initially looks like viral flu (fever, chills, myalgia, and a non-productive cough). Later on, the patient's mediastinal lymph nodes, where the spores germinate, enlarge, producing a widened mediastinum that can be seen on a chest x-ray film. Treatment is primarily with ciprofloxacin, prophylaxis to exposed personnel includes 60 days of ciprofloxacin. Mortality rate for inhaled anthrax is greater than 80%. Plague is caused by a Gram-negative coccobacillus (Yersinia pestis) and has two forms, bubonic and pneumonic. With the more common bubonic plague there is painful swelling of the lymph nodes (buboes) that can grow to 5 to 10 cm in diameter. The patients develop cyanosis, shock and gangrene in peripheral tissues (black death). If the lungs become infected, pneumonic plague develops, which if untreated has 100% mortality. Treatment is primarily with streptomycin, although gentamicin, tetracycline and chloramphenicol have been used. Botulism is caused by the toxin from Clostridium botulinum. Because this disease is due to a neurotoxin, it is not contagious. The neurotoxin affects cholinergic neurons and prevents the release of acetylcholine. Symptoms typically develop within 12 to 36 hours of exposure and include acute flaccid paralysis, decrease salivation, ileus and urinary retention. There are no sensory deficits. With appropriate supportive care and trivalent equine antitoxin, the mortality rate is less than 5%. Without the use of antitoxin, patients may take 2 to 8 weeks to recover. Mortality rate is 5% to 10%. There are more than 18 hemorrhagic fever viruses including the Ebola virus. The incubation period is 2 to 21 days and the patients present with fever, myalgias, headaches, thrombocytopenia and hemorrhagic complications (petechiae, ecchymosis). Untreated, the mortality rate for Ebola is 90%. Treatment includes the drug ribavirin (Barash: Clinical Anesthesia, ed 5, pp 1529-1533; Miller: Anesthesia, ed 6, pp 2516-2523; Stoelting: Basics of Anesthesia, ed 5, pp 621-626).
1 of

Unlock all answers in this set

Unlock answers
question
418. A 78-year-old patient with a history of hypertension and adult-onset diabetes for which she takes chlorpropamide (Diabinese) is admitted for elective cholecystectomy. On the day of admission, blood glucose is noted to be 270 mg/dL, and the patient is treated with 15 units of regular insulin subcutaneously (SQ) in addition to her regular dose of chlorpropamide. Twenty-four hours later after overnight fasting, the patient is brought to the operating room (OR) without her daily dose of chlorpropamide and is anesthetized. A serum glucose is measured and found to be 35 mg/dL. The most likely explanation for this is A. Insulin B. Chlorpropamide C. Hypovolemia D. Effect of general anesthesia E. It is a normal finding in fasting patients
answer
418. (B) Patients with insulin-dependent diabetes and noninsulin-dependent diabetes require special consideration when presenting for surgery. Geriatric age patients come to the OR in the fasting state and without having taken their morning dose of their oral diabetic agent. Chlorpropamide is the longest-acting sulfonylurea and has a duration of action up to 72 hours. Accordingly, it is prudent to measure serum glucose before inducing anesthesia and periodically during the course of the anesthetic and surgery. Regular insulin has a peak effect 2 to 3 hours after SQ administration and a duration of action approximately 6 to 8 hours and would therefore not cause a serum glucose of 35 mg/dL 24 hours after it was administered (Stoelting: Pharmacology and Physiology in Anesthetic Practice, ed 4, pp 479, 483-484).
question
419. Select the true statement. A. Dibucaine is an ester-type local anesthetic B. A dibucaine number of 20 is normal C. The dibucaine number represents the quantity of normal pseudocholinesterase D. Neuromuscular blockade with succinylcholine would last several hours in a patient with a dibucaine number of 80 E. None of the above
answer
419. (E) Dibucaine is an amide-type local anesthetic that inhibits normal pseudocholinesterase by approximately 80%. In patients who are heterozygous for atypical pseudocholinesterase, enzyme activity is inhibited by 40% to 60%. In patients who are homozygous for atypical pseudocholinesterase, enzyme activity is inhibited by only 20%. The dibucaine number is a qualitative assessment of pseudocholinesterase. Quantitative as well as qualitative determination of enzyme activity should be carried out in any patient who is suspected of having a pseudocholinesterase abnormality (Stoelting: Basics of Anesthesia, ed 5, p 140).
question
420. A 56-year-old patient with a history of liver disease and osteomyelitis is anesthetized for tibial débridement. After induction and intubation, the wound is inspected and débrided with a total blood loss of 300 mL. The patient is transported intubated to the recovery room, at which time the systolic blood pressure falls to 50 mm Hg. Heart rate is 120 beats/min, arterial blood gases (ABGs) are PaO2 103, PaCO2 45, pH 7.3, with 97% O2 saturation with 100% FIO2. Mixed venous blood gases are PvO2 60, PaCO2 50, pH 7.25, with 90% O2 saturation. Which of the following diagnoses is most consistent with this clinical picture? A. Myocardial infarction B. Congestive heart failure C. Cardiac tamponade D. Sepsis with acute respiratory distress syndrome E. Hypovolemia
answer
420. (D) All hypotension can be broadly broken down into two main categories: decreased cardiac output and decreased systemic vascular resistance. Flow or cardiac output can be further subdivided into problems related to decreased heart rate (i.e., bradycardia versus problems related to decreases in stroke volume). Normal PO2 in mixed venous blood is 40 mm Hg. Increased mixed venous arterial oxygen levels can be due to many conditions including high cardiac output, sepsis, left-to-right cardiac shunts, impaired peripheral uptake (e.g., cyanide), decreased oxygen consumption (e.g., hypothermia) as well as sampling error. The other choices in this question all represent conditions whereby cardiac output is diminished and consequently would not be consistent with the data given in the question (Morgan: Clinical Anesthesiology, ed 4, pp 560- 561).
question
421. Normal tracheal capillary pressure is A. 5 to 15 mm Hg B. 15 to 25 mm Hg C. 25 to 35 mm Hg D. 35 to 45 mm Hg E. 45 to 55 mm Hg
answer
421. (C) Tracheal capillary arteriolar pressure (25-35 mm Hg) is important to keep in mind in patients who are intubated with cuffed endotracheal tubes. If the endotracheal tube cuff exerts a pressure greater than capillary arteriolar pressure, tissue ischemia may result. Persistent ischemia may lead to destruction of tracheal rings and tracheomalacia. Endotracheal tubes with low-pressure cuffs are recommended in patients who are to be intubated for periods longer than 48 hours because this will minimize the chances for development of tissue ischemia (Miller: Anesthesia, ed 6, p 1630).
question
422. How many hours should elapse before performing a single-shot spinal anesthetic in a patient who is receiving 1 mg/kg enoxaparin twice a day for the treatment of a deep vein thrombosis? A. 2 hours B. 6 hours C. 12 hours D. 18 hours E. 24 hours
answer
422. (E) Enoxaparin, dalteparin, and ardeparin are low-molecular-weight heparins (LMWH). Because of the possibility of spinal and epidural hematoma in the anticoagulated patient with neuraxial blockade, caution is advised. The plasma half-life of LMWH is two to four times longer than standard heparin. These drugs are commonly used for prophylaxis for deep vein thrombosis. These drugs are also used at high doses for treatment of deep vein thrombosis and (off label) as "bridge therapy" for patients chronically anticoagulated with Coumadin. In these patients who are being prepared for surgery, Coumadin is discontinued and LMWH started. With high-dose enoxaparin administration (1 mg/kg twice daily) it is recommended to wait at least 24 hours before administration of a single-shot spinal anesthetic (Barash: Clinical Anesthesia, ed 5, p 713; Miller: Anesthesia, ed 6, pp 1677, 2742-2743; Second Consensus Conference on Neuraxial Anesthesia and Anticoagulation, April 25-28, 2002 www.asra.com/consensus-statements/2.html ).
question
423. Which of the following peripheral nerves is most likely to become injured in patients who are under general anesthesia? A. Ulnar nerve B. Median nerve C. Radial nerve D. Common peroneal nerve E. Sciatic and peroneal nerve
answer
423. (A) The principal mechanism of peripheral nerve injury is ischemia caused by stretching or compression of the nerves. Anesthetized patients are at increased risk for peripheral nerve injuries because they are unconscious and unable to complain about uncomfortable positions that an awake patient would not tolerate and because of reduced muscle tone that facilitates placement of patients into awkward positions. The ulnar nerve in particular is vulnerable because it passes around the posterior aspect of the medial epicondyle of the humerus. The ulnar nerve may become compressed between the medial epicondyle and the sharp edge of the operating table, leading to ischemia and possible nerve injury which may be transient or permanent (Stoelting: Basics of Anesthesia, ed 5, pp 299-300).
question
424. Renal failure associated with fluoride toxicity anesthesia most closely resembles A. Papillary necrosis B. Acute tubule necrosis C. Hepatorenal syndrome D. Central diabetes insipidus E. Nephrogenic diabetes insipidus
answer
424. (E) Methoxyflurane is extensively metabolized resulting in the liberation of the fluoride anion. When the inorganic fluoride levels are less than 50 µmol/L no evidence of renal injury is seen. With levels of 50 to 80 µmol/L (which develops with 2.5-3 MAC hours of methoxyflurane use) moderate injury occurs and with levels of 80 to 120 µmol/L severe injury develops. Several patients have died when levels were above 120 µmol/L. Fluoride in appropriate concentrations is capable of making the kidney unresponsive to ADH. This condition is known as nephrogenic diabetes insipidus and is the reason why methoxyflurane was withdrawn from clinical practice. Enflurane (now obsolete in the United States) also is capable of yielding free fluoride ions, but at levels much less than with methoxyflurane. Sevoflurane is defluorinated through oxidative metabolism and can produce serum fluoride levels that can peak above 50 µmol/L, although typically levels are commonly around 30 µmol/L. Renal failure has not been clinically seen with sevoflurane use, possibly because the drug is rapidly excreted via the lungs (methoxyflurane is very slowly excreted via the lungs). Very small amounts of free fluoride ions are produced through the metabolism of isoflurane and halothane. Desflurane is very resistant to defluorination and is not associated with nephrotoxicity (Miller: Anesthesia, ed 6, pp 248-251).
question
425. A 45-year-old obese male is in the intensive care unit (ICU) after an elective open lung biopsy. Which of the following would provide the best prophylaxis against deep vein thrombosis in this patient? A. Pneumatic compression boots B. Heparin 5000 units SQ every 8 hours C. Early ambulation D. Dextran 10 mL/kg IV during surgery E. Incentive spirometry
answer
425. (C) The incidence of deep vein thrombosis can be reduced from 30% to less than 10% in patients undergoing thoracoabdominal surgery if low-dose heparin (5000 units) is administered 2 hours before surgery and every 8 to 12 hours thereafter (until the patient is able to walk). Although the reduction in deep vein thrombosis in these patients is clear, it is not certain if this therapy prevents pulmonary embolism or reduces mortality. Aspirin, Coumadin, dextran, and compression boots may be of benefit in specific clinical situations. Early ambulation, however, is the best prophylaxis against deep vein thrombosis. Coughing does nothing to prevent deep vein thromboses (Hines: Stoelting's Anesthesia and Co-Existing Disease, ed 5, pp 155-157)
question
426. A patient with which of the following eye diseases would be at greatest risk for retinal damage from hypotension during surgery? A. Strabismus B. Cataract C. Glaucoma D. Severe myopia E. Open eye injury
answer
426. (C) Blood flow to the retina can be decreased by either a decrease in mean arterial pressure or an increase in intraocular pressure. Decreased blood flow and stasis are more likely in patients with glaucoma because of their elevated intraocular pressure. During periods of prolonged hypotension, the incidence of retinal artery thrombosis increases in these patients (Hines: Stoelting's Anesthesia and Co-Existing Disease, ed 5, p 235; Stoelting: Basics of Anesthesia, ed 5, pp 463-469).
question
427. Naltrexone is A. A narcotic with local anesthetic properties B. An opioid agonist-antagonist similar to nalbuphine C. A pure opioid antagonist with a shorter duration of action than naloxone D. An opioid antagonist used for treatment of previously detoxified heroin addicts E. A synthetic opioid derived from oxymorphone
answer
427. (D) Naloxone (Narcan) is a competitive inhibitor at all opioid receptors but has the greatest affinity for µ-receptors. Its duration of action is relatively short (elimination half-life of about 1 hour). For this reason, one must be vigilant for the possibility of renarcotization when reversing long acting narcotics. Naltrexone (ReVia) is the Ncyclopropylmethyl derivative of oxymorphone with a long elimination half-life of 8 to 12 hours. It is currently only available as an oral preparation and is used to block the euphoric effects of injected heroin in addicts who have been previously detoxified. Nalmefene (Revex) is another opioid antagonist that can be administered orally or parenterally and has an extremely long duration of action (elimination terminal half life of 8.5 hours) (Longnecker: Principles and Practice of Anesthe
question
428. Which of the following mechanisms is most frequently responsible for hypoxia in the recovery room? A. Ventilation/perfusion mismatch B. Hypoventilation C. Hypoxic gas mixture D. Intracardiac shunt E. Abnormal gas diffusion
answer
(A) In the recovery room, the most common cause of postoperative hypoxemia is a uneven ventilation/perfusion distribution caused by loss of lung volume resulting from small airway collapse and atelectasis. Risk factors for ventilation/perfusion mismatch in the postoperative period include old age, obstructive lung disease, obesity, increased intraabdominal pressure, and immobility. Supplemental oxygen should be administered to keep the PaO2 in the 80 to 100 mm Hg range, which is associated with a 95% saturation of hemoglobin. Other measures can be taken to restore lung volume, which include recovering obese patients in the sitting position, coughing, and deep breathing (Barash: Clinical Anesthesia, ed 5, pp 1391-1395).
question
429. Hypoparathyroidism secondary to the inadvertent surgical resection of the parathyroid glands during total thyroidectomy typically results in symptoms of hypocalcemia how many hours postoperatively? A. 1 to 2 hours B. 3 to 12 hours C. 12 to 24 hours D. 24 to 72 hours E. Greater than 72 hours
answer
429. (D) Airway obstruction after total thyroidectomy may be caused by a postoperative hematoma, compression of the trachea, tracheomalacia, bilateral recurrent laryngeal nerve damage, or hypocalcemia resulting from inadvertent removal of the parathyroid glands. Although the airway symptoms of hypocalcemia can develop as early as 1 to 3 hours after surgery, they typically do not develop until 24 to 72 hours postoperatively. Because the laryngeal muscles are particularly sensitive to hypocalcemia, early symptoms may include inspiratory stridor, labored breathing, and eventual laryngospasm. Therapy consists of IV administration of calcium gluconate or calcium chloride (Stoelting: Basics of Anesthesia, ed 5, pp 443-444).
question
430. Damage to which nerve may lead to wrist drop? A. Radial B. Axillary C. Median D. Musculocutaneous E. Ulnar
answer
430. (A) Damage to the radial nerve is manifested by weakness in abduction of the thumb, inability to extend the metacarpophalangeal joints, wrist drop, and numbness in the webbed space between the thumb and index fingers. The radial nerve passes around the humerus between the middle and lower portions in the spiral groove posteriorly. As it wraps around the bone, the radial nerve can become compressed between it and the OR table, resulting in nerve injury (Barash: Clinical Anesthesia, ed 5, p 650).
question
431. The most common cause of bronchiectasis is A. Cigarette smoking B. Air pollution C. ?1-Antitrypsin deficiency D. Recurrent bronchial infections E. Squamous cell carcinoma
answer
431. (D) Bronchiectasis is one of several obstructive lung diseases characterized by a diminished FEV1 when pulmonary function is evaluated. It is characterized by permanently dilated bronchi that frequently contain purulent secretions. The affected bronchi are often highly vascularized, giving rise to the possibility of hemoptysis. Collateral circulation through the intercostal and bronchial arteries is also possible in these patients. If these vessels connect with the pulmonary circulation, pulmonary hypertension and eventual cor pulmonale are possible sequelae. Any patient with chronic bronchial infections may develop bronchiectasis (Hines: Stoelting's Anesthesia and Co-Existing Disease, ed 5, p 175).
question
432. A 6-year-old child is transported to the recovery room after a tonsillectomy. The patient was anesthetized with isoflurane, fentanyl, and N2O. Twenty minutes before emergence and tracheal extubation, droperidol was administered. The anesthesiologist is called to the recovery room because the patient is "making strange eye movements." The patient's eyes are rolled back into his head, and his neck is twisted and rigid. The most appropriate drug for treatment of these symptoms is A. Dantrolene B. Thiopental C. Glycopyrrolate D. Chlorpromazine E. Diphenhydramine
answer
432. (E) Drugs that block dopamine receptors may cause acute dystonic reactions in some patients. The incidence with droperidol is about 1%. Treatment is the administration of a drug that crosses the blood brain-barrier with anticholinergic properties such as diphenhydramine or benztropine. Although glycopyrrolate is an anticholinergic drug, it would not be useful in this setting because it does not cross the blood-brain barrier (Hines: Stoelting's Anesthesia and Co-Existing Disease, ed 5, p 644; Stoelting: Pharmacology and Physiology in Anesthetic Practice, ed 4, p 414).
question
433. A 32-year-old military officer is unable to oppose the left thumb and left little finger after an 8-hour exploratory laparotomy under general anesthesia. He had an IV induction through a peripheral IV and had a second IV placed in the antecubital fossa after he was asleep. Damage to which of the following nerves would most likely account for this deficit? A. Radial B. Ulnar C. Median D. Musculocutaneous E. Median antebrachial cutaneous nerve
answer
433. (C) The median nerve is most frequently injured at the antecubital fossa by extravasation of IV drugs (e.g., thiopental) that are toxic to neural tissue, or by direct injury caused by the needle during attempts to cannulate the medial cubital or basilic veins. The median nerve provides sensory innervation to the palmar surface of the lateral three and one-half fingers and adjacent palm, and motor function to the abductor pollicis brevis, flexor pollicis brevis, and opponens pollicis muscles (Stoelting: Basics of Anesthesia, ed 5, p 301).
question
434. Pheochromocytoma would be most likely to coexist with which of the following? A. Insulinoma B. Pituitary adenoma C. Primary hyperaldosteronism (Conn's syndrome) D. Medullary carcinoma of the thyroid E. Carcinoid tumor
answer
434. (D) Pheochromocytoma is an endocrine tumor (with release of catecholamines) in which 90% of patients are hypertensive, 90% of the tumors originate in one adrenal medulla, and 90% of all pheochromocytomas are benign. This disease is rare (<0.1% of hypertension in adults), but when it occurs, it is often seen with a triad of diaphoresis, tachycardia, and headache in patients with hypertension. Other symptoms include palpitations, tremulousness, weight loss, hyperglycemia, hypovolemia, and in some cases dilated cardiomyopathy and congestive heart failure. Death as a result of pheochromocytoma is due to cardiac conditions (e.g., myocardial infarction, congestive heart failure) or an intracranial bleed. In about 5% of cases, pheochromocytomas show an autosomal dominant pattern and may coexist with other endocrine diseases such as medullary carcinoma of the thyroid and hyperparathyroidism. This combination is called multiple endocrine neoplasia or MEN type II or IIA (Sipple's syndrome). MEN type IIB consists of pheochromocytoma, medullary carcinoma of the thyroid, and neuromas of the oral mucosa. The von Hippel-Lindau disease consists of hemangiomas of the nervous system (i.e., retina or cerebellum) and 10% to 25% of these patients also have a pheochromocytoma. The average sized pheochromocytoma contains 100 to 800 mg of norepinephrine (Barash: Clinical Anesthesia, ed 5, p 1142; Hines: Stoelting's Anesthesia and Co-Existing Disease, ed 5, pp 388-393).
question
435. The plasma concentration of which of the following liver enzymes is increased in patients with biliary obstruction? A. Serum glutamic-oxaloacetic transaminase B. Serum glutamic-pyruvic transaminase C. Lactate dehydrogenase D. Alkaline phosphatase E. Alcoholic dehydrogenase
answer
435. (D) Serum glutamic-oxaloacetic transaminase (aspartate aminotransferase), serum glutamic-pyruvic transaminase (alanine aminotransferase), and lactate dehydrogenase all are elevated in patients with liver disease. The serum level of alkaline phosphatase may be a specific indicator of biliary obstruction. Because this enzyme is also produced in the intestines, bone, and placenta, other serum tests must be ordered to differentiate among these potential sources. Concurrent measurement of the serum ?-glutaryl transferase (GGT), leucine aminopeptidase, or 5?-nucleotidase levels can be measured simultaneously with alkaline phosphatase to determine the origin of the latter (Miller: Anesthesia, ed 6, pp 754-755).
question
436. The onset of delirium tremens after abstinence from alcohol usually occurs in A. 8 to 24 hours B. 24 to 48 hours C. 2 to 4 days D. 4 to 7 days E. Greater than 7 days
answer
436. (C) Although early mild symptoms of alcohol withdrawal can be seen within 6 to 8 hours after a substantial drop in the serum alcohol levels, delirium tremens (DTs) which is seen in about 5% of patients, is a life-threatening medical emergency which develops 2 to 4 days after the cessation of alcohol in alcoholics. Symptoms of DT's include hallucinations, combativeness, hyperthermia, tachycardia, hypertension or hypotension, and grand mal seizures. Treatment of severe alcohol withdrawal consists of fluid replacement, electrolyte replacement, and IV vitamin administration with particular attention paid to thiamine. Aggressive administration of benzodiazepines is indicated to prevent seizures (5 to 10 mg of diazepam every 5 minutes until the patient becomes sedated but not unconscious). ?-Blockers are used to suppress overactivity of the sympathetic nervous system, and lidocaine may be effective in the treatment of cardiac dysrhythmias (Hines: Stoelting's Anesthesia and CoExisting Disease, ed 5, p 543).
question
437. A 78-year-old retired coal miner with an intraluminal tracheal tumor is scheduled for tracheal resection. Which of the following is a relative contraindication for tracheal resection? A. Need for postoperative mechanical ventilation for underlying lung disease B. Tumor located at the carina C. Documented liver metastases D. Ischemic heart disease with a history of congestive heart failure E. Tracheal diameter of 0.5 cm at the level of the tumor
answer
437. (A) Operations on the trachea may be indicated in patients who have tracheal tumors or patients who had a previous trauma to the trachea resulting in tracheal stenosis or tracheomalacia. Eighty percent of the operations on the trachea involve segmental resection with primary anastomosis, 10% involve resection with prosthetic reconstruction, and another 10% involve insertion of a T-tube stent. These operations frequently are very complicated and require constant communication between the surgeon and the anesthesiologist. Preoperative pulmonary function tests are indicated in all patients who are to undergo elective tracheal resection. Severe lung disease necessitating postoperative mechanical ventilation is a relative contraindication for tracheal resection because positive airway pressure may cause wound dehiscence (Miller: Anesthesia, ed 6, pp 1912-1913).
question
438. A 78-year-old patient with multiple myeloma is admitted to the ICU for treatment of hypercalcemia. The primary risk associated with anesthetizing patients with hypercalcemia (levels of 14 to 16 mg/dL) is A. Coagulopathy B. Cardiac dysrhythmias C. Hypotension D. Laryngospasm E. Fluid imbalance
answer
438. (B) Hypercalcemia is associated with a number of signs and symptoms, including hypertension, dysrhythmias, shortening of QT interval, kidney stones, seizure, nausea and vomiting, weakness, depression, personality changes, psychosis, and even coma. Generally patients with total serum calcium levels of 12 mg/dL or less do not require any intervention, with the possible exception of rehydration with saline. Higher calcium levels may be associated with clinical symptoms and should be treated before anesthetizing the patient. Caution should be taken with digitalis administration to any patient who is hypercalcemic because some patients may exhibit extreme digitalis sensitivity (Fleisher: Anesthesia and Uncommon Diseases, ed 5, pp 414-415; Longnecker: Principles and Practice of Anesthesiology, ed 2, pp 309-310). NORMAL CALCIUM LEVELS Serum Calcium Serum Ionized Calcium Conventional units (mEq/L) 4.5-5.5 mEq/L 2.1-2.6 mEq/L (mEq/L) Conventional units mg/dL 9.0-11.0 mg/dL 4.25-5.25 mg/dL (mg/dL) SI units (mmol/L) 2.25-2.75 mmol/L 1.05-1.30 mmol/L
question
439. Just before induction of general anesthesia for an 85-year-old demented male with an ischemic bowel, he mentions to you that he forgot to take his green capped eye drops. He states that not taking it daily will result in blindness. The green capped eye drops are A. NaCl drops used to prevent his eye from drying out B. Antibiotic drops C. Steroids D. Used to produce miosis E. Any of a number of eye medication and surgery should be delayed until the eye drops are identified
answer
439. (D) Red-top eye drops cause mydriasis and should be used with caution in patients with closed angle glaucoma. Green top eye drops cause miosis and the pupillary constriction helps keep the drainage route open in patients with glaucoma and helps prevent an acute attack of glaucoma. Clear or white top eye drops do not change pupillary size
question
440. A normal healthy 3-year-old child was involved in a motor vehicle accident. He is coming emergently to the OR. Drug doses need to be calculated, but his weight is not known. What value should be used to estimate the 3-year-old child's weight? A. 8 kg B. 10 kg C. 12 kg D. 14 kg E. 16 kg
answer
440. (D) When reviewing growth curves, the normal 40-week term newborn weighs about 3.5 kg. Children then double their birthweight by 5 months and triple their weight by 1 year. Therefore, the average 1-year-old weighs 10 kg (22 pounds). From the age of 1 to 6 years, children gain about 2 kg per year. Thus, an average 2-year-old weighs 12 kg, 3-year-old weighs 14 kg, 4-year-old weighs 16 kg, 5-year-old weighs 18 kg, and 6-year-old weighs 20 kg. From age 6 to 10 years, children gain about 3 kg per year (Motoyama: Smith's Anesthesia for Infants and Children, ed 7, pp 1203-1205).
question
441. A 62-year-old male undergoes an emergency craniotomy for subdural hematoma. Two years earlier, a VVIpacemaker was placed for third-degree heart block. The patient received vancomycin 1 g IV, before arriving in the OR. General anesthesia is induced with thiopental 300 mg IV and the lungs are hyperventilated to a PaCO2 of 25 mm Hg by mask. Just before tracheal intubation, the patient's heart rate decreases from 70 to 40 beats/min and the pacemaker spikes that were previously present in lead II of the electrocardiogram disappear. The most likely cause of bradycardia in this patient is A. Hypocarbia B. Vancomycin allergy C. Acute increase in intracranial pressure D. A side effect of thiopental E. Pacemaker battery failure
answer
441. (A) Causes for acute pacemaker malfunction in the OR are numerous and include threshold changes, inhibition, generator failure, and lead or electrode dislodgement or breakage. A VVI pacemaker may be inhibited by myopotentials. In this regard, administration of succinylcholine could actually inhibit a VVI pacemaker. Similarly, electrocautery can inhibit a VVI pacemaker through electromagnetic interference. Should this occur, a magnet should be placed over the pacemaker to convert it into a VOO pacemaker, eliminating the possibility of further inhibition. Pacemakers should be evaluated preoperatively to eliminate the possibility of generator failure. Lead breakage or dislodgement is an unlikely cause of pacemaker failure unless the surgeon is working in the vicinity of the electrodes. Acute threshold changes are almost always associated with changes in the serum potassium concentration. In this particular patient, hyperventilation causes a respiratory alkalosis that results in the intracellular shifting of serum potassium. The net result is that the electrical threshold for the pacemaker is raised, preventing ventricular capture (Miller: Anesthesia, ed 6, pp 1426-1427; Thomas: Manual of Cardiac Anesthesia, ed 2, pp 382-383).
question
442. A 28-year-old obese patient has diminished breath sounds bilaterally at the lung bases 18 hours after an emergency appendectomy under general anesthesia. Which of the following maneuvers would be LEAST effective in preventing postoperative pulmonary complications in this patient? A. Coughing B. Voluntary deep breathing C. Performing a forced vital capacity D. Use of incentive spirometry E. Sitting up in bed
answer
442. (C) Therapies aimed at increasing functional residual capacity (FRC) of the lungs are useful in reducing the incidence of post-operative pulmonary complications. Forced expiratory maneuvers may lead to airway closure, which would be of no benefit for this patient (Miller: Anesthesia, ed 6, pp 2713, 2818-2819).
question
443. Below what value of cerebral blood flow (CBF) will signs of cerebral ischemia first begin to appear on the electroencephalogram (EEG)? A. 6 mL/100 g/min B. 15 mL/100 g/min C. 22 mL/100 g/min D. 31 mL/100 g/min E. 40 mL/100 g/min
answer
443. (C) The human brain is able to maintain neuronal function in the face of decreasing CBF below the normal level of 50 mL/100 g/min. Because O2 delivery is directly related to CBF, EEG evidence of cerebral ischemia will appear if CBF is diminished sufficiently. The CBF reserve, however, is substantial and the first signs of cerebral ischemia do not appear on EEG until CBF has fallen to approximately 22 mL/100 g/min. When CBF has fallen to 15/100 g/min, the EEG becomes isoelectric. Irreversible membrane damage and cellular death do not occur, however, until CBF falls to 6 mL/100 g/min. Areas of the brain in which CBF falls in the 6 to 15 mL/100 g/min range are referred to as zones of ischemic penumbra. Several hours may elapse in these areas of the brain before irreversible membrane damage occurs (Miller: Anesthesia, ed 6, pp 833-834).
question
444. A 67-year-old patient is mechanically ventilated in the ICU 2 days after repair of a ruptured abdominal aortic aneurysm. To maintain PaO2 in the 60 to 65 range, 10 cm H2O positive end-expiratory pressure (PEEP) is added to the ventilator cycle. The patient's blood pressure has averaged 110/65 before addition of PEEP. After addition of PEEP, the blood pressure is noted to slowly fall to an average of approximately 95/50. The best explanation for this decrease in blood pressure is A. Tension pneumothorax B. Decreased venous return to the heart C. Increased afterload on the right side of the heart D. Increased afterload on the left side of the heart E. Decreased cardiac output from global myocardial ischemia
answer
444. (B) Positive end-expiratory pressure (PEEP) is the maintenance of positive airway pressure during the entire ventilator cycle. The addition of PEEP to the ventilator cycle is often recommended when PaO2 is not maintained above 60 mm Hg, when breathing an FIO2 of 0.50 or greater. Although not completely understood, PEEP is thought to increase arterial oxygenation, pulmonary compliance, and FRC by expanding previously collapsed but perfused alveoli, thereby decreasing shunt and improving ventilation/perfusion matching. An important adverse effect of PEEP is a decrease in arterial blood pressure caused by a decrease in venous return, left ventricular filling and stroke volume, and cardiac output. These effects are exaggerated in patients with decreased intravascular fluid volume. Other potential adverse effects of PEEP include pneumothorax, pneumomediastinum, and subcutaneous emphysema (Miller Anesthesia, ed 6, pp 2820-2821; Stoelting: Basics of Anesthesia, ed 5, p 596).
question
445. A 64-year-old male undergoes an elective cholecystectomy. Other than essential hypertension, for which he takes propranolol, he is in good health. The patient is anesthetized with isoflurane, N2O, and fentanyl, and paralyzed with d-tubocurarine. At the end of the operation, neuromuscular blockade is antagonized with pyridostigmine and atropine, the trachea is suctioned, and the patient is extubated and taken to the recovery room. Oxymorphone is administered IV for analgesia. One hour after arrival in the recovery room, the patient's heart rate decreases from 70 to 40 beats/min. Which of the following would most likely account for bradycardia in this patient? A. Recurarization B. Oxymorphone C. Pyridostigmine D. Propranolol E. Paradoxical effect of atropine
answer
445. (C) Reversal of neuromuscular blockade with an anticholinesterase drug requires coadministration of an anticholinergic drug to prevent the muscarinic side effects (e.g., bradycardia and salivation) from the neuromuscular reversal agent. The onset of neuromuscular reversal activity is most rapid with edrophonium, followed by neostigmine and pyridostigmine. The durations of action of edrophonium and neostigmine are similar, but pyridostigmine has a longer duration of action. Of the anticholinergic drugs, glycopyrrolate has a longer duration of action than atropine and for this reason should be coadministered with pyridostigmine. In this question, the patient received long-acting pyridostigmine in combination with short-acting atropine. After the effects of atropine wore off, the antimuscarinic effects of pyridostigmine became evident, resulting in bradycardia (Stoelting: Pharmacology and Physiology in Anesthetic Practice, ed 4, pp 258-259).
question
446. Which of the following is most closely associated with minimum alveolar concentration (MAC)? A. Blood/gas partition coefficient B. Oil/gas partition coefficient C. Vapor pressure D. Brain/blood partition coefficient E. Molecular weight
answer
446. (B) As a rough approximation, if one divides 150 by the MAC for any given volatile anesthetic, the quotient will be approximately equal to the oil/gas partition coefficient. For example, if one were to divide the MAC of halothane (0.75) into 150, the quotient would be 200, which is very close to the actual oil/gas partition coefficient for halothane (224). Similarly, if one were to divide the MAC of enflurane (1.68) into 150, the quotient would be 89, which is very similar to the oil/gas partition coefficient for enflurane (98). The fact that anesthetics with a high oil/gas partition coefficient (i.e., lipid-soluble agents) have lower MACs supports the Meyer-Overton theory (critical volume hypothesis) (Stoelting: Pharmacology and Physiology in Anesthetic Practice, ed 4, p 29).
question
447. A 15-year-old, 65-kg patient with Cushing's disease is to undergo a transsphenoidal hypophysectomy to remove a pituitary adenoma. General anesthesia is induced with thiopental IV and tracheal intubation is facilitated with vecuronium 0.25 mg/kg IV. Anesthesia is maintained with isoflurane, N2O and O2. Mannitol 1 g/kg is administered IV to reduce intracranial pressure. At the end of the operation, the patient is extubated and taken to the ICU. Over the next 6 hours the patient has a total urine output of 8.3 L. Serum sodium concentration is 154 mEq/L, serum potassium concentration is 4.8 mEq/L, and serum glucose concentration is 160 mg/dL. Urine specific gravity is 1.002 and urine osmolality is 125 mOsm/L. The most likely cause of the large urine output is A. Osmotic diuresis from mannitol B. Excess mineralocorticoid activity C. Hyperglycemia D. Nephrogenic diabetes insipidus E. Central diabetes insipidus
answer
447. (E) Diabetes insipidus is characterized by hypernatremia, serum hyperosmolality, polyuria, and urine hypoosmolality. Diabetes insipidus may occur after any intracranial procedure, but it is particularly common in surgery involving the pituitary gland. It may develop intraoperatively, but it commonly develops 4 to 12 hours postoperatively. Intravenous half normal saline and dextrose 5% in water are started as replacement fluids. The pharmacologic treatment for diabetes insipidus is synthetic ADH, 1-(3-mercaptoproprionic acid)-D-arginine vasopressin (DDAVP) commonly started when the urine output is greater than 350-400 mL/hr. In a conscious patient it is not essential to administer DDAVP because the patient may increase his oral intake to compensate for polyuria. In the unconscious patient, however, administration of DDAVP is necessary. Vasopressin (DDAVP) may be administered SQ, IV, or intranasally. Fortunately, diabetes insipidus related to surgery and head trauma usually is transient (Hines: Stoelting's Anesthesia and Co-Existing Disease, ed 5, pp 403-404; Miller: Anesthesia, ed 6, p 2159)
question
448. Scopolamine should not be given as a premedication in patients with which of the following neurologic diseases? A. Parkinson's disease B. Alzheimer's disease C. Multiple sclerosis D. Narcolepsy E. Amyotrophic lateral sclerosis
answer
448. (B) The principal feature of Alzheimer's disease is progressive dementia. The onset typically occurs after age 60 years and may affect as many as 20% of patients more than age 80 years. In addition to age, other risk factors include history of serious head trauma (e.g., boxing), Down syndrome, and presence of the disease in a parent or sibling. One biochemical feature of this disease is a decrease in the enzyme choline acetyltransferase in the brain. There is a strong correlation between reduced enzyme activity and decreased cognitive function. Interestingly, administration of the anticholinergic drugs scopolamine or atropine (but not glycopyrrolate which does not cross the blood-brain barrier) causes confusion similar to that seen in the early stages of Alzheimer's disease. Conversely, administration of anticholinesterase drugs capable of penetrating the blood-brain barrier, such as donepezil (Aricept), galantamine, rivastigmine (Exelon), and tacrine are used to treat patients with Alzheimer's disease. Physostigmine may have beneficial effects in some patients as well. Scopolamine is therefore a poor choice for premedication in patients with Alzheimer's disease (Hines: Stoelting's Anesthesia and Co-Existing Disease, ed 5, p 227; Morgan: Clinical Anesthesiology, ed 4, p 652).
question
449. A 63-year-old male patient is scheduled to undergo a right hemicolectomy under general anesthesia. Anesthesia is induced with thiopental 4 mg/kg IV and fentanyl 100 µg IV. Succinylcholine 1.5 mg/kg IV is administered to facilitate tracheal intubation. Anesthesia is maintained with isoflurane and N2O. After all four twitches of the train-of-four stimulus return to baseline values, pancuronium 5 mg IV is administered. Gentamicin 80 mg and cefazolin 1 g are administered IV as a prophylactic treatment. At the end of surgery, two of four thumb twitches can be elicited to train-of-four stimulation of the ulnar nerve and neuromuscular blockade is antagonized with neostigmine 0.05 mg/kg IV and atropine 0.015 mg/kg IV. The patient, however, begins to move before the incision is completely closed, and succinylcholine 40 mg IV is given. Fifteen minutes later, all anesthetics are discontinued and the patient is ventilated with 100% O2, but the patient remains apneic. The most likely cause of apnea is A. Fentanyl B. Recurarization C. Succinylcholine D. Thiopental E. Gentamicin
answer
449. (C) At the end of any general anesthetic, spontaneous ventilation must be restored before the patient can be extubated. The differential diagnosis for persistent apnea includes muscle relaxants (inadequate reversal or pseudocholinesterase deficiency), volatile anesthetics, narcotics, hypocarbia, damage to the phrenic nerves bilaterally, and the possibility of a central nervous system (CNS) event. Succinylcholine is hydrolyzed by pseudocholinesterase to succinylmonocholine and choline. This is further hydrolyzed by plasma cholinesterase to succinic acid and choline. All of the anticholinesterase agents used to reverse nondepolarizing neuromuscular blockade also inhibit pseudocholinesterase. Administration of succinylcholine to any patient who has already received an anticholinesterase will result in a prolonged block from the succinylcholine because it can no longer be easily hydrolyzed. In this patient, therefore, succinylcholine would be by far the most likely cause of apnea at the end of the operation (Stoelting: Pharmacology and Physiology in Anesthetic Practice, ed 4, p 218).
question
450. A 53-year-old female with endometrial cancer is undergoing an abdominal hysterectomy under general anesthesia with enflurane. During the first hour of anesthesia, urine output is 100 mL. Blood loss is minimal. When the patient is placed in the Trendelenburg position, the urine output declines to virtually zero. The most likely explanation for this sudden decrease in urine output in this patient is A. Pooling of urine in the dome of the bladder B. Kinking of the urinary catheter C. Fluoride toxicity from enflurane D. Increased antidiuretic hormone (ADH) production from surgical stimulation E. Hypovolemia
answer
450. (A) Pooling of urine in the dome of the bladder should be considered as a possible cause of oliguria in a patient in the Trendelenburg position. Acute hypovolemia is an unlikely cause of oliguria in this patient in the absence of bleeding. Fluoride toxicity from the metabolism of enflurane is extremely rare and is associated with nonoliguric renal failure. See also the answer to question 424 (Morgan: Clinical Anesthesiology, ed 4, pp 739- 741).
question
451. Which of the following diseases is not associated with a decrease in DLCO? A. Emphysema B. Lung resection C. Pulmonary emboli D. Anemia E. Obesity
answer
451. (E) DL is defined as the diffusing capacity of the lung. When a nontoxic low concentration of carbon monoxide is used for the measurement it is called DLCO. The normal value of DLCO is 20 to 30 mL/min/mm Hg and is influenced by the volume of blood (hemoglobin) within the pulmonary circulation. Thus, diseases associated with a decrease in pulmonary blood volume (i.e., anemia, emphysema, hypovolemia, pulmonary hypertension) will be reflected by a decrease in the DLCO. DLCO is also decreased with oxygen toxicity as well as pulmonary edema. Conditions associated with an increased DLCO include the supine position, exercise, obesity and leftto-right cardiac shunts (Barash: Clinical Anesthesia, ed 5, pp 806-807; Miller: Anesthesia, ed 6, p 1011).
question
452. Each of the following postoperative complications of thyroid surgery can result in upper airway obstruction EXCEPT A. Tracheomalacia B. Tetany C. Cervical hematoma D. Bilateral recurrent laryngeal nerve injury E. Bilateral superior laryngeal nerve injury
answer
452. (E) Patients undergoing thyroid surgery are at risk for airway obstruction from a number of causes. Postoperative hemorrhage sufficient to cause a large hematoma could compress the trachea and cause airway obstruction because of the close proximity of the thyroid gland to the trachea. Permanent hypoparathyroidism is a rare complication that may cause hypocalcemia leading to progressive stridor followed by laryngospasm. The most common nerve injury after thyroid surgery is damage to the abductor fibers of the recurrent laryngeal nerve. Unilaterally, this is manifested as hoarseness. Bilateral recurrent laryngeal nerve damage, however, may lead to airway obstruction during inspiration. Selective injury of the adductor fibers of the recurrent laryngeal nerve is a possible complication of thyroid surgery. This injury would leave the vocal cords open because the abductor fibers would be unopposed, placing the patient at great risk for aspiration. The superior laryngeal nerve has an extrinsic branch that innervates the cricothyroid muscle (which tenses the vocal cords), and an internal branch that provides sensory innervation to the pharynx above the vocal cords. Bilateral damage to this nerve would result in hoarseness and would predispose the patient to aspiration but would not lead to airway obstruction per se (Stoelting: Basics of Anesthesia, ed 5, pp 443-444).
question
453. The most sensitive early sign of MH during general anesthesia is A. Tachycardia B. Hypertension C. Fever D. Hypoxia E. Increased end-expiratory CO2 tension (PECO2)
answer
453. (E) Malignant hyperthermia is a clinical syndrome that may develop rapidly or take hours to manifest, sometimes not occurring until the patient is in the recovery room. Clinical signs include hypertension, tachycardia, respiratory acidosis, metabolic acidosis, muscle rigidity, myoglobinuria, and fever. The diagnosis of MH is unlikely, however, if only one of these signs is manifested. Because MH is a metabolic disorder, one of the first sensitive signs is an increase in the production of CO2 and concomitant respiratory acidosis. This is the most reliable early sign of the syndrome (Miller: Anesthesia, ed 6, p 1178).
question
454. A 78-year-old female is anesthetized for a right hemicolectomy for 3 hours. At the end of the operation the patient's blood pressure is 130/85 mm Hg, heart rate is 84 beats/min, core body temperature is 35.4° C, and PECO2 on mass spectrometer is 38 mm Hg. Which of the following would be the LEAST plausible reason for prolonged apnea in this patient? A. Residual neuromuscular blockade B. Narcotic overdose C. Cerebral hemorrhage D. Unrecognized obstructive pulmonary disease and high baseline PaCO2 E. Persistent intraoperative hyperventilation
answer
454. (E) Hyperventilation to PaCO2 of 20 mm Hg or higher for more than 2 hours will result in active transport of HCO3 out of the CNS. This results in spontaneous breathing at a lower (not higher) PaCO2. The other choices should be included in the differential diagnosis of apnea (Hines: Stoelting's Anesthesia and Co-Existing Disease, ed 5, pp 359-361; Miller: Anesthesia, ed 6, p 718).
question
455. A 68-year-old woman with severe rheumatoid arthritis undergoes pulmonary function evaluation before an elective abdominal surgery. Forced expiratory volume in one second (FEV1) and forced vital capacity (FVC) are within normal limits; however, the maximum voluntary ventilation (MVV) is only 40% of predicted. The next step in the pulmonary function evaluation of this patient should be to A. Obtain ABGs on room air B. Obtain a flow-volume loop C. Obtain a measurement of peak flow D. Obtain a ventilation/perfusion scan E. Assume, in the face of normal FEV1, poor effort on the part of the patient and proceed
answer
455. (B) Maximum voluntary ventilation (MVV) is a nonspecific pulmonary function test that measures the endurance of the ventilatory muscles and indirectly reflects the compliance of the lung and thorax as well as airway resistance. A decreased MVV may be caused by impairment to inspiration or expiration. In this patient, FEV1 is normal, which strongly suggests that the ventilatory impairment is during inspiration. A flow-volume loop would be a very useful confirmatory test (Barash: Clinical Anesthesia, ed 5, pp 805-808, 815-817).
question
456. Which of the following is NOT a component of the post-anesthetic discharge scoring system (PADSS) used to evaluate the suitability of a patient to be discharged from an ambulatory surgical facility? A. Drinking B. Ambulation C. Nausea and vomiting D. Pain E. Surgical bleeding
answer
456. (A) Guidelines for safe discharge of patients from ambulatory surgical centers include stable vital signs, ability to walk without dizziness, controlled pain, absence of nausea and vomiting, and minimal surgical bleeding. The PADSS is a tool for objectively assessing a patient's readiness for discharge from the surgical center and includes these five criteria. Requirements to drink fluids and to void before home discharge are controversial and are not parameters included in the PADSS (Barash: Clinical Anesthesia, ed 5, pp 1242-1243; Miller: Anesthesia, ed 6, pp 2708-2709).
question
457. During emergency repair of a mandibular jaw fracture in an otherwise healthy 19-year-old male, the patient's temperature is noted to rise from 37° C on induction to 38° C after 2 hours of surgery. Which of the following informational items would be LEAST useful in ruling out MH in this patient? A. Normal heart rate and blood pressure B. History of negative caffeine-halothane contracture test carried out 6 months earlier C. History of a uncomplicated general anesthetic at age 16 years with halothane and succinylcholine D. Normal ABGs drawn when the patient's temperature reached 38° C E. No increase in respiration rate with spontaneous breathing
answer
457. (C) Malignant hyperthermia is a difficult diagnosis to make on clinical grounds alone. Signs of MH may be fulminant or very subtle. They may occur immediately after induction or may not be manifested until the patient has reached the recovery room or even later. Malignant hyperthermia is a disorder of metabolism and is associated with hypertension, tachycardia, dysrhythmias, respiratory acidosis, metabolic acidosis, muscular rigidity, rhabdomyolysis, and fever. Contrary to what one might believe based on the name of this disease, fever is typically a late finding. Other diseases that may mimic MH include alcohol withdrawal, acute cocaine toxicity, bacteremia, pheochromocytoma, hyperthyroidism, and neuroleptic malignant syndrome. An elevation in temperature alone with normal blood gases, heart rate, and blood pressure, and no evidence of muscle breakdown would very likely not be due to MH. If a patient had been previously subjected to muscle biopsy and caffeine-halothane contracture testing with negative results, MH would be exceedingly rare, although a false-negative result is possible. A history of a previous anesthetic without MH triggering would be of little reassurance in a patient in whom a MH episode is suspected. It is not uncommon for MH-susceptible individuals to not trigger when a triggering anesthetic is administered initially but develop fulminant MH with a subsequent anesthetic (Miller: Anesthesia, ed 6, pp 1180-1186).
question
458. Which of the following drugs is useful in the treatment of asthma by specifically interfering with the leukotriene pathway? A. Fluticasone (Flovent) B. Ipratropium bromide (Atrovent) C. Triamcinolone (Azmacort) D. Montelukast (Singulair) E. Salmeterol (Serevent)
answer
458. (D) Asthma is an inflammatory illness that has bronchial hyperreactivity and bronchospasm as a result. Treatment is first directed at the inflammatory component as the underlying problem, reserving bronchodilators for symptomatic use. Because leukotrienes may function as inflammatory mediators, the leukotriene pathway inhibitors such as zileuton and the leukotriene receptor antagonist montelukast (Singulair) are being used for treatment of asthma. Zileuton and montelukast are only available as oral preparations, whereas the other drugs listed are given by inhalation. Fluticasone and triamcinolone are anti-inflammatory corticosteroids. Ipratropium is a quaternary ammonium compound formed by the introduction of an isopropyl group to the N atom of atropine and produces effects similar to those of atropine. One unexpected finding is a relative lack of effect on mucociliary clearance, which makes it useful in patients with airway disease, especially if parasympathetic tone of the airways is increased. Salmeterol is a ?2-selective adrenergic drug (Hardman: Goodman ; Gilman's The Pharmacological Basis of Therapeutics, ed 11, pp 721-725, 730-731).
question
459. A 68-year-old, 100-kg patient is undergoing a transurethral resection of the prostate gland under general anesthesia. Upon arrival in the recovery room, the patient appears restless and confused. Serum sodium is checked and found to be 110 mEq/L. How many mEq of sodium are needed to raise the serum [Na+] to 120 mEq/L? A. 300 mEq B. 400 mEq C. 500 mEq D. 600 mEq E. 700 mEq
answer
459. (D) Acute decreases in serum sodium, due to absorption of bladder irrigating fluids, rarely cause symptoms unless the sodium level drops below 120 mEq/L. At this level, tissue edema may develop and clinical neurologic signs (e.g., restlessness, nausea, confusion, seizures, coma) or ECG changes (e.g., widening of the QRS complex, elevation of the ST segment, ventricular tachycardia, or ventricular fibrillation) may be manifested. Treatment of mild decreases in serum sodium (i.e., 120 to 135 mEq/L with no neurologic or ECG changes) is by fluid restriction and/or administration of a diuretic such as furosemide. When the sodium levels drops below 120 mEq/L and neurologic symptoms or changes in the ECG develop, sodium chloride administration is needed. To calculate the amount needed, one multiplies the patient's total body water (i.e., 0.6 × body weight = TBW) by the change in sodium desired. In this case, the TBW is 60 L (0.6 × 100 kg) and the change of sodium is 10 mEq (120 mEq/L ? 110 mEq/L), thus 60 L × 10 mEq/L = 600 mEq. Caution is advised in administering sodium because too rapid administration may lead to demyelinating CNS lesions. The recommended rate of 3% sodium chloride (513 mEq/L) is 1 to 2 mL/kg/hour. Serum sodium levels should be checked at least every hour until the sodium level increases above 120 mEq/L (Barash: Clinical Anesthesia, ed 5, pp 188-192, 1026-1028).
question
460. Trismus after administration of succinylcholine IV signals the onset of MH in what percentage of patients? A. Less than 50% B. 50% C. 75% D. 80% E. Greater than 80%
answer
460. (A) Trismus (masseter spasm) is characterized by rigidity of the jaw muscles while the limb muscles remain flaccid after administration of succinylcholine. Trismus may herald the onset of MH in some patients, but may be due to a number of other causes and may occur in normal patients. It previously had been believed that 50% of patients who experience trismus after administration of succinylcholine would go on to develop MH. Recent evidence suggests, however, that the incidence is less. If masseter spasm occurs in a patient after administration of succinylcholine, the most conservative course would be to cancel the operation. If cancellation of the operation is not feasible, then a nontriggering anesthetic should be used and the anesthesiologist should pay close attention for any signs of MH (Miller: Anesthesia, ed 6, pp 1180-1181).
question
461. A 45-year-old male is brought to the OR emergently for repair of a ruptured abdominal aortic aneurysm. The patient is pretreated with d-tubocurarine 3 mg, anesthesia is induced with ketamine 2 mg/kg IV, and tracheal intubation is facilitated with succinylcholine 1.5 mg/kg IV. Immediately after tracheal intubation, the patient's blood pressure falls from 110/80 to 50/20 mm Hg. What is the most likely cause of the sudden severe hypotension in this patient? A. Hypovolemia B. Direct myocardial depression from ketamine C. Vasovagal response to direct laryngoscopy D. Arteriolar vasodilation from succinylcholine-mediated histamine release E. Ganglionic blockade from d-tubocurarine
answer
461. (B) Ketamine is unique among the IV induction agents in that it usually produces cardiac stimulation manifested by increased heart rate, mean arterial pressure, and cardiac output. Ketamine is believed to have a centrally mediated sympathetic nervous system-stimulating effect. This effect is, however, not related to dose. In isolated rabbit and canine hearts and in intact dogs, ketamine has been demonstrated to produce myocardial depression. Clinically, however, the myocardial depressant properties of ketamine are overridden by its sympathetic nervous system stimulating properties. When systemic catecholamines have been depleted or when the patient is under deep anesthesia, the myocardial depressant properties of ketamine may predominate (Stoelting: Pharmacology and Physiology in Anesthetic Practice, ed 4, p 172).
question
462. Malignant hyperthermia is believed to involve a generalized disorder of membrane permeability to A. Sodium B. Potassium C. Calcium D. Magnesium E. Phosphate
answer
462. (C) In the normal muscle cell, depolarization results in release of calcium from the sarcoplasmic reticulum. The increased intracellular calcium concentration results in muscle contraction. The calcium then is rapidly taken up via calcium pumps back into the sarcoplasmic reticulum, resulting in relaxation. Both the release and reuptake of calcium are energy-requiring processes, i.e., result in the hydrolysis of adenosine triphosphate (ATP). Dantrolene, the pharmacologic treatment for MH, blocks release of calcium from the sarcoplasmic reticulum without affecting the reuptake process. The defect in MH is thought to be decreased control of intracellular calcium stores preventing muscle relaxation (Miller: Anesthesia, ed 6, pp 1170-1179).
question
463. A 25-year-old male with a history of testicular cancer is scheduled to undergo an exploratory laparotomy under general anesthesia. He has received bleomycin for metastatic disease. Which of the following is an important consideration concerning the pulmonary toxicity of bleomycin? A. N2O should not be used B. Preoperative pulmonary function tests should be obtained C. The patient should be ventilated at a slow rate and inspiratory-to-expiratory (I:E) ratio of 1:3 D. Aminophylline should be started preoperatively E. FIO2 should be less than 0.3
answer
463. (E) Approximately 4% of patients treated with bleomycin develop pulmonary toxicity, which manifests as severe pulmonary fibrosis and hypoxemia. Death from severe pulmonary toxicity occurs in approximately 1% to 2% of patients treated with bleomycin. Patients who are at greater risk for bleomycin-induced pulmonary toxicity include elderly patients, those receiving more than 200 to 400 mg, those with coexisting lung disease, and those recently exposed to bleomycin. In addition, there is evidence that prior radiotherapy and possibly receipt of enriched concentrations of O2 (i.e., inspired oxygen > 30%) during surgery increase risk of pulmonary toxicity. Clinically, patients gradually develop dyspnea, a nonproductive cough, and hypoxemia, and pulmonary function tests typically demonstrate changes in gas flow and lung volumes consistent with restrictive pulmonary disease. If radiographic evidence such as bilateral diffuse interstitial infiltrates appears, pulmonary fibrosis usually is irreversible (Stoelting: Pharmacology and Physiology in Anesthetic Practice, ed 4, pp 564- 565).
question
464. A 39-year-old obese female undergoes an abdominal hysterectomy under general anesthesia. Induction of anesthesia is uneventful. SaO2 is 98% during the first 15 minutes of the operation. However, when her head is flexed and she is placed in the Trendelenburg position to improve surgical exposure, SaO2 falls to 90%. The most likely explanation for this desaturation is A. Diffusion hypoxia B. Decreased functional residual capacity (FRC) C. Mainstem intubation D. Decreased cardiac output E. Venous air embolism
answer
464. (C) Head flexion can advance the tube up to 1.9 cm toward the carina and in some cases convert an endotracheal intubation into an endobronchial intubation. Extension of the head has the opposite effect and can withdraw the tube up to 1.9 cm, resulting in extubation of some patients. Turning the head laterally can move the distal tip of the endotracheal tube about 0.7 cm away from the carina. The Trendelenburg position causes a cephalad shift of the mediastinum and can cause the endotracheal tube to migrate distally as well (Lobato: Complications in Anesthesiology, p 834; Stoelting: Basics of Anesthesia, ed 5, p 232).
question
465. How long after intravitreal injection of sulfur hexafluoride and air can N2O be used without risk of increasing intraocular pressure? A. 1 hour B. 24 hours C. 10 days D. 1 month E. Never
answer
465. (C) Sulfur hexafluoride is sometimes injected in the vitreous in patients with a detached retina to mechanically facilitate reattachment. To prevent changes in the size of the gas bubble, the patients should be given 100% O2 15 minutes before injection of sulfur hexafluoride. If these patients are anesthetized with general anesthesia within 10 days, N2O should not be given because N2O can diffuse into the gas bubble, increasing intraocular pressure, and may result in blindness (Barash: Clinical Anesthesia, ed 5, p 982).
question
466. A 54-year-old female is undergoing a total thyroidectomy under general anesthesia. The patient is awakened in the OR, the mouth and pharynx are suctioned, and after intact laryngeal reflexes are demonstrated, the endotracheal tube is removed. Two days later, the anesthesiologist is consulted because the patient has severe stridor and upper airway obstruction. The most likely cause of airway obstruction in this patient is A. Damage to the recurrent laryngeal nerve B. Damage to the superior laryngeal nerve C. Tracheomalacia D. Hypocalcemia E. Hematoma
answer
466. (D) The symptoms of hypocalcemia, which may be manifested as laryngospasm or laryngeal stridor, usually develop within the first 24 to 96 hours after total thyroidectomy. After the airway is established and secured, the patient should be treated with IV calcium in the form of either calcium gluconate or calcium chloride (Barash: Clinical Anesthesia, ed 5, pp 199, 1133).
question
467. A 27-year-old obese woman is scheduled to undergo foot surgery under general anesthesia. She underwent a subtotal thyroidectomy 3 years ago and takes levothyroxine (Synthroid). Which of the following laboratory tests would be the most useful in evaluating whether this patient is euthyroid? A. Total plasma thyroxine (T4) B. Total plasma triiodothyronine (T3) C. Thyroid-stimulating hormone (TSH) D. Resin triiodothyronine uptake E. Radioactive iodine uptake
answer
467. (C) Because the circulating levels of T3 and T4 regulate TSH release from the anterior pituitary gland by a negative feedback mechanism, a normal plasma concentration of TSH confirms an euthyroid state. The pharmacologic treatment of choice for patients with hypothyroidism is sodium levothyroxine (T4). Sodium levothyronine (triiodothyronine, T3) and desiccated thyroid are alternate therapeutic agents (Barash: Clinical Anesthesia, ed 5, pp 1130-1134; Hines: Stoelting's Anesthesia and Co-Existing Disease, ed 5, pp 378-381).
question
468. An 85-year-old male with no previous medical history except for cataracts is undergoing a transurethral resection of the prostate gland under spinal anesthesia. Twenty minutes into the procedure the patient becomes restless. Over the next 20 minutes his blood pressure increases from 110/70 to 140/90 mm Hg and his heart rate slows from 90 to 50 beats/min. The patient is noted to have some difficulty breathing. The most likely cause of these symptoms in this patient is A. Volume overload B. Hyponatremia C. High spinal D. Bladder perforation E. Autonomic hyperreflexia
answer
468. (A) Large quantities of irrigating fluid can be absorbed during transurethral resection of the prostate gland because the open venous sinuses in the prostate allow the irrigation fluid to be absorbed. From 10 to 30 mL of fluid per minute are absorbed on the average. During long cases, this can amount to several liters, causing hypertension, reflex bradycardia, and pulmonary congestion. Treatment consists of fluid restriction and a loop diuretic (e.g., furosemide) when the [Na+] level is greater than 120 mEq/L. Rarely does the amount of fluid absorbed cause significant hyponatremia (i.e., ;120 mEq/L). In these cases of significant hyponatremia, 3% sodium chloride may be infused slowly intravenously (in addition to the loop diuretic and fluid restriction) until the sodium level reaches 120 mEq/L (Barash: Clinical Anesthesia, ed 5, pp 1027-1028).
question
469. A 17-year-old patient with third-degree burns over 30% of his body is scheduled for débridement and skin grafting 12 days after sustaining a thermal injury. Select the true statement regarding the use of depolarizing and nondepolarizing muscle relaxants in this patient, compared with normal patients. A. Sensitivity to both depolarizing and nondepolarizing muscle relaxants is increased B. Sensitivity to both depolarizing and nondepolarizing muscle relaxants is decreased C. Sensitivity to depolarizing muscle relaxants is increased while sensitivity to nondepolarizing muscle relaxants is decreased D. Sensitivity to depolarizing muscle relaxants is decreased while sensitivity to nondepolarizing muscle relaxants is increased E. Sensitivity to nondepolarizing is unchanged while sensitivity to depolarizing muscle relaxants is increased
answer
469. (C) Patients who have sustained thermal injuries are at risk for massive potassium release and potential cardiac arrest if succinylcholine is administered 24 hours or more after they sustain the burn, and they remain at risk until the burn has healed. This increased sensitivity to succinylcholine is thought to be related to proliferation of extrajunctional receptors. These same receptors are thought to be related to the increased requirement for nondepolarizing neuromuscular blocking agents in these patients (Barash: Clinical Anesthesia, ed 5, p 1288).
question
470. A nervous 57-year-old woman with morbid fear of intramuscular injections is scheduled for breast biopsy and is given a premedication in the waiting area before surgery. The anesthesiologist is summoned 45 minutes later because the patient is complaining of dry mouth and has a heart rate of 45 beats/min. Which of the following premedications is most likely responsible for these side effects? A. Scopolamine B. Meperidine C. Midazolam D. Clonidine E. Droperidol
answer
470. (D) Scopolamine, an anticholinergic, has stronger antisialagogue effects than glycopyrrolate or atropine. Scopolamine has better sedative and amnesic effects than atropine. Glycopyrrolate, which does not cross the blood-brain barrier, has no CNS or bradycardiac effects. Although heart rate may decrease with atropine and scopolamine, these effects are minimal (i.e., about 4 to 8 beats/min). These anticholinergics usually are administered parenterally; however, a scopolamine patch could be applied topically. Meperidine may produce a dry mouth in some patients, but it is much more likely to increase the heart rate rather than lower it because of its modest atropine-like effects. Meperidine is well absorbed from the gastrointestinal tract but is usually administered parenterally. Midazolam, which is usually administered parenterally, may produce bradycardia or tachycardia in selected patients but does not cause a dry mouth and may produce excessive salivation. Droperidol can produce tachycardia is some patients and is usually given parenterally. Clonidine is the most likely agent to cause bradycardia, hypotension and dry mouth. The oral dose is 5 ?g/kg (Stoelting: Basics of Anesthesia, ed 5, pp 170-172; Stoelting: Pharmacology and Physiology in Anesthetic Practice, ed 4, pp 340-344).
question
471. A 65-year-old patient with a history of chronic obstructive pulmonary disease and coronary artery disease (CAD) undergoes an appendectomy uneventfully under general anesthesia. In the recovery room, ABGs are as follows: PaO2 60 mm Hg, PaCO2 50 mm Hg, pH 7.35, and hemoglobin 8.1 g/dL. Which of the following steps would produce the greatest increase in O2 delivery to the myocardium? A. Administration of 100% O2 with a close-fitting mask B. Administration of 35% O2 with a Venturi mask C. Withhold narcotics D. Transfuse with 2 units of packed red blood cells (RBCs) E. Administer 1 ampule of HCO3
answer
471. (D) One gram of hemoglobin can combine with 1.34 mL of O2. None of the other choices in this question will do as much to increase the O2-carrying capacity of this patient's blood as a transfusion (Stoelting: Pharmacology and Physiology in Anesthetic Practice, ed 4, p 849).
question
472. Allergic reactions occurring during the immediate perioperative period are most commonly attributable to administration of A. Muscle relaxants B. Local anesthetics C. Antibiotics D. Opioids E. ?-Blockers
answer
472. (A) Many of the drugs commonly administered during surgery and anesthesia have the potential to evoke allergic reactions (e.g., pentothal, propofol, local anesthetics, antibiotics, and protamine, as well as other materials used during surgery, such as vascular graft material, chymopapain, and latex). Virtually all drugs administered IV have been reported to cause allergic reactions. Possible exceptions include benzodiazepines and ketamine. An allergic reaction should be considered when there is an abrupt fall in blood pressure accompanied by increases in heart rate that exceed 30% of the control values. Greater than 60% of all drug-induced allergic reactions observed during the perioperative period are attributable to muscle relaxants. Latex allergy is thought to be responsible for 15% of allergic reactions under anesthesia, sometimes including reactions originally attributed to other substances. Patients at risk for latex allergy include health care workers and patients with spina bifida. Although most drug-induced allergic reactions develop within 5 to 10 minutes of exposure, latex signs typically take more than 30 minutes to develop (Hines: Stoelting's Anesthesia and Co- Existing Disease, ed 5, pp 527-530; Miller: Anesthesia, ed 6, p 1092).
question
473. Caution is advised when using succinylcholine in patients with Huntington's chorea because A. They are at increased risk for MH B. Potassium release may be excessive C. They may have a decreased concentration of pseudocholinesterase D. There may be adverse interactions between succinylcholine and phenothiazine E. Succinylcholine increases intracranial pressure
answer
473. (C) Decreased levels of pseudocholinesterase have been reported in patients with Huntington's chorea. For this reason, the effects of succinylcholine may be prolonged in some of these patients. It has been suggested that the sensitivity to nondepolarizing muscle relaxants is also increased (Hines: Stoelting's Anesthesia and Co- Existing Disease, ed 5, p 229).
question
474. Which of the following would not result in an increase in intraocular pressure? A. Increase in PaCO2 from 35 to 40 mm Hg B. Arterial hypoxemia C. 100 mg IM succinylcholine D. Acute rise in venous pressure from coughing E. 100 mg IV succinylcholine in patient in whom eye muscles have been detached from the globe
answer
474. (A) Normal intraocular pressure is 10 to 22 mm Hg. In general, IV anesthetics, with the possible exception of ketamine, decrease intraocular pressure. In addition, nondepolarizing neuromuscular blockers, inhaled anesthetics, narcotics, carbonic anhydrase inhibitors, osmotic diuretics, and hypothermia decrease intraocular pressure. However, elevation of PaCO2 out of the physiologic range, as seen with hypoventilation as well as arterial hypoxemia, will increase intraocular pressure. Depolarizing neuromuscular blockers, such as succinylcholine, also increase intraocular pressure. This increase in intraocular pressure occurs when succinylcholine is administered IM or IV. Pretreatment with a nondepolarizing muscle relaxant before administering succinylcholine may attenuate the rise in intraocular pressure. The mechanism for the increase in intraocular pressure after succinylcholine use is related to drug-induced cycloplegia rather than contraction of extraocular muscles, as this increase in intraocular pressure will occur even if the intraocular muscles are cut. The greatest increase in intraocular pressure occurs with coughing or vomiting, where the intraocular pressure may increase as much as 35 to 50 mm Hg. The proposed mechanism for the acute increase in intraocular pressure is an increase in venous pressure. There does not appear to be a change in intraocular pressure with changes within normal physiologic ranges in arterial blood pressure or PaCO2 (Barash: Clinical Anesthesia, ed 5, pp 978-980; Stoelting: Basics of Anesthesia, ed 5, pp 464-465).
question
475. An apnea hypopnea index of 30 means? A. Episodes of hypopnea are 30 times more common than apnea B. Episodes of apnea are 30 times more common than hypopnea C. Episodes of apnea and hypopnea occur at a rate of 30 per hour D. Apnea/hypopnea episodes last 30 seconds E. Apnea/hypopnea episodes occur at rate of 30 per sleep cycle
answer
475. (C) The apnea-hypopnea index (AHI) is used to quantify the number of apnea or hypopnea episodes that occur per hour. Apnea is defined as no ventilation for periods of 10 seconds or more. Hypopnea is defined as a 50% decrease in airflow or a decrease sufficient to cause a decrease in oxygen saturation of 4%. An apnea- hypopnea index of greater than 30 signifies severe OSA (Barash: Clinical Anesthesia, ed 5, p 1042; Lobato: Complications in Anesthesiology, p 625).
question
476. Which of the following preoperative pulmonary function tests is NOT associated with an increased operative risk for pneumonectomy? A. FEV1 less than 50% of the FVC B. FEV1 less than 2 L C. Maximum breathing capacity less than 50% of predicted D. Residual volume/total lung capacity less than 50% E. Hypercarbia on room air ABGs
answer
476. (D) Any patient who is scheduled for a pneumonectomy should undergo a series of preoperative pulmonary function tests. These tests are generally conducted in three phases. The tests listed in this question pertain to the first battery of pulmonary function tests, which are whole-lung tests. Residual volume to total lung capacity greater than 50% (not <50%) is associated with an increased operative risk. If the results of any of the initial whole-lung tests are below the acceptable limits, a second phase of testing should be carried out in which the function of each lung is evaluated separately. The predicted postoperative FEV1 after the second phase of pulmonary function testing is carried out should be greater than 0.85 L. If the criteria for the second level of pulmonary function testing cannot be met and pneumonectomy is still desired, then a third level of testing should be carried out. During the third phase of testing, postoperative conditions mimicking pneumonectomy are produced by occluding the pulmonary artery with a balloon on the side that is to be resected. Results of this test that are consistent with poor outcome after pneumonectomy include mean pulmonary artery pressure greater than 40 mm Hg, PaCO2 greater than 60 mm Hg, or PaO2 less than 45 mm Hg (Miller: Anesthesia, ed 6, pp 1852-1854).
question
477. A 26-year-old male patient is undergoing an emergency exploratory laparotomy under general anesthesia with isoflurane. SaO2 is 89% on the pulse oximeter. PaO2 on ABGs is 77 mm Hg. The patient's core body temperature is 35° C. What is the corrected PaO2? A. 68 mm Hg B. 72 mm Hg C. 77 mm Hg D. 86 mm Hg E. 92 mm Hg
answer
477. (A) Measured PaO2 should be decreased about 6% for each degree Celsius cooler the patient's temperature is than the electrode (37° C). Because the patient is 2° C cooler than the electrode, a 12% decrease (9 mm Hg) would be expected in this patient (77 mm Hg ? 9 mm Hg = 68 mm Hg) (Stoelting: Basics of Anesthesia, ed 5, p 325).
question
478. A 27-year-old patient with a 10-year history of Crohn's disease is scheduled to undergo drainage of a rectal abscess under general anesthesia. His preoperative medications include prednisone, sulfasalazine, and cyanocobalamin. He has no known allergies and is otherwise healthy. Before induction of anesthesia the patient is noted to have central cyanosis and the pulse oximeter shows a SaO2 of 89%, which does not increase after the administration of 100% O2 for 2 minutes. ABGs are as follows: PaO2 490 mm Hg, PaCO2 32 Hg, pH 7.43, SaO2 89%. The most likely cause of these findings is A. Presence of sulfhemoglobin B. Presence of methemoglobin C. Presence of cyanhemoglobin D. Presence of carboxyhemoglobin E. Blood gas error
answer
478. (A) The two main causes of central cyanosis are decreased arterial oxygen saturation and hemoglobin abnormalities (e.g., methemoglobinemia and sulfhemoglobinemia). Sulfasalazine (Azulfidine) can cause the formation of sulfhemoglobin. Sulfhemoglobin, like methemoglobin, may cause low O2 saturation in the face of high PaO2. There is no treatment for sulfhemoglobinemia except to wait for the destruction of the erythrocytes (Hines: Stoelting's Anesthesia and Co-Existing Disease, ed 5, pp 287-288; Kasper: Harrison's Principles of Internal Medicine, ed 16, pp 210-211).
question
479. The muscle relaxant of choice (i.e., minimal cardiovascular changes) during resection of a pheochromocytoma is A. Mivacurium B. Pancuronium C. Curare D. Atracurium E. Vecuronium
answer
479. (E) Muscle relaxants that stimulate histamine release or cause increased sympathetic outflow should not be given to patients with pheochromocytoma. Vecuronium has no histamine-releasing properties and does not stimulate the sympathetic nervous system. Mivacurium, curare, and metocurine and atracurium all release histamine to some degree. Pancuronium is associated with tachycardia and should be avoided in patients with pheochromocytoma. Of the listed relaxants in this question, vecuronium is the best choice; however, cisatracurium and rocuronium also have minimal effects on the cardiovascular system and would be good choices in patients with pheochromocytomas as well (Hines: Stoelting's Anesthesia and Co-Existing Disease, ed 5, p 392; Stoelting: Pharmacology and Physiology in Anesthetic Practice, ed 4, pp 223-241).
question
480. In a given patient, if a creatinine of 1.0 corresponds to a glomerular filtration rate (GFR) of 120 mL/min, a creatinine of 4.0 would correspond to A. 20 mL/min B. 30 mL/min C. 40 mL/min D. 50 mL/min E. 60 mL/min
answer
480. (B) Serum creatinine is inversely proportional to the GFR. With the increase in creatinine by a factor of 4, the GFR is divided by 4; i.e., 120/4 = 30 mL/min. (Lobato: Complications in Anesthesiology, p 433; Miller: Anesthesia, ed 6, pp 786-788).
question
481. The incidence of each of the following is increased in patients with Down syndrome (trisomy 21) EXCEPT A. Malignant hyperthermia B. Hypothyroidism C. Smaller trachea D. Occipito-atlantoaxial instability E. Congenital heart disease
answer
481. (A) Trisomy 21 or Down syndrome is the most common human chromosomal syndrome seen. An increase incidence of congenital hypothyroidism occurs. About one fourth of children with Down syndrome and many adults have smaller tracheas than predicted and require an endotracheal tube that is one or two sizes smaller. One should avoid unnecessary flexion or extension of the neck during intubation because occipito-atlantoaxial instability occurs in about 15% to 20% of patients. Because subluxation is relatively uncommon, routine neck radiographs for all Down patients are excessive. More than 40% of Down syndrome children have congenital heart disease, (e.g., endocardial cushion defects, ventricular septal defects, tetralogy of Fallot, patent ductus arteriosus). Although some children have hypotonia, an increased incidence of MH has not been reported in these patients (Baum: Anesthesia for Genetic, Metabolic, and Dysmorphic Syndromes of Childhood, ed 2, pp 105-107; Hines: Stoelting's Anesthesia and Co-Existing Disease, ed 5, pp 611-612; Miller: Anesthesia, ed 6, p 1099).
question
482. A 55-year-old male is to undergo a transurethral resection of the prostate gland under general anesthesia. The patient has a 40-pack-per-year smoking history and a history of congestive heart failure. The patient receives metoclopramide and scopolamine preoperatively. General anesthesia is induced with ketamine and the patient undergoes the procedure uneventfully. However, in the recovery room the patient complains of not being able to see objects "up close." Which of the following would be the most likely cause of this complaint? A. Emergence delirium from ketamine anesthesia B. Effect of scopolamine C. Effect of glycine in the irrigating solution D. Corneal abrasion E. Hyponatremia
answer
482. (B) Scopolamine is an anticholinergic that may produce mydriasis and cycloplegia. This can result in the inability of patient's eyes to accommodate (Stoelting: Basics of Anesthesia, ed 5, p 172).
question
483. Malignant hyperthermia and neuroleptic malignant syndrome share each of the following characteristics EXCEPT A. Generalized muscular rigidity B. Hyperthermia C. Effectively treated with dantrolene D. Tachycardia E. Flaccid paralysis after administration of vecuronium
answer
483. (E) Neuroleptic malignant syndrome is a potentially fatal disease that affects 0.5% to 1% of all patients being treated with neuroleptic (antipsychotic) drugs. The syndrome develops gradually over 1 to 3 days in young males and is characterized by the following: (1) hyperthermia, (2) skeletal muscle rigidity, (3) autonomic instability manifested by changes in blood pressure and heart rate, and (4) fluctuating levels of consciousness. The mortality from neuroleptic malignant syndrome is 20% to 30%. Liver transaminases and creatine phosphokinase levels are often elevated in these patients. Treatment includes supportive care and administration of dantrolene. This disease may mimic MH because of its many similarities. One difference between neuroleptic malignant syndrome and MH is the fact that nondepolarizing muscle relaxants such as vecuronium or cisatracurium will cause flaccid paralysis in patients with neuroleptic malignant syndrome but not in patients with MH (Stoelting: Pharmacology and Physiology in Anesthetic Practice, ed 4, p 412).
question
484. A 23-year-old male involved in a motor vehicle accident is brought to the OR for open reduction and internal fixation of bilateral leg fractures under general anesthesia. During the surgery the patient is transfused with 7 units of type AB, Rh-negative packed RBCs and 3 units of platelets. At the end of the procedure the endotracheal tube is removed and the patient is taken to the ICU. Postoperatively the patient complains of shortness of breath and arterial hypoxemia is noted. His temperature is 38° C, heart rate is 146 beats/min, blood pressure is 105/69 mm Hg, and respiratory rate is 36 breaths/min. In addition, the patient is noted to have a fine petechial rash on his neck, chest and shoulders. Which of the following is the most likely cause of these signs and symptoms? A. Pulmonary embolism B. Transfusion reaction from packed RBCs C. Transfusion reaction from platelets D. Fat embolism E. Sepsis
answer
484. (D) The classic signs of fat embolism include tachycardia, dyspnea, mental confusion, and fever, and frequently there may be a petechial rash on the upper part of the body. Fat embolism is more common after long bone fractures (e.g., femur and tibia) and usually occurs between 12 and 72 hours after long bone fractures (Hines: Stoelting's Anesthesia and Co-Existing Disease, ed 5, p 193).
question
485. Remifentanil is metabolized primarily by A. Kidneys B. Liver C. Hoffman elimination D. Pseudocholinesterase E. Nonspecific esterases
answer
485. (E) Remifentanil is an ultrashort-acting narcotic. Chemically it is a derivative of piperidine (like fentanyl), but remifentanil has an ester linkage and is rapidly broken down by nonspecific plasma as well as tissue esterases. The elimination half-life is less than 20 minutes and is best administered by a continuous infusion. Pseudocholinesterase deficiency or renal or hepatic failure does not affect remifentanil's rapid metabolism (Barash: Clinical Anesthesia, ed 5, pp 371, 374; Stoelting: Pharmacology and Physiology in Anesthetic Practice, ed 4, p 114).
question
486. A 3-year-old child is brought to the OR after aspiration of a peanut. After an inhalation induction the trachea is intubated. The peanut is extracted through a rigid bronchoscope but then is lost in the upper airway. The anesthesiologist notes that he can no longer ventilate the patient's lungs. What should be the next step in the management of this problem? A. Needle cricothyroidotomy B. Emergency tracheotomy C. Placement of a chest tube D. Push the peanut more distally E. Attempt jet ventilation
answer
486. (D) If a peanut or other foreign body becomes lost in the upper airway such that ventilation of the patient is impossible and retrieval is not feasible, the person performing the bronchoscopy should push the foreign body distally past the carina so that gas exchange can take place. Once the patient is stabilized, another attempt to retrieve the foreign body can be made (Motoyama: Smith's Anesthesia for Infants and Children, ed 7, pp 815- 818).
question
487. Patients who undergo extracorporeal shock-wave lithotripsy are at increased risk for A. Venous air embolism B. Pneumothorax C. Peripheral neuropathies D. Postdural puncture headache with spinal anesthesia E. Hypotension with regional anesthesia at the end of the procedure
answer
487. (E) Anesthesia for extracorporeal shock wave lithotripsy may be accomplished with either general anesthesia or epidural anesthesia. When a patient is submerged in the stainless steel tub, the peripheral vasculature becomes compressed by the hydrostatic pressure, resulting in an increase in preload. Removing the patient from the tank has the opposite effect. In patients who have received epidural anesthesia, there is an increased incidence of hypotension caused by epidural-induced sympathectomy after they emerge from the bath (Barash: Clinical Anesthesia, ed 5, pp 1030-1032).
question
488. The most common reason for admitting outpatients to the hospital following general anesthesia is A. Angina B. Inability to void C. Inability to ambulate D. Surgical pain E. Nausea and vomiting
answer
488. (E) The most common reason for unexpected hospital admission after outpatient general anesthesia, as well as a prolonged recovery-room stay (for both adults and children), is nausea and vomiting. Two other reasons for a prolonged recovery-room stay are pain and drowsiness (Barash: Clinical Anesthesia, ed 5, p 1242).
question
489. A 37-year-old male with myasthenia gravis arrives in the emergency room confused and agitated after a 2- day history of weakness and increased difficulty breathing. ABGs on room air are PaO2 60 mm Hg, PaCO2 51mm Hg, HCO3? 25 mEq/L, pH 7.3, SaO2 of 90%. His respiratory rate is 30 breaths/min and tidal volume (VT) is 4 mL/kg. After administration of edrophonium 2 mg IV, his VT declines to 2 mL/kg. What should be the most appropriate step in the management of this patient at this time? A. Tracheal intubation and mechanical ventilation B. Repeat the test dose of edrophonium C. Administer neostigmine 1 mg IV D. Administer atropine 0.4 mg IV E. Emergency tracheostomy and mechanical ventilation
answer
489. (A) Cholinergic crisis can be differentiated from myasthenic crisis by administering small IV doses of anticholinesterases. With a cholinergic crisis, there are significant muscarinic effects (e.g., salivation, bradycardia, miosis) and an accentuated muscle weakness. Because this patient's VT decreased with the administration of edrophonium, the diagnosis of cholinergic crisis is made. Although atropine may be needed to treat the cholinergic symptoms, muscle weakness will be worse and these patients need to be intubated until the muscle strength returns (Hines: Stoelting's Anesthesia and Co-Existing Disease, ed 5, p 452).
question
490. Select the FALSE statement regarding tramadol (Ultram). A. Ondansetron may interfere with part of tramadol's analgesia B. Tramadol is associated with seizures in patients taking selective serotonin reuptake inhibitors (SSRIs) C. It exhibits monoamine oxidase (MAO) inhibiting effects D. Its analgesic effects are partially antagonized by naloxone E. It is relatively safe in patients whose pain makes them suicidal
answer
490. (E) Tramadol, a synthetic codeine analogue, is a centrally acting analgesic. It can be used for mild to moderate pain but is not as effective as morphine or meperidine for severe or chronic pain. One drawback for tramadol's perioperative use is its high incidence of nausea and vomiting. Its mechanism of action for analgesia is complex. It is a weak ?-receptor agonist, it inhibits serotonin and norepinephrine reuptake, and it enhances serotonin release. Tramadol-induced analgesia is not entirely reversed with naloxone, however, the respiratory depression and sedation can be reversed. Ondansetron, a serotonin antagonist, may interfere with part of tramadol's analgesic action. Because of its low ?-receptor agonist activity, it may be less likely to produce physical dependence than other stronger narcotics. Seizures have been reported in patients receiving tramadol alone. The drug should be used with caution in patients taking drugs that lower the seizure threshold, such as tricyclic antidepressants and SSRIs. It has some MAO inhibiting activity and should not be used in patients taking MAO inhibitors. Another warning is its use in patients who are depressed or suicidal. Tramadol is not recommended in depressed or suicidal patients since excessive doses, either alone or with other CNS depressants including alcohol, are a major cause of drug-related deaths with fatalities reported within the first hour of overdosage. Patients who are depressed or suicidal are better managed with non- narcotic analgesics (Hardman: Goodman and Gilman's The Pharmacological Basis of Therapeutics ed 10, p 590; Physicians Desk Reference-2009, ed 63, pp 2428-2431; Stoelting: Pharmacology and Physiology in Anesthetic Practice, ed 4, p 117).
question
491. Which of the following patients would not be a good candidate for outpatient inguinal hernia repair under general anesthesia? A. A 62-year-old pharmacist who lives 10 miles away B. A 20-year-old healthy college student who had a renal transplant 3 years earlier C. A 38-year-old housewife with a hiatal hernia D. A premature infant who is 43 weeks postconceptual age E. A 29-year-old diabetic who is well controlled on insulin
answer
491. (D) Premature infants are at increased risk for development of apnea until they have reached 60 weeks postconceptual age (PCA), which is defined as gestational age at birth (GA) plus chronologic age. Apnea in these patients is central apnea, that is, apnea associated with the absence of respiratory effort related to immaturity of the CNS. Infants with a history of apnea as well as anemia (Hct <30) are especially at risk. Because postoperative apnea is highest in the first 4 to 6 hours and may present up to 12 hours after surgery, monitoring of all infants (38 weeks GA) that have not yet reached 44 weeks PCA and healthy preterm infants (<38 weeks) that have not reached 50 weeks PCA are admitted for overnight monitoring. See also question 631 (Motoyama: Smith's Anesthesia for Infants and Children, ed 7, pp 24-25, 562-563, 875- 876, 1163-1164).
question
492. A 72-year-old male undergoes emergency repair of an abdominal aortic aneurysm. In the first hour after release of the suprarenal cross-clamp, urine output is only 10 mL. After administration of furosemide 20 mg IV, urine output increases to 100 mL/hr. Urine [Na+] is 43 mEq/L and urine osmolality is 210 mOsm/L. The most likely cause of the initial oliguria is A. Fluoride toxicity B. Renal hypoperfusion C. Acute tubular necrosis D. Increased ADH E. Impossible to differentiate
answer
492. (E) In the absence of diuretics, oliguria associated with urine sodium concentration greater than 40 mEq/L and urine osmolality less than 400 mOsm/L is strongly suggestive of intrinsic renal disease (e.g., acute tubule necrosis) whereas prerenal causes have urine sodium concentration less than 20 mEq/L and urine osmolality greater than 400 mOsm/L. Furosemide, mannitol and dopamine, however, obscures the accurate diagnosis (Hines: Stoelting's Anesthesia and Co-Existing Disease, ed 5, pp 325-327; Stoelting: Basics of Anesthesia, ed 5, pp 430-431).
question
493. A healthy 25-year-old man is anesthetized for a sagittal split osteotomy. Anesthesia is induced with propofol, morphine, and vecuronium and maintained with 1.5% isoflurane and 50% N2O. After induction, the nose is prepped with 4% lidocaine and 1% phenylephrine, and the patient is intubated through the right naris. Before emergence, the surgeon performs a bilateral inferior alveolar nerve block. The patient is reversed with neostigmine and glycopyrrolate. When the patient awakens he is noted to have an 8-mm pupil on the right and a 3-mm pupil on the left. Results of physical examination are otherwise unremarkable. The most likely explanation for the dilated pupil is A. Right stellate ganglion block B. Accidental introduction of lidocaine into right eye C. Accidental introduction of phenylephrine into right eye D. Right ciliary ganglion block E. Glycopyrrolate
answer
493. (C) In an unconscious patient, a unilateral dilated pupil would be a matter of grave concern. In an awake patient with a normal neurologic examination, however, it is less worrisome. An inferior alveolar nerve block involves injection of about 2 mL of 2% lidocaine around the inferior alveolar nerve just behind the molars in the lower jaw. Even a grossly misdirected needle probably could not reach the stellate ganglion, but were it possible, the result would be a Horner's syndrome (miosis, not mydriasis, ptosis, anhidrosis, and vasodilation over the face). Blockade of the ciliary ganglion could cause mydriasis on the ipsilateral side, but reaching the ciliary ganglion, located between the optic nerve and lateral rectus muscle about 1 cm from the posterior limit of the orbit, would be almost impossible with a needle directed toward the mandible. Glycopyrrolate administered systemically does not cause mydriasis, as it is not capable of crossing the blood-brain barrier. Lidocaine instilled directly into the eye does not produce mydriasis, but phenylephrine does. Care must be taken not to spray local anesthetic (with or without vasoconstrictor) into the eyes while applying topical anesthesia to the nares (Stoelting: Pharmacology and Physiology in Anesthetic Practice, ed 4, p 304).
question
494. A 40-year-old male is undergoing a left inguinal hernia repair under general anesthesia in San Diego, Calif. N2O is administered at 3 L/min, O2 at 1 L/min, and isoflurane at 0.85%. What minimum alveolar concentration (MAC) is this patient receiving? A. 0.8 B. 1.25 C. 1.50 D. 1.75 E. 2.0
answer
494. (C) MAC is the minimum alveolar concentration of anesthetic that will prevent movement of 50% of patients when a skin incision is made at sea level (e.g., San Diego). MAC × 1.3 will prevent movement in 95% of patients. In this question, total gas flow is 4 L/min (1 L/min + 3 L/min). Roughly 75% of the total gas is N2O. The MAC of N2O is 104%. The patient is receiving about 0.75 MAC N2O. The MAC for isoflurane is 1.15. A concentration of 0.85% would represent 0.75 MAC. Because MACs are additive, the total MAC would be 1.5 (Barash: Clinical Anesthesia, ed 5, pp 397-398; Stoelting: Basics of Anesthesia, ed 5, p 82).
question
495. An otherwise healthy 140-kg, 24-year-old male is scheduled for thyroid surgery under general anesthesia. Which of the following statements concerning his cardiac output at 140 kg compared with his cardiac output at his ideal body weight (70 kg) is correct? A. Cardiac output is diminished by factor of 2 B. Cardiac output is diminished by 10% C. Cardiac output is the same D. Cardiac output is increased by 10% E. Cardiac output is doubled
answer
495. (E) Cardiac output increases by about 100 mL/min for each kilogram of weight gained. It is estimated that every (E) kilogram of adipose tissue contains nearly 3000 m of additional blood vessels. The additional cardiac output is due to ventricular dilation and increased stroke volume, as resting heart rates are not increased in obese patients (Hines: Stoelting's Anesthesia and Co-Existing Disease, ed 5, p 302; Stoelting: Basics of Anesthesia, ed 5, p 448).
question
496. Fenoldopam may be used as an alternative to which of the following? A. Epinephrine B. Phenylephrine C. Dopexamine D. Dopamine E. Sodium nitroprusside
answer
496. (E) Fenoldopam (Corlopam) is a selective dopamine-1 receptor agonist with significant vasodilating properties. It has moderate affinity for ?2-receptors but has no affinity for dopamine-2, ?1, ?, 5-HT1, or 5-HT2 receptors. It is used for treatment of patients with severe hypertension (especially with reduced renal function) and is administered as an IV infusion. It can be used as an alternative to sodium nitroprusside and has the advantage of no thiocyanate toxicity, rebound effect, or "coronary steal" effect, but it does contain sodium bisulfite and is contraindicated in patients with a known sulfite sensitivity. Dopexamine (Dopacard) is a synthetic analogue related to dopamine with intrinsic activity at dopamine as well as ?2-receptors and is used as an inotropic agent (Hardman: Goodman ; Gilman's The Pharmacologic Basis of Therapeutics, ed 10, p 227; Miller: Anesthesia, ed 6, pp 649-650; Stoelting: Pharmacology and Physiology in Anesthetic Practice, ed
question
497. A 58-year-old hemophiliac is scheduled for total knee arthroplasty. His factor VIII levels are 35% of normal. Which of the following would be the most appropriate therapy before surgery? A. Administer sufficient cryoprecipitate to raise factor VIII levels to 50% normal B. Administer sufficient factor VIII concentrate and platelets to raise levels to 50% normal C. Transfuse fresh frozen plasma until factor VIII levels are 100% normal D. Administer factor VIII concentrates until levels are 100% normal E. None of the above
answer
497. (D) Ideally, factor VIII levels should be raised to 100% predicted before elective surgery to ensure that the levels will not fall below 30% intraoperatively. Thirty percent of the normal factor VIII concentration or greater is thought to be necessary for a patient who is to undergo major surgery. Elimination half-time of factor VIII is 12 hours. This may be accomplished with factor VIII concentrate or cryoprecipitate. Fresh frozen plasma is no longer considered therapy for hemophilia (Hines: Stoelting's Anesthesia and Co-Existing Disease, ed 5, pp 419-420).
question
498. A 16-year-old boy whose maternal uncle has hemophilia A is scheduled for wisdom tooth extraction. Which test below would be the best screening test for hemophilia A? A. Partial thromboplastin time (PTT) B. Prothrombin time (PT) C. Thrombin time D. Platelet count E. Bleeding time
answer
498. (A) Hemophilia A is associated with decreased levels of factor VIII. PTT tests the intrinsic coagulation cascade (A) and would be abnormally elevated in all but the most mild disease. A normal PTT is 25 to 35 seconds. Platelet count PT and bleeding times are normal (Hines: Stoelting's Anesthesia and Co-Existing Disease, ed 5, p 420; Kasper: Harrison's Principles of Internal Medicine, ed 16, pp 680-681).
question
499. The reason four twitches are used in the train-of-four to determine degree of neuromuscular blockade versus five (or more) is A. Comparison of greater than four twitches is too difficult B. Four twitches informs the user of the degree of blockade in the useful clinical range (i.e., 75% to 100% blockade) C. Post-tetanic facilitation will begin to appear after four twitches D. Additional twitches may damage the nerve by overstimulation E. There would be no additional decrement in twitch height after four twitches
answer
499. (E) Conventional peripheral nerve stimulators deliver four twitches at 2 Hz spaced 0.5 second apart. These devices were designed with the knowledge that successive twitches deplete acetylcholine stores. After the fourth twitch, there is no additional decrement in twitch height (Stoelting: Basics of Anesthesia, ed 5, p 150).
question
500. A 57-year-old male is undergoing a right hemicolectomy under general anesthesia. The patient has no history of cardiac disease. During the operation 5-mm ST-segment elevation is noted on lead II and the patient develops complete heart block. The coronary artery most likely affected is A. Circumflex coronary artery B. Right coronary artery C. Left main coronary artery D. Left anterior descending coronary artery E. Branch to obtuse margin
answer
500. Inferior ischemia is associated with blockage or spasm of the right coronary artery. The right coronary artery (B) supplies blood to the atrioventricular node in 90% of patients. Complete heart block therefore is not unexpected in patients with severe CAD involving the right coronary artery (Hines: Stoelting's Anesthesia and Co-Existing Disease, ed 5, p 19).
question
501. Each of the following may increase MAC for volatile anesthetics EXCEPT A. Cocaine B. Hyperthyroidism C. Monoamine oxidase (MAO) inhibitor therapy D. Tricyclic antidepressants E. Hypernatremia
answer
501. (B) Minimum alveolar concentration (MAC) is influenced by a variety of disease states, conditions, drugs, and (B) other factors. Drugs that increase CNS catecholamines, such as MAO inhibitors, tricyclic antidepressants, acute amphetamine ingestion, and cocaine, increase MAC. Other factors that increase MAC include hyperthermia, hypernatremia, patients with natural red hair, and infancy. It is interesting that MAC values are higher for infants than for neonates or older children and adults. Thyroid gland dysfunction including hyperthyroidism does not affect the MAC. Factors that lower MAC include narcotics, IV anesthetics, local anesthetics (except cocaine) and other sedatives, age (6% per decade), hypothermia, hypoxia, severe anemia (e.g., Hgb < 5). The following table modified from the references in this question summarizes the impact of various factors on MAC (Barash: Clinical Anesthesia, ed, 5 pp 397-398; Morgan: Clinical Anesthesiology, ed 4, p 165; Stoelting: Basics of Anesthesia, ed 5, p 83
question
502. A 37-year-old patient with history of manic-depressive illness is scheduled to undergo surgery for removal of an intramedullary rod in the left tibia. Which of the following statements regarding potential untoward effects of lithium therapy is NOT true? A. Long-term administration may be associated with nephrogenic diabetes insipidus B. Administration of succinylcholine to patients treated with lithium may result in hyperkalemia C. Long-term therapy may be associated with hypothyroidism D. Duration of action of pancuronium may be prolonged E. Administration of thiazide diuretics may increase plasma lithium concentrations
answer
502. (B) Long-term lithium therapy in patients with manic-depressive illness may be associated with nephrogenic (B) diabetes insipidus. Hypothyroidism may develop in about 5% of patients because lithium can inhibit the release of thyroid hormones. Lithium is almost 100% renally excreted. Reabsorption occurs at the proximal convoluted tubule and is inversely related to the concentration of sodium in the glomerular filtrate. Consequently, administration of diuretics (mainly thiazide, but to a lesser extend loop diuretics) may lead to the development of toxic lithium levels. Lithium has sedative properties and may reduce the need for IV and inhalational anesthetic agents. It may prolong the duration of action of both pancuronium and succinylcholine, but it is not associated with an exaggerated release of potassium when succinylcholine is administered (Hardman: Goodman & Gilman's The Pharmacological Basis of Therapeutics, ed 10, pp 508-509; Hines: Stoelting's Anesthesia and Co-Existing Disease, ed 5, p 539).
question
503. Treatment of hypotension in a patient anesthetized for resection of metastatic carcinoid would be best accomplished with A. Epinephrine B. Ephedrine C. Vasopressin (DDAVP) D. Angiotensin E. Octreotide
answer
503. (E) Carcinoid tumors can arise wherever enterochromaffin cells are present. Most (>70%) originate in the intestine (E) and about 20% originate in the lung. Of those that originate in the gastrointestinal tract, 50% occur in the appendix, 25% in the ileum, and 20% in the rectum. These interesting tumors were called carcinoid because they were originally believed not to metastasize. We now know this is not true. The hormones released by the nonmetastatic tumors reach the liver by the portal vein and are rapidly inactivated. However, once metastases reach the liver, the released hormones reach the systemic circulation and produce signs and symptoms of the "carcinoid syndrome." Symptoms include cutaneous flushing, abdominal pain, vomiting, diarrhea, hypotension or hypertension, bronchospasm, and hyperglycemia. The natural hormone somatostatin suppresses the release of serotonin and other vasoactive substances from the tumor. Because the half-life is about 3 minutes, somatostatin is given by infusion. Octreotide is a synthetic somatostatin analogue with a half-life of 2.5 hours and is given SQ or IV for the prevention and treatment of carcinoid symptoms (e.g., hypotension, hypertension, bronchospasm). However, the treatment of hypotension in patients with carcinoid disease is different because ephedrine, epinephrine, and norepinephrine can release vasoactive hormones from the tumor and make the hypotension worse. Hypotension is best treated with fluids and IV octreotide or somatostatin. Hypertension is treated with deepening the anesthetic and administering octreotide, somatostatin or labetalol. Bronchospasm is treated with IV octreotide, somatostatin or nebulized ipratropium. When giving anesthesia to these patients it is probably wise to avoid drugs that release histamine and other vasoactive hormones that may precipitate symptoms. Propofol or etomidate are good induction agents, followed by maintenance anesthesia with a volatile anesthetic (e.g., isoflurane, sevoflurane or desflurane) and/or nitrous oxide with oxygen. Vecuronium, cisatracurium and rocuronium appear to be safe muscle relaxants. Fentanyl, sufentanil, alfentanil and remifentanil and benzodiazepines are also safe to use. The serotonin antagonist ondansetron is a useful antiemetic (Barash: Clinical Anesthesia, ed 5, pp 1058-1059; Hines: Stoelting's Anesthesia and Co-Existing Disease, ed 5, pp 289-291; Physicians Desk Reference-2009, ed 63, pp 2300- 2306).
question
504. A 75-year-old male patient is scheduled to undergo elective orchiectomy for prostate cancer. The patient has selected spinal anesthesia. What is the minimum dermatomal level that must be achieved to carry out this operation? A. T1 B. T4 C. T10 D. L3 E. S1
answer
504. (C) Testicular innervation can be traced up to the T10 dermatomal level. For this reason, any operation that (C) involves manipulation or traction on the testicles must have adequate anesthesia to prevent pain. This can be achieved with spinal or epidural anesthesia, which is associated with a T10 level of blockade (Barash: Clinical Anesthesia, ed 5, p 701).
question
505. A 31-year-old patient has been in the ICU on a ventilator for 24 hours after a motor vehicle accident. The patient does not open his eyes to any stimulus and has no verbal or motor response. The Glasgow Coma Scale corresponding to this patient would be A. 0 B. 1 C. 2 D. 3 E. 4
answer
505. (D) The Glasgow Coma Scale has three categories: eye opening for which a maximum of 4 points can be (D) received; best verbal response for a maximum of 5 points; and best motor response for a maximum of 6 points. The higher the score the better the response, the minimal score for each category is 1. Mild head injury scores are 13 to 15, moderate are 9 to 12 and severe are 3 to 8. This severe head-injured patient is totally unresponsive and would receive a score of 3 (Barash: Clinical Anesthesia, ed 5, p 782).
question
506. Hypoglycemia is more likely to occur in the diabetic surgical patient with which of the following diseases? A. Renal disease B. Rheumatoid arthritis requiring high-dosage prednisone C. Chronic obstructive lung disease treated with a terbutaline inhaler and aminophylline D. Manic-depressive disorder treated with lithium E. Congestive heart failure
answer
506. (A) Insulin metabolism involves both the liver and kidneys. Renal dysfunction, however, has a greater impact on (A) insulin metabolism than does hepatic dysfunction. In fact, unexpected prolonged effects of insulin sometimes are seen in patients with renal disease (Stoelting: Pharmacology and Physiology in Anesthetic Practice, ed 4, p 478).
question
507. Which of the following is most likely to be associated with a falsely elevated SaO2 as measured by pulse oximetry? A. Hemoglobin F B. Carboxyhemoglobin C. Bilirubin D. Fluorescein dye E. Methylene blue dye
answer
507.(B) Most pulse oximeters illuminate tissue with two wavelengths of light; 660-nm red light and 940-nm infrared (B) light. Since carboxyhemoglobin has an absorbance at 660 nm, very similar to O2 hemoglobin, it produces a falsely elevated SaO2 when present in the blood. Hemoglobin F, bilirubin, and fluorescein dye have no effect on pulse oximetry. Methylene blue, as well as indigo carmine and indocyanine green, lowers the SaO2 as measured by pulse oximetry. Methemoglobin absorbs red and infrared light equally well and gives saturation readings of 85% (Barash: Clinical Anesthesia, ed 5, pp 672, 1280; Stoelting: Basics of Anesthesia, ed 5, pp 312-313).
question
508. The most sensitive test for detecting primary hypothyroidism in the preoperative evaluation of a patient in whom hypothyroidism is suspected is A. Thyroid-stimulating hormone (TSH) level B. Total plasma T3 level C. Total plasma T4 level D. Resin T3 uptake E. Antithyroid antibodies
answer
508.(A) Hypothyroidism occurs when there is a decreased amount of the circulating thyroid hormones (T4, T3, or (A) both). It is estimated to occur in about 0.5% to 0.8% of the adult population. There are many causes of hypothyroidism, and they are classified into two main groups. The most common group (95% of cases) is primary hypothyroidism, where the thyroid gland is not capable of making enough hormones despite an adequate amount of TSH (thyrotropin). Secondary hypothyroidism (5% of cases) occurs when there is CNS dysfunction, i.e., hypothalamic or pituitary disease and low levels of TSH. It is the free and not total T3 or T4 level that parallels thyroid status. The T3 or T4 resin uptake test is used to measure free amounts of T3 or T4, but it is the TSH level that is used to differentiate primary from secondary hypothyroidism. Although antithyroid antibodies such as thyroid antimicrosomal antibody are seen in 95% of patients with multinodular diffuse goiter of Graves' disease, the most common type of hyperthyroidism, it also is seen in about 10% of adults with no thyroid disease (Barash: Clinical Anesthesia, ed 5, pp 1130-1131; Hines: Stoelting's Anesthesia and Co-Existing Disease ed 5, pp 384-386; Miller: Anesthesia, ed 6, pp 1047-1048).
question
509. Gabapentin (Neurontin) as used in the treatment of chronic pain belongs to the same broad class of drugs as A. Carbamazepine B. Imipramine C. Clonidine D. Aspirin E. Fluoxetine (Prozac)
answer
509. (A) Gabapentin, an anticonvulsant, was developed to be a centrally active GABA agonist but does not appear to (A) interact with GABA receptors. Its mechanism for producing analgesia is unclear, but it may involve inhibition of voltage-activated calcium channels as well as potentiating GABA release. Carbamazepine slows the recovery rate of voltage-gated sodium channels, but it also is an anticonvulsant. Carbamazepine is indicated in the treatment of trigeminal neuralgia (Stoelting: Pharmacology and Physiology in Anesthetic Practice, ed 4, pp 571-575; Loeser: Bonica's Management of Pain, ed 3, pp 106, 1728-1730).
question
510. Which of the following medical conditions would not prompt the administration of prophylactic antibiotics for this 67-year-old patient scheduled to undergo a right hemicolectomy? A. A prosthetic cardiac valve B. Mitral valve prolapse C. Cardiac transplant recipient with cardiac valvulopathy D. Previous history of treated infective endocarditis E. All need prophylactic antibiotics
answer
510. (B) In 2007, the American Heart Association revised the guidelines for "Prevention of Infective Endocarditis." (B) Patients with mitral valve prolapse (MVP) do not need prophylaxis. Only patients in the highest risk groups, such as prosthetic cardiac valves, previous infective endocarditis, several types of congenital heart disease and cardiac transplantation recipients who develop cardiac valvulopathy need prophylactic antibiotics for elective GI or GU procedures (see also question 915) (Wilson W, Taubert KA, Gewitz, et al.; Prevention of infective endocarditis - Guidelines from the American Heart Association. Circulation, 2007; 115:1736-1754.
question
511. A 47-year-old morbidly obese patient develops bilateral blindness (only able to perceive light) after a 6 hour, 3 segment laminectomy and fusion. The patient received 6 units of blood and 5 L of Lactated Ringer's solution. A mean arterial blood pressure was maintained at 50 to 60 mm Hg. The most likely structure involved in this visual loss is A. Central retinal artery B. Optic nerve C. Retina D. Cerebral cortex E. Central retinal vein
answer
511. (B) Perioperative visual loss associated with non-ocular surgery is rare and may result from corneal trauma, retinal artery occlusion, retinal vein occlusion, optic nerve ischemia or cortical disease. Although overall a rare problem, it may develop in up to 1% of prone spinal surgical cases and is most commonly due to ischemic optic neuropathy. The cause is unknown and multifactorial. Associated factors include prolong intraoperative hypotension, anemia (Hgb ; 8), large intraoperative blood loss, prolonged surgery and facial edema. It is more common in males and in patients with peripheral vascular disease, diabetes mellitus, and tobacco users (Lobato: Complications in Anesthesiology pp 418-422; Miller: Anesthesia, ed 6, pp 3005-3011).
question
512. All of the following concerning postoperative shivering are true EXCEPT A. May increase metabolism and oxygen consumption significantly B. May be treated with meperidine C. May be treated with droperidol D. May be treated with clonidine E. Does not occur in the absence of hypothermia
answer
512. (E) Postoperative shivering or postanesthetic tremor can occur during recovery from all types of general anesthesia. If profound, shivering can increase metabolic rate and O2 consumption (100%-200%) with an associated increase in cardiac output and minute ventilation. Although shivering usually occurs in patients with decreased body temperature, it also may occur in patients with normal body temperature after anesthesia. Postanesthesia shivering is best treated by a combination of supplemental oxygen, rewarming the patient and/or administering IV meperidine. Other less frequently used pharmacologic treatments include clonidine, magnesium sulfate, calcium chloride, chlorpromazine, droperidol, and other opioids (e.g., butorphanol). Application of radiant heat to the face, head, neck, chest, and abdomen has been shown to eliminate shivering within minutes in postoperative patients, despite low core body temperatures (Longnecker: Principles and Practice of Anesthesiology, ed 2, pp 2318-2319; Morgan Clinical Anesthesiology, ed 4, pp 771- 772; Stoelting: Basics of Anesthesia, ed 5, pp 574-575).
question
513. Electrocardiographic (ECG) changes associated with hyperkalemia include A. Increased P wave amplitude B. Shortened PR interval C. Narrowed QRS complex D. Narrowed and peaked T waves E. Increase in U-wave amplitude
answer
513. (D) The ECG signs of hyperkalemia include narrowed and peaked T waves (earliest manifestation of hyperkalemia), decrease in P wave amplitude, prolonged PR interval, and a widened QRS interval. In extreme cases, the ECG can appear as a sine wave as well as cardiac arrhythmias (e.g., sinus arrest, supraventricular tachycardia, atrial fibrillation, PVCs, ventricular tachycardia and ventricular fibrillation). These changes are potentiated by hypocalcemia and intravenous calcium can rapidly correct some of these ECG changes. An increase in U-wave amplitude suggests hypokalemia, not hyperkalemia (Miller: Anesthesia, ed 6, p 1106).
question
514. A 24 year old is undergoing open reduction of an ankle fracture under general anesthesia with sevoflurane, N2O and O2 through an laryngeal mask airway (LMA). Just after the vaporizer dial is turned up to 2% the patient begins spontaneously breathing, but the inspiratory valve is not fully closing. The likely result of this (malfunctioning valve) is an increased in the inspired concentration of A. N2O B. CO2 C. O2 D. Sevoflurane E. All of the above
answer
514. (B) If the inspiratory valve becomes stuck in the open position, it will "malfunction" only during exhalation since, during inhalation, it is supposed to be open. During the exhalation phase of breathing, exhaled gases will exit through the expiratory valve into the expiratory limb of the circuit and beyond (proper path) as well as through the inspiratory valve into the inspiratory limb of the circuit (errant path). Gases traveling into the inspiratory limb (old gas) will be returned to the patient with next breath. The volume of recently exhaled gas is now drawn back into the patient's lungs along with the "new" gas that would be inspired in a fully functional breathing circuit. The net effect is that oxygen, sevoflurane, and N2O will all be diluted, but the patient rebreathes CO2, thus it will be the only gas with an increased inspired concentration (normal inspired CO2 is zero) as a result of the stuck inspiratory valve (Barash: Clinical Anesthesia, ed 5, p 580).
question
515. All of the following are associated with acromegalic patients undergoing transsphenoidal hypophysectomy EXCEPT A. Enlargement of the tongue and epiglottis B. Narrowing of the glottic opening C. Nasal turbinate enlargement D. 20% to 30% incidence of difficult intubation E. Continuous positive airway pressure (CPAP) should be used postoperatively since obstructive sleep apnea (OSA) is very common
answer
515. (E) Enlargement of the tongue and epiglottis predisposes the patient to upper airway obstruction and makes visualization of the vocal cords more difficult. The vocal cords are enlarged, making the glottic opening narrower. In addition, subglottic narrowing may be present as well as tracheal compression from an enlarged thyroid (seen in about 25% of acromegalic patients). This often necessitates the use of a narrower endotracheal tube than one might choose based on the facial enlargement. The placement of nasal airways may be more difficult due to the enlarged nasal turbinates. The use of CPAP is contraindicated after transsphenoidal hypophysectomy (Barash: Clinical Anesthesia, ed 5, pp 776, 1149; Fleisher: Anesthesia and Uncommon Diseases, ed 5, pp 21-22; Hines: Stoelting's Anesthesia and Co-Existing Disease, ed 5, pp 402- 403).
question
516. Evidence of an anaphylactic reaction to atracurium 1 to 2 hours after the episode could be best established by measuring blood levels of A. Tryptase B. Laudanosine C. Histamine D. Bradykinin E. Cortisol
answer
516. (A) There are four types of immune mediated allergic reactions. Anaphylaxis is a Type I IgE mediated reaction (A) that involves mast cells and basophils. Anaphylactoid reactions appear like anaphylaxis but are not immune mediated. Tryptase is a neutral protease normally stored in mast cells but is released into systemic circulation during anaphylactic but not anaphylactoid reactions. Tryptase levels would need to be measured within 1 to 2 hours of the suspected allergic reaction. Plasma histamine levels return to baseline within 30 to 60 minutes of an anaphylactic reaction. Laudanosine is a normal metabolic product of atracurium metabolism (Hines: Stoelting's Anesthesia and Co-Existing Disease, ed 5, p 526; Morgan: Clinical Anesthesiology, ed 4, p 972).
question
517. Which of the following findings is NOT consistent with a diagnosis of malignant hyperthermia? A. PaCO2 150 mm Hg B. MVO2 50 mm Hg C. pH 6.9 D. Arterial oxygen saturation 85% on 100% FIO2 E. Onset of symptoms an hour after end of operation
answer
517. (B) Signs of MH reflect the hypermetabolic state (up to 10 times normal) that develops. Clinical signs include (B) tachycardia, tachypnea, arterial hypoxemia, hypercarbia (e.g., PaCO2 100-200 mm Hg), metabolic and respiratory acidosis (e.g., pH 6.80-7.15), hyperkalemia, hypotension, muscle rigidity, trismus after succinylcholine administration, and increased body temperature. Mixed venous oxygen tension would be very low. The clinical presentations are quite variable and some reactions may not develop until the postoperative period (Hines: Stoelting's Anesthesia and Co-Existing Disease, ed 5, pp 620-622).
question
518. A 52-year-old business executive undergoes a radical retropubic prostatectomy uneventfully under general isoflurane anesthesia. He takes fluoxetine (Prozac) for depression. Upon discharge, which of the following analgesics would be the best choice for post-operative pain management in this patient? A. Oxycodone plus aspirin (Percodan) B. Hydrocodone with acetaminophen (Vicodin) C. Codeine with acetaminophen (Tylenol #3) D. Hydromorphone (Dilaudid) E. All would be equally effective
answer
518. (D) The selective serotonin reuptake inhibitor (SSRI) fluoxetine, is one of the most potent inhibitors of the (D) cytochrome P-450 enzymes CYP3A4 and CYP2D6. CYP2D6 facilitates the conversion of codeine to morphine, meaning the response from a "normal" dose would be less than expected because of decreased conversion. Oxycodone and hydrocodone are metabolized by CYP2D6 to their active form as well and a "normal" dose of these would give less response than expected. Thus, codeine, oxycodone and hydrocodone would be poor analgesic choices for patients taking SSRIs. CYP3A4 is responsible for the metabolism of fentanyl, sufentanil and alfentanil. Remifentanil is metabolized by non-specific plasma esterases (Miller: Anesthesia, ed 6, p 101).
question
519. Anesthesia is induced in a 50-year-old, 125-kg man for anterior cervical fusion. The patient is placed on a ventilator. Peak airway pressure is noted to be 20 cm H2O with O2 saturation 99% on pulse oximeter. An hour later, the peak airway pressure rises to 40 cm H2O, PaCO2 is 38 mm Hg on mass spectrometer, on O2 saturation falls to 88%. Blood pressure and heart rate are unchanged. The most likely cause of these findings is? A. Mainstem intubation B. Thrombotic pulmonary embolism C. Tension pneumothorax D. Venous air embolism E. Laboratory error
answer
519. (A) Symptoms of a mainstem or bronchial intubation include asymmetric chest expansion, unilateral breath (A) sounds, elevation of peak airway pressures, and arterial blood gas abnormalities (e.g., hypoxemia). Frequently, bronchial intubation is intentional (e.g., thoracic surgery with double lumen endotracheal tubes) but if undetected with a single lumen tube, atelectasis, hypoxia and pulmonary edema may result in time. Peak airway pressures can also increase with many conditions such as airway obstruction (e.g., kinked endotracheal tube, secretions, overinflated cuffs), bronchospasm, increasing VT, increase in chest wall muscle tone (rigid chest with narcotics, coughing), and tension pneumothorax. If a tension pneumothorax develops, associated hypotension usually is present. Pulmonary embolism would not cause the peak airway pressure to rise as in this case (Lobato: Complications in Anesthesiology, pp 101-102).
question
520. The phase of liver transplantation where the greatest degree of hemodynamic instability is expected is A. Induction B. Dissection phase C. Anhepatic phase D. Reperfusion phase E. Emergence
answer
520. (D) Although hemodynamic instability can occur at any time during liver transplantation, it is during the initial part (D) of the reperfusion phase, when the vascular clamps are removed from the liver graft, when cardiovascular instability is most marked. At this time there can be profound hypotension, reduced cardiac contractility, cardiac arrhythmias as well as hyperkalemic cardiac arrest. Epinephrine, atropine, calcium, and sodium bicarbonate should be available, as well as blood products, during this critical part of the surgery (Miller: Anesthesia, ed 6, pp 2249-2252; Stoelting: Basics of Anesthesia, ed 5, p 534).
question
521. Which of the following drugs is (are) likely to prolong nondepolarizing neuromuscular blockade? A. Prednisone B. Diltiazem C. Clindamycin D. Dantrolene E. All of the above
answer
521. (E) Metabolic and physiologic conditions as well as certain medications can contribute to a prolonged duration of (E) action of nondepolarizing neuromuscular blockade. Metabolic and physiologic conditions include respiratory acidosis, myasthenia syndromes, hepatic/renal failure, hypocalcemia, hypothermia, and hypermagnesemia. Both inhaled and local anesthetics as well as corticosteroids, many antibiotics (e.g., polymyxins, aminoglycosides, lincosamines [e.g., clindamycin], metronidazole [Flagyl]), calcium channel blockers, dantrolene and furosemide can prolong nondepolarizing neuromuscular blockade (Stoelting: Basics of Anesthesia, ed 5, p 566).
question
522. Drugs considered suitable for patients who are susceptible to malignant hyperthermia MH include all EXCEPT A. Etomidate B. N2O C. Calcium chloride D. Ketamine E. All of the above are safe
answer
522. (E) Drugs considered unsafe for patients susceptible to MH include all of the volatile anesthetics and the (E) depolarizing muscle relaxant succinylcholine. All local anesthetics (both amide and ester), N2O, opiates, barbiturates, ketamine, propofol, vasoactive drugs, calcium salts, antibiotics, antihistamines, nondepolarizing muscle relaxants, as well as drugs used to reverse nondepolarizing muscle relaxants, are safe (i.e., nontriggering) in patients susceptible to MH (Barash: Clinical Anesthesia, ed 5, pp 533-534; Miller: Anesthesia, ed 6, p 1182).
question
523. Near the end of a three-hour colectomy the surgeon complains that the patient is not relaxed. Two twitch monitor placed at different locations show only one twitch of a train of four. Blood gases are reported to be pH 6.9, CO2 82, K 4.6 and acetate 4.6. Most appropriate action would be A. Administer more vecuronium B. Administer bicarbonate C. Administer succinylcholine D. Increase minute ventilation E. Administer dantrolene
answer
523. (E) Rare muscle diseases can have dramatic anesthetic implications. Malignant hyperthermia (MH) is among the (E) most important manifestations of a muscular disorder. Malignant hyperthermia is thought to be caused by alterations in calcium control in muscle sarcoplasmic reticulum in response to succinylcholine or potent volatile anesthetics (most likely mediated by mutations of the ryanodine receptor). Because MH is a disorder in muscle metabolism, rigidity during administration of a volatile anesthetic or after succinylcholine use may be the presenting sign. Additionally, administration of any muscle relaxant would not provide muscle relaxation and succinylcholine would be contraindicated. The patient does have a respiratory and metabolic acidosis and significantly increasing minute ventilation with 100% oxygen and the use of sodium bicarbonate would be needed; however, stopping the triggering agent and administration of dantrolene is most important (Hines:
question
524. A 22-year-old parturient is anesthetized for an emergency laparoscopic cholecystectomy. She is in the 24th week of gestation and receives general sevoflurane anesthesia and has received rocuronium for muscle relaxation. Just prior to emergence, muscle relaxation is reversed with glycopyrrolate and neostigmine. Three minutes later, the fetal heart rate falls to 88 beats per minute. The most likely cause of this is A. Fetal head compression B. Maternal hypoxia C. Fetal hypoxia D. Uteroplacental insufficiency E. Reversal agents
answer
524. (E) Atropine and scopolamine cross the placenta easily, whereas glycopyrrolate is poorly transferred across the (E) placenta. Although neostigmine crosses the placenta poorly, enough does cross the placenta and can cause fetal bradycardia in utero. That is why it is better to reverse muscle relaxants in pregnant patients for non-delivery surgery with neostigmine and atropine (Chestnut: Obstetric Anesthesia, ed 3, p 60).
question
525. Each of the following is associated with an increased risk of nausea and vomiting with anesthesia EXCEPT A. Smoking B. Female gender C. History of motion sickness D. History of postoperative nausea and vomiting E. Postoperative codeine
answer
525. (A) The overall estimated incidence of postoperative nausea and vomiting (PONV) for all surgical procedures is (A) 25% to 30%. Several factors are associated with an increased risk of PONV including patient related factors (e.g., a previous history of PONV, history of motion sickness, nonsmoking, female gender, pregnancy, obesity), certain surgical procedures (e.g., eye muscle surgery, ear, nose, and throat (ENT), dental), as well as postoperative pain and narcotic use (Lobato: Complications in Anesthesiology pp 571-573; Stoelting: Basics of Anesthesia, ed 5 p 575).
question
526. Ketorolac is contraindicated in patients undergoing scoliosis surgery because of A. Renal effects B. Risk of postoperative hemorrhage C. Effects on bone healing D. Effects on pulmonary function E. Paradoxical effect with bone pain
answer
526. (C) Ketorolac is one of the few nonsteroidal anti-inflammatory drugs (NSAIDs) approved for parenteral use. (C) Although NSAIDs have analgesic and anti-inflammatory effects without ventilatory depression; they also inhibit platelet aggregation, can produce gastric ulceration, are associated with renal dysfunction and may impair bone healing. Nonsteroidal anti-inflammatory drugs are contraindicated in patients undergoing spinal fusion, where bone healing is essential to a successful surgical procedure (Stoelting: Pharmacology and Physiology in Anesthesia Practice, ed 4, pp 276-282, 2872-2888).
question
527. Causes of sickling in patients with sickle-cell anemia include all of the following EXCEPT A. Inhaled nitric oxide B. Dehydration C. Metabolic acidosis D. Hypothermia E. Hypoxemia
answer
527.(A) Sickle-cell anemia is an inherited disease that affects approximately 0.3% to 1% of the black population in the (A) United States. Affected patients are homozygous for hemoglobin S such that 70% to 98% of the hemoglobin found in their RBCs is of the unstable S type, resulting in severe hemolytic anemia. Factors that favor the formation of sickle cells include arterial hypoxemia, acidosis, dehydration, and reductions in body temperature. Inhaled nitric oxide and other new investigational drugs may help reduce the sickling process and may even unsickle cells (Fleisher: Anesthesia and Uncommon Diseases, ed 5, pp 362-363; Hines: Stoeling's Anesthesia and Co-Existing Disease, ed 5, pp 411-412).
question
528. Which of the following factors is the greatest predictor of sleep apneas in an adult? A. Neck circumference B. Edentulousness C. Weight D. Body mass index E. Micrognathia
answer
528. (C)?? Although many books suggest that obesity is the most common cause of OSA, more recent data suggests (C) that a large neck circumference (>44 cm) reflects pharyngeal fat deposition and is more strongly correlated with OSA than obesity (BMI >30). Other risk factors include male gender, middle age, evening alcohol consumption or sleep-inducing medications (Hines: Stoelting's Anesthesia and Co-Existing Disease, ed 5, pp 299-301; Miller: Anesthesia, ed 6, p 1030; Stoelting: Basics of Anesthesia, ed 5, pp 411-412).
question
529. Greatest number of malpractice claims made against anesthesiologists (according to the ASA closed claims task force) is associated with which adverse outcome? A. Airway trauma B. Brain damage C. Nerve damage D. Death E. Eye injury
answer
529. (D) The ASA closed claims task force lists the leading causes of malpractice claims against anesthesiologists in (D) the 1990s to be death (22%), followed by nerve damage (21%), and brain damage (10%) (Barash: Clinical Anesthesia, ed 5, p 101).
question
530. Resynchronization therapy A. Is indicated for short QRS complexes B. Synchronizes QRS with breathing C. Requires pacemaker implantation D. Is usually accomplished with biphasic defibrillator E. Is contraindicated in patients with coronary artery disease
answer
530. (C) Cardiac resynchronization therapy (CRT) is used in patients with heart failure (EF ; 35%) and ventricular (C) conductive delay (prolonged QRS complex usually is 120-150 msec). The conduction delay creates a mechanical dyssynchrony and worsens the heart failure. Cardiac resynchronization therapy requires biventricular pacing with one lead in the coronary sinus to activate the LV. Cardiac resynchronization therapy has nothing to do with breathing. Although CRT has nothing to do with an ICD, many patients may require both as the typical scenario is a patient with poor LV function is also at risk for sudden death. Most of these patients also have underlying CAD (Hines: Stoelting's Anesthesia and Co-Existing Disease, ed 5, p 112; Miller: Anesthesia, ed 6, pp 1418-1419, 1427-1428).
question
531. The underlying feature in patients with syndrome X is A. Hypertension B. Coronary artery disease C. Hypoglycemia D. Insulin resistance E. Morbid obesity
answer
531. (D) Patients with syndrome X (also called metabolic syndrome X) have insulin resistance that leads to elevated (D) levels of insulin and the metabolic changes that occur with elevated insulin levels, except that hypoglycemia does not develop. Associated with it are low levels of high-density lipoproteins, hypertension, and increased plasminogen activator inhibitor-1 levels, which are associated with coronary artery disease. Many of these patients are obese (Miller: Anesthesia, ed 6, p 1777).
question
532. A 65-year-old hospitalized patient is being treated for pain from pancreatic cancer and is well-controlled on 30 mg IV morphine per day. What is the equivalent total oral daily dosage of morphine in this patient for discharge planning? A. 10 mg B. 30 mg C. 90 mg D. 120 mg E. 150 mg
answer
532. (C) The parenteral to oral conversion for morphine sulfate is 1:3, thus 30 mg morphine parenterally would be (C) similar to 30 mg × 3 = 90 mg of morphine orally. The parenteral to oral conversion for methadone is 1:2 (Hardman: Goodman ; Gilman's The Pharmacological Basis of Therapeutics, ed 11, p 606).
question
533. The effect of inhaled anesthetics is determined by A. The partial pressure of the anesthetic B. The percent of the anesthetic in the inspired gas C. Partial pressure for N2O and percent of inspired gas for volatiles D. Partial pressure for N2O and desflurane and percent inspired gas for all others E. Partial pressure only for agents with vapor pressure less than or equal to 250
answer
533. (A) It is the partial pressure of the inhalation anesthetic in the brain (Pbr) that determines activity (Stoelting: (A) Basics of Anesthesia, ed 5, pp 82-83).
question
534. Hazards of O2 administration include A. Retinopathy of prematurity B. Retention of CO2 C. Adsorption atelectasis D. Bronchopulmonary dysplasia E. All of the above
answer
534. Retinopathy of prematurity (retrolental fibroplasia) is a hazard associated with O2 administration to neonates (E) up to 44 weeks (gestational age + life age). It is especially a hazard in the extremely premature (birth weight less than 1000 g and gestational age less than 28 weeks). Bronchopulmonary dysplasia is a chronic lung disorder that afflicts infants who required mechanical ventilation at birth to treat respiratory distress syndrome. CO2 retention is a hazard in patients with chronic obstructive lung disease. Adsorption atelectasis is a potential hazard of oxygen administration in any patient receiving oxygen concentrations greater than 50%. It results from rapid uptake of oxygen into the circulation greater than the delivery of oxygen by ventilation. Normally, the presence of nitrogen serves as an internal splint, protecting the alveoli from collapse. Prolonged high concentration of oxygen can damage "normal lungs" if given for prolonged periods of time and may lead from mild irritation to tracheobronchitis to pulmonary interstitial edema to pulmonary fibrosis (Miller: Anesthesia, ed 6, pp 711, 716, 717, 2853; Morgan: Clinical Anesthesiology, ed 4, pp 1028-1029).
question
535. Which of the following nerves is NOT derived from a cranial nerve? A. Great auricular B. Infraorbital C. Supratrochlear D. Supraorbital E. Mental
answer
535. (A) All of the nerves listed in this question are derived from the fifth cranial nerve (trigeminal nerve) except the (A) great auricular nerve. The ophthalmic nerve (V1 branch of trigeminal nerve) gives rise to the supratrochlear, infratrochlear, and supraorbital nerves. The infraorbital nerve is a branch of V2 (maxillary branch of the trigeminal nerve). The mental nerve is a branch of V3 (mandibular nerve). The great auricular nerve arises from branches of C2 and C3 spinal nerves and innervates the skin of the outer ear, the mastoid process and the parotid gland (Miller: Anesthesia, ed 6, pp 1706-1707, 2548).
question
536. A 45-year-old female is experiencing progressive mental deterioration over a 6 hour period, 5 days after emergency evacuation of a large subarachnoid hemorrhage and clipping of a middle cerebral artery aneurysm. The most likely cause for deterioration is: A. Cerebral edema B. Hyponatremia C. Recurrent cerebral hemorrhage D. Vasospasm E. Improper placement of the aneurysm clip
answer
536. (D) Cerebral vasospasm is often associated in patients who have suffered a subarachnoid bleed. Angiographic (D) evidence of vasospasm can be noted in up to 70% of patients, however, clinical vasospasm with detectable ischemia (e.g., mental confusion, lethargy, focal motor and speech impairments) is detected in about 30% of patients. When clinical vasospasm develops, it usually occurs between 4 and 12 days after the bleed. Although it may resolve spontaneously, it may also progress to coma and death within a few hours or days. Rebleeding tends to occur earlier, i.e., within 24 hours (Barash: Clinical Anesthesia, ed 5, p 778).
question
537. Which of the following treatments should not be used in the management of thyrotoxicosis? A. Aspirin B. Cold crystalloid C. Cholestyramine D. Dexamethasone E. Esmolol
answer
537. (A) Thyroid storm (thyrotoxicosis) is a medical emergency with a 10% to 75% mortality rate. It usually occurs in (A) poorly controlled or undiagnosed Graves' disease patients. It may develop intraoperatively but more likely develops 6 to 18 hours after surgery. Symptoms include mental status changes, tachycardia, fever, cardiac arrhythmias. Treatment includes large doses of dexamethasone, which inhibits the synthesis, release and conversion of T4 to T3. Propylthiouracil inhibits thyroid hormone production. Sodium iodine is used to inhibit release of thyroid hormone from the thyroid gland. Propranolol helps to control the tachycardiac response and may inhibit conversion of T4 in the periphery. Acetaminophen is used to help control the temperature. Aspirin is contraindicated since it displaces thyroid hormones from thyroglobin and could aggravate the disease (Morgan: Clinical Anesthesiology, ed 4, pp 1016, 1017; Stoelting: Basics of Anesthesia, ed 5, p 443).
question
538. The most common adverse cardiac event in the pediatric population is A. Hypotension B. Bradycardia C. Premature ventricular contraction (PVC) salvos D. Bigeminy E. Tachycardia
answer
538. (B) The most common adverse cardiac event in pediatric population is bradycardia. An outcome study from the (B.) Medical College of Virginia examined the incidence of bradycardia in nearly 8000 children younger than 4 years old. The most common causes of bradycardia are cardiac disease or surgery and inhalation anesthesia, followed by hypoxemia. Of those children who had bradycardia, hypotension occurred in 30%, asystole or ventricular fibrillation in 10%, and death in 8%. Tachycardia, which is common, is not an adverse event (Motoyama: Smith's Anesthesia for Infants and Children, ed 7, p 1161).
question
539. Each of the following is a predictor of difficulty with mask ventilation EXCEPT A. Presence of beard B. BMI greater than 26 C. Presence of teeth D. Age greater than 55 E. History of snoring
answer
539. (C) Mask ventilation, one of the most basic anesthesia techniques, can be challenging in some patients. Patients (C) who are prone to airway obstruction can be more difficult due to extra airway tissue (i.e. obese patients with a BMI > 26), patients without teeth (i.e., tongue is closer to the roof of the mouth and face conformity may not fit the mask well), patients who snore (i.e., already have reason for airway obstruction). Mask ventilation can also be more difficult in patients who have a beard (i.e., harder to get a good mask seal), patients whose age is greater than 55, patients with facial tumors and patients with facial trauma. Use of an oral airway may be needed in many of these patients (Stoelting: Basics of Anesthesia, ed 5, p 215; Miller: Anesthesia, ed 6, pp 1623-1625).
question
540. In a patient with compartment syndrome, which of the following signs would be the last to appear? A. Pulselessness B. Pain C. Swelling D. Paralysis E. Paresthesia
answer
540. (A) Whenever perfusion to an extremity is inadequate (e.g., trauma or poor perfusion), hypoxic edema develops, (A) producing swelling. When this occurs in a compartment, tissue pressures rise, decreasing capillary perfusion. Symptoms of compartment syndrome include extreme pain unrelieved by analgesics, paresthesias, paralysis and pallor. Extensive rhabdomyolysis may develop as well as permanent nerve and muscle injury in the compartment. Because the problem is at the tissue level, pulses and capillary refill may still be present. Treatment includes fasciotomy to relieve the elevated pressure (Barash: Clinical Anesthesia, ed 5, pp 652- 653, 1279; Miller: Anesthesia, ed 6, pp 2478-2479).
question
541. Select the TRUE statement regarding the dose and duration, respectively, of local anesthetics for spinals in infants compared with adults A. Greater dose and longer duration B. Greater dose and shorter duration C. Greater dose and duration is the same D. Smaller dose and longer duration E. Smaller dose and shorter duration
answer
541. (B) The amount and distribution of CSF is different in neonates compared with adults. The neonate has about 4 mL/kg of CSF compared to the adult's 2 mL/kg. In addition, almost half of the neonate's CSF is in the spinal subarachnoid space, compared with about a quarter of the adult's CSF in the spinal subarachnoid space. These factors help explain why the dose is greater in neonates and infants and of shorter duration compared to adults (Motoyama: Smith's Anesthesia for Infants and Children, ed 7, p 467).
question
542. A No. 6 endotracheal tube indicates which size? A. 6 mm internal diameter B. 6 mm external diameter C. 6 mm external circumference D. 6 mm internal circumference E. 6 cm external radius
answer
542. (A) Endotracheal tube sizes are measured according to the internal diameter (ID). They are available in 0.5 mm ID increments (Stoelting: Basics of Anesthesia, ed 5, p 218).
question
543. If a patient were to become trapped in the MRI scanner by a metal object and the engineers decided to quench the magnet, the greatest hazard to the patient would be A. Heat B. Cold C. Fire D. Explosion E. Noise
answer
543. (B) Magnetic resonance imaging (MRI) scanners have superconducting electrical currents that produce large magnetic fields (up to 6 m) and are always "on". The presence of any ferromagnetic objects in the room may cause a missile-type injury when the objects are strongly attracted to the scanner. If a patient is pinned into the scanner by a magnetic object that flew into the scanner, the MRI technicians may have to turn off the superconducting magnet. During magnetic shutdown (quench) the scanner will become extremely cold (Stoelting: Basics of Anesthesia, ed 5, pp 553-554).
question
544. A 25-year-old black male is brought to the emergency room unconscious. Supplemental oxygen is administered and a pulse oximeter is placed on his finger and a reading of 98% is recorded. Arterial gas sampling at the same time shows PaO2 of 190 mm Hg, pH 7.2 and O2 saturation of 90%. Presence of which of the following could explain the discrepancies between these two readings? A. Methemoglobin (Hb Met) B. Sickle cell hemoglobin C. Carboxyhemoglobin (HbCO) D. Hemoglobin shifted to right E. Pulse oximeter error
answer
544. (C) Carbon monoxide is a colorless, odorless gas which binds to hemoglobin with an affinity over 200 times stronger than oxygen. Inhalation of CO is a major cause of morbidity and mortality in the United States. A dual wave (660 nm and 940 nm) pulse oximeter is incapable of distinguishing CO hemoglobin from oxyhemoglobin, but the distinction is easily made in the clinical laboratory with a co-oximeter. Significant quantities of methemoglobin would result in a saturation of 85% of the pulse oximeter. The slight right shift from a mild acidemia would be insufficient to account for 90% saturation in the face of a PaO2 of 190. Furthermore, the pulse oximeter reading would be nearly the same as the co-oximeter value (Miller: Anesthesia, ed 6, pp 1450- 1451; Hines: Stoelting's Anesthesia and Co-Existing Disease, ed 5, p 552).
question
545. During surgery for correction of scoliosis, somatosensory evoked potential (SSEP) monitoring is employed. An increase in SSEP latency and decrease in amplitude could be explained by each of the following EXCEPT A. Anterior spinal artery syndrome B. Ischemia of posterior tibial nerve C. Hypotension D. 2 MAC isoflurane anesthesia E. Propofol infusion (200 µg/kg/min)
answer
545. (A) The pathway for SSEP monitoring of the lower extremity starts with a stimulus of the posterior tibial nerve, (A) which generates an impulse that passes through the dorsal root ganglion into the dorsal (posterior) columns and then to the dorsal column nuclei. Second order nerves carry the impulse across the midline to the thalamus and the impulse travels over third order nerves to the sensory cortex of the brain. Electrodes in the scalp record the electrical activity in the brain. Severe hypotension or ischemia in any portion of the pathway along which the induced signal is conducted can result in a reduced evoked potential amplitude or increased latency. Volatile anesthetic administration in MAC values greater than 0.5 to 0.75 can produce a similar effect. Barbiturates, benzodiazepines, propofol and other sedative drugs can likewise interfere with SSEP monitoring. Anterior spinal artery syndrome affects the anterior (motor) portion of the spinal cord and does not interfere with SSEP monitoring (Stoelting: Basics of Anesthesia, ed 5, pp 313-314).
question
546. In which of the following conditions would the response to atropine be most pronounced? A. Diabetic autonomic neuropathy B. Brain death C. Status post heart transplant D. High (C8) spinal anesthesia E. A patient with chronic atrial fibrillation, complete heart block with VVI pacemaker
answer
546. (D) Diabetic autonomic neuropathy can affect the autonomic nervous system to such an extent that atropine and propranolol would have little effect (because there would be nothing to block). After heart transplantation, the new heart (donor heart) is denervated and will not respond to autonomic nervous system blocking drugs. Brain death by definition is associated with absence of autonomic function. A high spinal would be associated with total sympathectomy and propranolol would have no effect on heart rate, but the vagus nerve would be unaffected. Atropine would have no effect on a patient with atrial fibrillation and complete heart block (Hines: Stoelting's Anesthesia and Co-Existing Disease, ed 5, pp 22, 213, 375; Stoelting: Basics of Anesthesia, ed 5, pp 260, 438-439).
question
555. Decreased FEV1/FVC ratio 556. Decreased total pulmonary compliance 557. Increased total lung capacity 558. Decreased FRC 559. Decreased FEV1, normal FEV1/FVC ratio 560. Increased lung compliance due to loss of elastic recoil of the lung A. Pulmonary emphysema B. Chronic bronchitis C. Restrictive pulmonary disease D. Pulmonary emphysema and chronic bronchitis E. Pulmonary emphysema and restrictive pulmonary disease
answer
555. 556. (C) 557. (D) 558. (C) 559. (C) 560. (A) (D) Pulmonary function tests can be used to classify patients with chronic pulmonary disease into those with obstructive airway diseases (e.g., asthma, pulmonary emphysema, and chronic bronchitis) and those with restrictive pulmonary diseases (e.g., pulmonary fibrosis, scoliosis). The forced expiratory volume in 1 second or FEV1 is the amount of air expired in 1 second and commonly is expressed as a percentage of the forced vital capacity, or FEV1/FVC. The normal FEV1/FVC is 75% to 80%. In the presence of obstructive airway disease, FEV1 of less than 70% has mild obstruction, less than 60% has moderate obstruction and less than 50% has severe obstruction. Patients with obstructive lung disease also have a normal (asthma) or increase in (bronchitis, emphysema) total lung capacity and functional residual capacity or FRC. In the presence of restrictive pulmonary disease, FEV1 is reduced but because FVC is also reduced, the FEV1/FVC is normal. Patients with restrictive disease have a total lung capacity (TLC), FRC, and total pulmonary compliance that are reduced. In patients with pulmonary emphysema, lung compliance is increased because the elastic recoil of the lungs is decreased (Miller: Anesthesia, ed 6, pp 999-1010; Stoelting: Basics of Anesthesia, ed 5, pp 406-411).
question
561. Weakness of all muscles below the knee 562. Foot drop; loss of dorsal extension of the toes 563. Weakness of the muscles that extend the knee 564. Inability to adduct the leg; diminished sensation over the medial side of the thigh 565. Most commonly caused by placement of patient into the lithotomy position 566. Numbness over the lateral aspect of the thigh A. Sciatic nerve injury B. Common peroneal nerve injury C. Femoral nerve injury D. Obturator nerve injury E. Lateral femoral cutaneous nerve injury
answer
561. 562. (B) 563. (C) 564. (D) 565. (B) 566. (E) (A) In many cases of peripheral nerve injuries the mechanism of injury is largely unknown; however, stretching or compression of the nerves can lead to nerve ischemia and damage. In the lithotomy position, hyperflexion of the hips and/or extension of the knees can aggravate stretch of the sciatic nerve. Also in the lithotomy position, compression of the common peroneal nerve between the head of the fibula and the metal supporting frame can occur. The common peroneal nerve is the most common nerve injured in the lithotomy position. Proper padding between the metal leg braces and positioning of the legs will limit the occurrence of these injuries. The sciatic nerve provides motor function for all the skeletal muscles below the knees and sensory innervation for the lateral half of the leg and most of the foot. Injury to the common peroneal nerve, a branch of the sciatic nerve, causes a foot drop from the impaired ankle dorsiflexion and the loss of foot eversion and toe extension. Injury to the femoral or obturator nerves can occur with excessive retraction during lower abdominal surgery. The obturator nerve can also be injured during a difficult forceps vaginal delivery or by excessive flexion of the thigh to the groin. Injury to the femoral nerve will manifest as decreased extension of the knee (paresis of the quadriceps femoris muscle) and numbness over the anterior aspect of the thigh and medial/anteromedial side of the leg. The inability to adduct the leg and thigh as well as numbness over the medial side of the thigh are clinical manifestations consistent with damage to the obturator nerve. Excessive flexion of the hip on the abdomen can cause a neuropathy of the lateral femoral cutaneous nerve (sensory only) resulting in numbness of the lateral aspect of the thigh (Miller: Anesthesia, ed 6, pp 1155-1159; Stoelting: Basics of Anesthesia, ed 5, pp 292, 294, 301).
question
547. Skin lesions all appear at the same stage and at the same time 548. Ciprofloxacin for 60 days is prophylaxis for exposed patients 549. Not contagious 550. Treatment may include streptomycin, gentamicin or tetracycline 551. Treatment includes trivalent equine antitoxin 552. Three primary types: cutaneous, gastrointestinal and inhalation 553. Vaccine may prevent or greatly attenuate symptoms if given within 4 days of exposure 554. Hemorrhagic fever A. Smallpox B. Anthrax C. Plague D. Botulism E. Ebola virus
answer
547. 548. (B) 549. (D) 550. (C) 551. (D) 552. (B) (A) 553. 554. (E) (A) There are three categories of biological weapons (A,B,C). All of the diseases in this question are in the highly contagious Category A agents. Smallpox is caused by a virus (Variola major) and in 1980 was declared extinct by the World Health Organization. The incubation period was 7 to 14 days and patients with the disease presented with malaise, headache, fever. Two to 4 days later a characteristic rash develops where all lesions are at the same stage (papules, vesicles, pustules and scabs). Exposed patients and health care workers who received a vaccination within 4 days of exposure had greatly attenuated symptoms. Unvaccinated patients who were untreated had a mortality rate of greater than 30%. Patients who previously had been vaccinated had a lower mortality rate. Treatment includes the drug cidofovir. Anthrax is caused by an aerobic Gram-positive spore forming bacillus (Bacillus anthracis) and has three primary forms (cutaneous, gastrointestinal, inhalational). Weaponized anthrax is mainly an inhalational disease. Inhalational anthrax symptoms occur within 1 to 7 days of exposure and initially looks like viral flu (fever, chills, myalgia, and a non-productive cough). Later on, the patient's mediastinal lymph nodes, where the spores germinate, enlarge, producing a widened mediastinum that can be seen on a chest x-ray film. Treatment is primarily with ciprofloxacin, prophylaxis to exposed personnel includes 60 days of ciprofloxacin. Mortality rate for inhaled anthrax is greater than 80%. Plague is caused by a Gram-negative coccobacillus (Yersinia pestis) and has two forms, bubonic and pneumonic. With the more common bubonic plague there is painful swelling of the lymph nodes (buboes) that can grow to 5 to 10 cm in diameter. The patients develop cyanosis, shock and gangrene in peripheral tissues (black death). If the lungs become infected, pneumonic plague develops, which if untreated has 100% mortality. Treatment is primarily with streptomycin, although gentamicin, tetracycline and chloramphenicol have been used. Botulism is caused by the toxin from Clostridium botulinum. Because this disease is due to a neurotoxin, it is not contagious. The neurotoxin affects cholinergic neurons and prevents the release of acetylcholine. Symptoms typically develop within 12 to 36 hours of exposure and include acute flaccid paralysis, decrease salivation, ileus and urinary retention. There are no sensory deficits. With appropriate supportive care and trivalent equine antitoxin, the mortality rate is less than 5%. Without the use of antitoxin, patients may take 2 to 8 weeks to recover. Mortality rate is 5% to 10%. There are more than 18 hemorrhagic fever viruses including the Ebola virus. The incubation period is 2 to 21 days and the patients present with fever, myalgias, headaches, thrombocytopenia and hemorrhagic complications (petechiae, ecchymosis). Untreated, the mortality rate for Ebola is 90%. Treatment includes the drug ribavirin (Barash: Clinical Anesthesia, ed 5, pp 1529-1533; Miller: Anesthesia, ed 6, pp 2516-2523; Stoelting: Basics of Anesthesia, ed 5, pp 621-626).
Get an explanation on any task
Get unstuck with the help of our AI assistant in seconds
New